You are on page 1of 100

COMBINATIONS

REVISION:1984

26 MARCH 2024
AZIZ MANVA
AZIZMANVA@GMAIL.COM

ALL RIGHTS RESERVED


Get all the files at: https://bit.ly/azizhandouts
Aziz Manva (azizmanva@gmail.com)

TABLE OF CONTENTS
TABLE OF CONTENTS ................................. 2 §2. COMBINATIONS .................................. 52
§1. THE HANDSHAKE FORMULA ............. 3 §2.1 Combinations 52
§2.2 Counting Numbers 65
§1.1 Overcounting: Handshakes and Pairs 3 §2.3 Standard Restrictions 68
§1.2 Rotational Symmetry 16 §2.4 Algebraic Applications 91
§1.3 2D Geometry 19 §2.5 Geometric Applications 96
§1.4 3D Geometry 34 §2.6 Equations/Further Topics 100

P a g e 2 | 100
Get all the files at: https://bit.ly/azizhandouts
Aziz Manva (azizmanva@gmail.com)

§1. THE HANDSHAKE FORMULA


§1.1 Overcounting: Handshakes and Pairs
§1.1.A Handshakes

Example 1.1: Enumeration


A group of five friends (𝐴, 𝐵, 𝐶, 𝐷, 𝐸) goes for a picnic. At the end of the picnic, they all shake each other’s hands.
List the handshakes that will take place. How many handshakes take place?

Consider the handshakes that A will get into, which is with everybody:
𝐴𝐵, 𝐴𝐶, 𝐴𝐷, 𝐴𝐸

When we get to B, we do not look back, since 𝐵𝐴 is the same as 𝐴𝐵, so we only have 3 additional handshakes:
𝐵𝐶, 𝐵𝐷, 𝐵𝐸
Similarly for C and D, we have:
𝐶𝐷, 𝐶𝐸
𝐷𝐸

And there are no handshakes for E that have not already been counted.

We can arrange the above handshakes into a beautiful triangular pattern:


𝐴𝐵, 𝐴𝐶, 𝐴𝐷, 𝐴𝐸
𝐵𝐶, 𝐵𝐷, 𝐵𝐸
𝐶𝐷, 𝐶𝐸
𝐷𝐸

Example 1.2: Logical Counting


A group of five friends goes for a picnic. At the end of the picnic, they all shake each other’s hands. How many
handshakes take place?

Counting Decreasing Handshakes


First person will shake hands with 4 people
Second person cannot shake hands with the first person anymore, since that has already been counted. He
shakes with only three people.
Third person shakes hands with two people
Fourth person shakes hand with one person

𝑇𝑜𝑡𝑎𝑙 𝐻𝑎𝑛𝑑𝑠ℎ𝑎𝑘𝑒𝑠 = ⏟
4 + ⏟
3 + ⏟
2 + ⏟
1 + ⏟
0 = 10
𝑭𝒊𝒓𝒔𝒕 𝑺𝒆𝒄𝒐𝒏𝒅 𝑻𝒉𝒊𝒓𝒅 𝑭𝒐𝒖𝒓𝒕𝒉 𝑭𝒊𝒇𝒕𝒉
𝑷𝒆𝒓𝒔𝒐𝒏 𝑷𝒆𝒓𝒔𝒐𝒏 𝑷𝒆𝒓𝒔𝒐𝒏 𝑷𝒆𝒓𝒔𝒐𝒏 𝑷𝒆𝒓𝒔𝒐𝒏

Example 1.3
There are 20 cities in a certain country. Every pair of cities is connected by an air route. How many air routes
are there? (NMTC Primary-Final, 2004/3)

Method I
Name the cities
𝐴1 , 𝐴2 , 𝐴3 , … , 𝐴20

P a g e 3 | 100
Get all the files at: https://bit.ly/azizhandouts
Aziz Manva (azizmanva@gmail.com)

You can take


19 𝐹𝑙𝑖𝑔ℎ𝑡𝑠 𝑓𝑟𝑜𝑚 𝑡ℎ𝑒 𝐴1
From the second city, you can also 19 flights. But the flight 𝐴2 𝐴1 was already counted above. Hence, we will get
18 𝑎𝑑𝑑𝑖𝑡𝑖𝑜𝑛𝑎𝑙 𝑓𝑙𝑖𝑔ℎ𝑡𝑠 𝑓𝑟𝑜𝑚 𝐴2

Continuing this way, we get:


19 × 20
1 + 2 + ⋯ + 19 = = 190
2
Method II
From each city, you have 19 𝑓𝑙𝑖𝑔ℎ𝑡𝑠. Hence, the total number of air routes should be:
19 × 20
However, a flight from 𝐴1 to 𝐴2 operates on the same air route as the flight from 𝐴2 to 𝐴1 :
19 × 20
= 190
2

Example 1.4
Consider the first five natural numbers 1,2,3,4,5. This set of five numbers is divided into two sets A and B, where
𝐴 contains two numbers, and 𝐵 contains the other three numbers. One example is 𝐴 = {2,4}, and 𝐵 = {1,3,5}.
How many such pairs of 𝐴, 𝐵 are there? (NMTC Primary-Final, 2004/7)

Method I
Once we select A, the numbers in B are automatically selected. Hence, we only count the possible sets that you
can make for A:
{1,2}, {1,3}, {1,4}, {1,5}, {2,3}, {2,4}, {2,5}, {3,4}, {3,5}, {4,5}
Method II
4×5
= 10
2
§1.1.B Handshakes Formula
When you shake hands, it’s very important to note that
𝐴 𝑠ℎ𝑎𝑘𝑖𝑛𝑔 ℎ𝑎𝑛𝑑𝑠 𝑤𝑖𝑡ℎ 𝐵 ⇒ ⏟
⏟ 𝐵 𝑠ℎ𝑎𝑘𝑖𝑛𝑔 ℎ𝑎𝑛𝑑𝑠 𝑤𝑖𝑡ℎ 𝐴
𝑬𝒗𝒆𝒏𝒕 𝑿 𝑬𝒗𝒆𝒏𝒕 𝒀
Event 𝑋 and Event 𝑌 are actually the same event. So, if we want to count the number of handshakes, we need to
find
𝑁𝑜. 𝑜𝑓 𝑃𝑒𝑜𝑝𝑙𝑒 × 𝑁𝑜. 𝑜𝑓 𝐻𝑎𝑛𝑑𝑠 𝑠ℎ𝑎𝑘𝑒𝑛 𝑏𝑦 𝑒𝑎𝑐ℎ 𝑝𝑒𝑟𝑠𝑜𝑛
2

Example 1.5
A group of five friends goes for a picnic. At the end of the picnic, they all shake each other’s hands. How many
handshakes take place?
Counting Total Handshakes and Correcting Overcounting
We can count the total number of handshakes that happen in the group.
𝐸𝑎𝑐ℎ 𝑝𝑒𝑟𝑠𝑜𝑛 𝑠ℎ𝑎𝑘𝑒𝑠 ℎ𝑎𝑛𝑑𝑠 𝑤𝑖𝑡ℎ 4 𝑝𝑒𝑜𝑝𝑙𝑒
𝑇𝑜𝑡𝑎𝑙 𝐻𝑎𝑛𝑑𝑠ℎ𝑎𝑘𝑒𝑠 = 5 × 4 = 20
But, the above calculation overcounts the actual number of handshakes by a factor of two, since A shaking
hands with B, is the same as B shaking hands with A.
Hence, to find the actual number of handshakes, we need to divide by 2:
20
= 10
2

P a g e 4 | 100
Get all the files at: https://bit.ly/azizhandouts
Aziz Manva (azizmanva@gmail.com)

1.6: Counting Handshakes among 𝒏 people


The number of handshakes when 𝒏 people all shake hands with each other is:
𝒏(𝒏 − 𝟏)
𝟐
Add up Handshakes
𝑛(𝑛 − 1)
(𝑛 − 1) + ⏟
⏟ (𝑛 − 2) + ⏟
(𝑛 − 3) + ⋯ + ⏟
1 + ⏟
0 =
(𝒏−𝟏)𝒕𝒉
2
𝑭𝒊𝒓𝒔𝒕 𝑺𝒆𝒄𝒐𝒏𝒅 𝑻𝒉𝒊𝒓𝒅 𝒏𝒕𝒉
𝑷𝒆𝒓𝒔𝒐𝒏 𝑷𝒆𝒓𝒔𝒐𝒏 𝑷𝒆𝒓𝒔𝒐𝒏 𝑷𝒆𝒓𝒔𝒐𝒏 𝑷𝒆𝒓𝒔𝒐𝒏

Correct for Overcounting


There are 𝑛 people in the group. Each person shakes hand with the remaining 𝑛 − 1 people, giving a total of
𝑛 × (𝑛
⏟ ⏟ − 1) = 𝑛(𝑛 − 1)
𝑷𝒆𝒐𝒑𝒍𝒆 𝑯𝒂𝒏𝒅𝒔𝒉𝒂𝒌𝒆𝒔
Since this overcounts the number of handshakes by a factor of two, the actual number of handshakes is:
𝑛(𝑛 − 1)
2

Example 1.7
How many handshakes will there be in a party with 14 people, if everyone shakes hands with everyone else?

Each person will shake hands with 13 other people. Since, Person A shaking hands with Person B, is the same as
Person B shaking hands with Person A, we are overcounting by a factor of two.
14 × 13
= 91
2

Example 1.8: Multiplication Principle


Tyler has entered a buffet line in which he chooses one kind of meat, two different vegetables and one dessert. If
the order of food items is not important, how many different meals might he choose?
Meat: beef, chicken, pork
Vegetables: baked beans, corn, potatoes, tomatoes
Dessert: brownies, chocolate cake, chocolate pudding, ice cream (AMC 8 2001/14)

Method I: Handshake Formula


Use the multiplication principle.
4×3
𝑁𝑢𝑚𝑏𝑒𝑟 𝑜𝑓 𝑀𝑒𝑎𝑙𝑠 = ⏟
3 × × ⏟
4 = 3 × 6 × 4 = 72
𝑪𝒉𝒐𝒊𝒄𝒆𝒔
⏟2 𝑪𝒉𝒐𝒊𝒄𝒆𝒔
𝒇𝒐𝒓 𝑴𝒆𝒂𝒕 𝑪𝒉𝒐𝒊𝒄𝒆𝒔 𝒇𝒐𝒓 𝒇𝒐𝒓 𝑫𝒆𝒔𝒔𝒆𝒓𝒕
𝑽𝒆𝒈𝒆𝒕𝒂𝒃𝒍𝒆𝒔

Method II: Combinations


3
𝐶ℎ𝑜𝑜𝑠𝑒 1 𝑀𝑒𝑎𝑡 𝑜𝑢𝑡 𝑜𝑓 3 = ( )
1
4
𝐶ℎ𝑜𝑜𝑠𝑒 2 𝑉𝑒𝑔𝑒𝑡𝑎𝑏𝑙𝑒𝑠 𝑜𝑢𝑡 𝑜𝑓 4 = ( )
2
4
𝐶ℎ𝑜𝑜𝑠𝑒 1 𝐷𝑒𝑠𝑠𝑒𝑟𝑡 𝑜𝑢𝑡 𝑜𝑓 4 = ( )
1

And since we need to make all the choices independently, we multiply them all:
3 4 4 3! 4! 4!
( )×( )×( )= × × = 3 × 6 × 4 = 72
1 2 1 1! 2! 2! 2! 1! 3!

P a g e 5 | 100
Get all the files at: https://bit.ly/azizhandouts
Aziz Manva (azizmanva@gmail.com)

Example 1.9: Multiplication Principle versus Overcounting


The chess club wants to send its two best players to represent the club at the district tourney. It has identified
star players to choose from, of which there are seven. In how many ways can the coach make the selection:
A. If the players have to be identified as 𝑓𝑖𝑟𝑠𝑡 𝑠𝑒𝑒𝑑, and 𝑠𝑒𝑐𝑜𝑛𝑑 𝑠𝑒𝑒𝑑.
B. If only the two best players have to be identified.
Part A
Since the player are identified as first seed and second seed, order is important, and hence, we will use the
multiplication principle.
For example

(𝑃𝑙𝑎𝑦𝑒𝑟
⏟ 𝐴, ⏟
𝑃𝑙𝑎𝑦𝑒𝑟 𝐵 ) ≠ (𝑃𝑙𝑎𝑦𝑒𝑟
⏟ 𝐵, ⏟
𝑃𝑙𝑎𝑦𝑒𝑟 𝐴 )
𝑭𝒊𝒓𝒔𝒕 𝑺𝒆𝒆𝒅 𝑺𝒆𝒄𝒐𝒏𝒅 𝑺𝒆𝒆𝒅 𝑭𝒊𝒓𝒔𝒕 𝑺𝒆𝒆𝒅 𝑺𝒆𝒄𝒐𝒏𝒅 𝑺𝒆𝒆𝒅
Using the multiplication principle, the number of ways to select the players if they need to be identified:

7 × ⏟ 6 = 42
𝑭𝒊𝒓𝒔𝒕 𝑺𝒆𝒄𝒐𝒏𝒅
𝑺𝒆𝒆𝒅 𝑺𝒆𝒆𝒅

Part B
If we don’t identify the players by seeds, we are overcounting by the number of ways that two players can be
arranged among themselves, which is 2.
For example:

(𝑃𝑙𝑎𝑦𝑒𝑟
⏟ 𝐴, ⏟
𝑃𝑙𝑎𝑦𝑒𝑟 𝐵 ) = (𝑃𝑙𝑎𝑦𝑒𝑟
⏟ 𝐵, ⏟
𝑃𝑙𝑎𝑦𝑒𝑟 𝐴 )
𝑭𝒊𝒓𝒔𝒕 𝑺𝒆𝒆𝒅 𝑺𝒆𝒄𝒐𝒏𝒅 𝑺𝒆𝒆𝒅 𝑭𝒊𝒓𝒔𝒕 𝑺𝒆𝒆𝒅 𝑺𝒆𝒄𝒐𝒏𝒅 𝑺𝒆𝒆𝒅
Hence, the number of ways to select two players from seven is:
7×6
= 21
2
§1.1.C Back Calculations
It is possible to calculate the number of people, given the number of handshakes, by substituting it into the
formula. Rather, than solving an equation a guess and check method is usually faster.

Example 1.10
A. In the 𝐵𝐼𝐺 𝑁, a middle school football conference, each team plays every other team exactly once. If a
total of 21 conference games were played during the 2012 season, how many teams were members of
the 𝐵𝐼𝐺 𝑁 conference? (AMC 8 2012/14)
B. At the end of a match between two teams, the players, the two coaches, and the two referees all shook
hands with each other. If the total number of handshakes was 55, what is the number of players?

Part A
The number of games between two teams is the same as the number of pairs that can be formed.
To form a pair, suppose that at the beginning of each match, the two opposing captains shake hands.

Then, the number of games is the same as the number of handshakes that can take place.

Let there be 𝑛 teams. Then, the number of games is:


𝑛(𝑛 − 1)
= 21 ⇒ 𝑛(𝑛 − 1) = 42 = 6 × 7
2
This is a quadratic. But it is better to solve by factoring the RHS:
𝑛(𝑛 − 1) = 42 = 6 × 7 ⇒ 𝑛 = 7
Part B
We first find the number of people:

P a g e 6 | 100
Get all the files at: https://bit.ly/azizhandouts
Aziz Manva (azizmanva@gmail.com)

𝑛(𝑛 − 1)
= 55 ⇒ 𝑛 = 𝑁𝑜. 𝑜𝑓 𝑃𝑒𝑜𝑝𝑙𝑒 = 11
2
From the people, we subtract the non-players (two coaches, and two referees = 4):
𝑁𝑜. 𝑜𝑓 𝑃𝑙𝑎𝑦𝑒𝑟𝑠 = 𝑛 − 4 = 11 − 4 = 7
§1.1.D Forming Pairs
Again, the concept of forming pairs is directly related to the concept of handshakes, since we can imagine that
each pair that is formed shakes hands once.

More difficult questions can provide a real scenario life that will require careful reading to
➢ Identify the parts of the question that matter
➢ Ignore the parts of the question that extra information and/or flavor text

Further, the question can also make minor variations on what is being asked to ensure that you have read it
carefully.

Example 1.11: Dinner in Pairs


A group of nine mutual friends who all get into MIT decides to celebrate by going out for dinner. Since they want
to prolong the celebration, they only go out for dinner in pairs. How many dinners will be needed to ensure that
everyone has dinner in a pair with everyone else possible?

Assume that at the start of every dinner, the pair going for the dinner shakes hands.
9×8
𝑁𝑜. 𝑜𝑓 𝐷𝑖𝑛𝑛𝑒𝑟𝑠 = 𝑁𝑜. 𝑜𝑓 𝐻𝑎𝑛𝑑𝑠ℎ𝑎𝑘𝑒𝑠 = = 36
2

Example 1.12: Study Pairs


A school teacher forms study pairs in a history class of 31 students. Every pair has two students. What is the
number of distinct study pairs that can be formed?

31 × 30
𝑁𝑜. 𝑜𝑓 𝑆𝑡𝑢𝑑𝑦 𝑃𝑎𝑖𝑟𝑠 = 𝑁𝑜. 𝑜𝑓 𝐻𝑎𝑛𝑑𝑠ℎ𝑎𝑘𝑒𝑠 = = 465
2

Example 1.13
8 long-lost friends meet and hug each other. How many hugs will take place?

If a group of friends hugs each other, the number of hugs will be the same as the number of handshakes, since
we are still forming pairs.

Imagine that the friends shake hands before exchanging hugs. Then:
8×7
𝑁𝑜. 𝑜𝑓 𝐻𝑢𝑔𝑠 = 𝑁𝑜. 𝑜𝑓 𝐻𝑎𝑛𝑑𝑠ℎ𝑎𝑘𝑒𝑠 = = 28
2

Challenge 1.14: Standing guard


Frodo Baggins, Samwise Gamgee, Merry Brandybuck, and Pippin Took are four hobbits on a quest in dangerous
lands to rid the world from the 𝑂𝑛𝑒 𝑅𝑖𝑛𝑔. They travel by day and set up camp by night. One of them stands
guard the first half, and another the second half of the night. For this question, the order of standing guard in a
single night does not matter.
A. Which is the first night where the pair standing guard must repeat?
B. The 𝐹𝑒𝑙𝑙𝑜𝑤𝑠ℎ𝑖𝑝 𝑜𝑓 𝑡ℎ𝑒 𝑅𝑖𝑛𝑔 consists of the four aforementioned hobbits, Gandalf, Aragorn, the dwarf
Gimli, the elf Legolas, and Boromir. Answer the same question as before.

P a g e 7 | 100
Get all the files at: https://bit.ly/azizhandouts
Aziz Manva (azizmanva@gmail.com)

Imagine that at the change of guard, the outgoing guard shakes the hand of the incoming guard.
Part A
4×3
𝐹𝑜𝑢𝑟 𝐻𝑜𝑏𝑏𝑖𝑡𝑠: 𝑁𝑜. 𝑜𝑓 𝐺𝑢𝑎𝑟𝑑 𝑃𝑎𝑖𝑟𝑠 = 𝑁𝑜. 𝑜𝑓 𝐻𝑎𝑛𝑑𝑠ℎ𝑎𝑘𝑒𝑠 = =6
2
We have six unique pairs, which can stand guard for six consecutive nights.
On the seventh night, the guard pair must repeat.

Part B
9×8
𝑁𝑖𝑛𝑒 𝑀𝑒𝑚𝑏𝑒𝑟𝑠: 𝑁𝑜. 𝑜𝑓 𝐺𝑢𝑎𝑟𝑑 𝑃𝑎𝑖𝑟𝑠 = 𝑁𝑜. 𝑜𝑓 𝐻𝑎𝑛𝑑𝑠ℎ𝑎𝑘𝑒𝑠 = = 36
2
We have thirty-six unique pairs, which can stand guard for thirty-six consecutive nights.
On the thirty seventh night, the guard pair must repeat.

Example 1.15
How many different pairs of positive whole numbers have a greatest common factor of 4 and a lowest common
multiple of 4620? (CEMC Gauss Grade 7 2021/24)

Since the GCF of the two numbers is 4, both numbers must be a multiple of 4. Let the numbers be:
(4𝑥, 4𝑦), 𝑥 𝑎𝑛𝑑 𝑦 𝑎𝑟𝑒 𝑐𝑜 − 𝑝𝑟𝑖𝑚𝑒

Since the numbers are co-prime


4620
𝐿𝐶𝑀(𝑥, 𝑦) = 𝑥𝑦 = = 1,155 = 3 × 5 × 7 × 11
4

For each prime factor, we have two choices: factor of 𝑥, or factor of 𝑦.



2 × ⏟
2 × ⏟ 2 × ⏟ 2 = 24 = 16
𝐶ℎ𝑜𝑖𝑐𝑒𝑠 𝐶ℎ𝑜𝑖𝑐𝑒𝑠 𝑓𝑜𝑟 𝐶ℎ𝑜𝑖𝑐𝑒𝑠 𝐶ℎ𝑜𝑖𝑐𝑒𝑠
𝑓𝑜𝑟 3 𝑓𝑜𝑟 5 𝑓𝑜𝑟 7 𝑓𝑜𝑟 11

However, the pair (𝑥, 𝑦) is the same as the pair (𝑦, 𝑥) since the pairs are not ordered.

Hence, we need to divide 16 by 2 to prevent overcounting. Hence, the final answer is:
16
=8
2

Challenge 1.16
In 𝑆𝑒𝑐𝑟𝑒𝑡 𝑆𝑎𝑛𝑡𝑎, each person in the office receives a gift from someone unknown to them, and they also give a
gift back. If 𝑆𝑒𝑐𝑟𝑒𝑡 𝑆𝑎𝑛𝑡𝑎 is held every year, after how many years must the pairs who give gifts to each other
start repeating if:
A. The office has 22 people
B. The office has 𝑛 people
C. Are there any restrictions on the value of 𝑛?
22 × 21
𝑇𝑜𝑡𝑎𝑙 𝑁𝑜. 𝑜𝑓 𝑃𝑎𝑖𝑟𝑠 2 22 × 21
𝑁𝑜. 𝑜𝑓 𝑌𝑒𝑎𝑟𝑠 = = = = 21
𝑃𝑎𝑖𝑟𝑠 𝑝𝑒𝑟 𝑌𝑒𝑎𝑟 22 22
2

P a g e 8 | 100
Get all the files at: https://bit.ly/azizhandouts
Aziz Manva (azizmanva@gmail.com)

𝑛(𝑛 − 1)
𝑇𝑜𝑡𝑎𝑙 𝑁𝑜. 𝑜𝑓 𝑃𝑎𝑖𝑟𝑠 2 𝑛(𝑛 − 1)
𝑁𝑜. 𝑜𝑓 𝑌𝑒𝑎𝑟𝑠 = = 𝑛 = = 𝑛−1
𝑃𝑎𝑖𝑟𝑠 𝑝𝑒𝑟 𝑌𝑒𝑎𝑟 𝑛
2

𝑛 must be an even number. Else, one person will be left out of 𝑆𝑒𝑐𝑟𝑒𝑡 𝑆𝑎𝑛𝑡𝑎.

§1.1.E Selective Handshakes


To the base formula for handshakes, we can make the question a little more difficult by adding conditions that
increase or decrease the number of handshakes.

If certain people do not shake hands among themselves, then those handshakes will need to be subtracted in
order to find the actual count of handshakes.

𝐻𝑎𝑛𝑑𝑠ℎ𝑎𝑘𝑒𝑠𝐴𝑐𝑡𝑢𝑎𝑙 = 𝐻𝑎𝑛𝑑𝑠ℎ𝑎𝑘𝑒𝑠𝑀𝑎𝑥 𝑃𝑜𝑠𝑠𝑖𝑏𝑙𝑒 − 𝐻𝑎𝑛𝑑𝑠ℎ𝑎𝑘𝑒𝑠𝑤ℎ𝑖𝑐ℎ 𝑑𝑖𝑑 𝑛𝑜𝑡 ℎ𝑎𝑝𝑝𝑒𝑛

The formula is based on the concept of complementary counting:


𝑇𝑜𝑡𝑎𝑙 − 𝑈𝑛𝑤𝑎𝑛𝑡𝑒𝑑 = 𝑊𝑎𝑛𝑡𝑒𝑑

Example 1.17
The 𝐷𝑎𝑛𝑐𝑒 𝐼𝑛𝑑𝑖𝑎 𝐷𝑎𝑛𝑐𝑒 competition has six dancing pairs. At the end of the competition the contestants shake
hands among themselves. However, they do not shake hands with their respective partners. Find the number of
handshakes that take place.

There are six dancing pairs, which gives the number of contestants as
12
Complementary Counting
Find the number of handshakes with no restrictions, and then subtract the handshakes that would have
happened among the dancing pairs:
12 × 11
− 6 = 66 − 6 = 60
2
Direct Method
12 contestants will each shake hands with everyone, except their partner.
This means, every contestant will shake hands with 10 other people.
1
12
⏟ × 10
⏟ × = 60
⏟2
𝑵𝒐.𝒐𝒇 𝑷𝒆𝒐𝒑𝒍𝒆 𝑵𝒐.𝒐𝒇 𝑯𝒂𝒏𝒅𝒔𝒉𝒂𝒌𝒆𝒔
𝑶𝒗𝒆𝒓𝒄𝒐𝒖𝒏𝒕𝒊𝒏𝒈
𝑭𝒂𝒄𝒕𝒐𝒓

Example 1.18
A party has fourteen people. Everyone at the party shakes hands with everyone else, except their spouse. Find
the number of handshakes if:
A. The party has seven husband-wife pairs.
B. The party has five husband-wife pairs, and the rest are singles.

Since there 14 people, the number of handshakes with no restrictions


14 × 13
= = 7 × 13 = 91
2
Part A
If there are seven pairs which do not shake hands among themselves, then the number of handshakes

P a g e 9 | 100
Get all the files at: https://bit.ly/azizhandouts
Aziz Manva (azizmanva@gmail.com)

91 − 7 = 84
Part B
If there are five pairs which do not shake hands among themselves, then the number of handshakes
91 − 5 = 86

§1.1.F Location Based Restrictions

Example 1.19
At the beginning of the month, four senior citizens decide to sit on a park bench in a line every morning, and
hug the person next to them. Once they all hug each other, they celebrate by taking a walk in the park on the
same day that they completed the round of hugs. What is the minimum number of days from when they start
the process that they will take the first walk?
Count Number of Hugs
Count the number for everyone to hug each other.
4×3
𝑁𝑜. 𝑜𝑓 𝐻𝑢𝑔𝑠 = 𝑁𝑜. 𝑜𝑓 𝐻𝑎𝑛𝑑𝑠ℎ𝑎𝑘𝑒𝑠 = =6
2
We not need to count the number of handshakes/hugs, which is 6. We also need to see how many days it will
take. In particular, there is a restriction on location.
How many days does it take?
Four people sitting in a row gives us gives us 3 gaps, which means 3 hugs every
Day 1 A B C D
day.
6 𝑨𝑩, 𝑩𝑪, 𝑪𝑫
Minimum number of days = = 2 Day 2 C A D B
3
But, we don’t know whether this is achievable till we actually make them sit and 𝑪𝑨, 𝑨𝑫, 𝑫𝑩
hug each other.
{𝐴, 𝐵, 𝐶, 𝐷} ⇒ ⏟
⏟ 𝑨𝑩, 𝑨𝑪, 𝑨𝑫, 𝑩𝑪, 𝑩𝑫, 𝑪𝑫
𝑃𝑒𝑜𝑝𝑙𝑒 𝑫𝒊𝒔𝒕𝒊𝒏𝒄𝒕 𝑯𝒖𝒈𝒔

Sub-Part 1.20
If the number of senior citizens increases to five, what is the answer?
Count Number of Hugs
Count the number for everyone to hug each other.
5×4
𝑁𝑜. 𝑜𝑓 𝐻𝑢𝑔𝑠 = 𝑁𝑜. 𝑜𝑓 𝐻𝑎𝑛𝑑𝑠ℎ𝑎𝑘𝑒𝑠 = = 10
2
How many days does it take?
Five people sitting in a row gives us gives us 4 gaps, which means 4 hugs Day 1 A B C D E
every day. 𝑨𝑩, 𝑩𝑪, 𝑪𝑫, 𝑫𝑬
10 Day 2 C A D B E
Minimum number of days = = 2.5 ⇒ 𝑅𝑜𝑢𝑛𝑑 𝑈𝑝 = 3
4 𝑪𝑨, 𝑨𝑫, 𝑫𝑩, 𝑩𝑬
Day 3 A E C B D
{𝐴,
⏟ 𝐵, 𝐶, 𝐷, 𝐸} ⇒ ⏟
𝑨𝑩, 𝑨𝑪, 𝑨𝑫, 𝑨𝑬, 𝑩𝑪, 𝑩𝑫, 𝑩𝑬, 𝑪𝑫, 𝑪𝑬, 𝑫𝑬 𝑨𝑬, 𝑪𝑬
𝑃𝑒𝑜𝑝𝑙𝑒 𝑫𝒊𝒔𝒕𝒊𝒏𝒄𝒕 𝑯𝒖𝒈𝒔

§1.1.G Differential Handshakes

Example 1.21
I go to a fund-raising party, which is attended by 30 representatives from companies, 40 representatives from

P a g e 10 | 100
Get all the files at: https://bit.ly/azizhandouts
Aziz Manva (azizmanva@gmail.com)

angel investors and thirty-one representatives from venture capitalists. Find the number of handshakes if all the
𝑐𝑜𝑚𝑝𝑎𝑛𝑦
⏟ 𝑟𝑒𝑝𝑟𝑒𝑠𝑒𝑛𝑡𝑎𝑡𝑖𝑣𝑒𝑠 shake hands with all the 𝑎𝑛𝑔𝑒𝑙
⏟ 𝑖𝑛𝑣𝑒𝑠𝑡𝑜𝑟𝑠 once, and all the 𝑣𝑒𝑛𝑡𝑢𝑟𝑒
⏟ 𝑐𝑎𝑝𝑖𝑡𝑎𝑙𝑖𝑠𝑡𝑠
𝑪𝑹 𝑨𝑰 𝑽𝑪
twice. All the angel investors shake hands with the venture capitalists thrice.

30 × 40 + ⏟
⏟ 30 × 31 × 2 + ⏟
40 × 31 × 3 = 1200 + 1860 + 3720 = 6780
𝑪𝑹 𝒘𝒊𝒕𝒉 𝑨𝑰 𝑪𝑹 𝒘𝒊𝒕𝒉 𝑽𝑪 𝑨𝑰 𝒘𝒊𝒕𝒉 𝑽𝑪
Note that in the above calculation, we did not count:
➢ Angel Investors shaking hands with Company Representatives
➢ Venture Capitalists shaking hands with Company Representatives
➢ Venture Capitalists shaking hands with Angel Investors
Hence each handshake is counted only once, not twice.
And hence, we do not need to divide by two.

Example 1.22
The Little Twelve Basketball Conference has two divisions, with six teams in each division. Each team plays each
of the other teams in its own division twice and every team in the other division once. How many conference
games are scheduled? (AMC 8 2005/14)

Method I
First, calculate the number of games that each team will play.
Each division has 6 teams. So, within its division, a team has five other teams to play with. And with each of
those teams, it will play two games each, giving:

5 × ⏟
2 = 10
𝑁𝑜.𝑜𝑓 𝐺𝑎𝑚𝑒𝑠
𝑇𝑒𝑎𝑚𝑠 𝑝𝑒𝑟 𝑇𝑒𝑎𝑚
Outside the division, there are six teams, and each team will play with each of those six teams, giving:

6 × ⏟
1 =6
𝑁𝑜.𝑜𝑓 𝐺𝑎𝑚𝑒𝑠
𝑇𝑒𝑎𝑚𝑠 𝑝𝑒𝑟 𝑇𝑒𝑎𝑚
Hence, the total number of games that each team will play is:
10 + 6 = 16
Finally, the total number of games across all teams is:
16 × 12
= 8 × 12 = 96
2
(where we divided by 2, because like handshakes, Team A playing Team B is the same as Team B playing against
Team A).
Method II
Games within the first division:
6×5
2× = 30
2
Games within the second division:
6×5
2× = 30
2
Games between the divisions:
6×6
× 2 = 36
2

Example 1.23
At a carrom tournament, people compete in teams of two, and pairs in a team shake hands with their partner
once, and with everyone else three times each. How many handshakes will happen at the:
A. Final

P a g e 11 | 100
Get all the files at: https://bit.ly/azizhandouts
Aziz Manva (azizmanva@gmail.com)

B. Semi-Final
C. Quarter Final

Part A: Final 8−2×


⏟ ⏟
3 = 6 × 3 = 18
Count the number of handshakes for each person: 𝑵𝒐.𝒐𝒇 𝑯𝒂𝒏𝒅𝒔𝒉𝒂𝒌𝒆𝒔
𝑷𝒆𝒐𝒑𝒍𝒆 𝒑𝒆𝒓 𝑷𝒆𝒓𝒔𝒐𝒏
Handshakes within the team: ⏟ 1 × ⏟
1
𝑵𝒐.𝒐𝒇 𝑯𝒂𝒏𝒅𝒔𝒉𝒂𝒌𝒆𝒔
Total handshakes per person:
𝑷𝒆𝒐𝒑𝒍𝒆 𝒑𝒆𝒓 𝑷𝒆𝒓𝒔𝒐𝒏 1 + 18 = 19
=1 Total Handshakes in All
Handshakes outside the team: ⏟
4−2 × ⏟
3 19 × 8
𝑵𝒐.𝒐𝒇 𝑯𝒂𝒏𝒅𝒔𝒉𝒂𝒌𝒆𝒔
= = 76
𝒑𝒆𝒓 𝑷𝒆𝒓𝒔𝒐𝒏
2
𝑷𝒆𝒐𝒑𝒍𝒆
Part C: Quarter-Final
=2×3=6 Count the number of handshakes for each person:
Total handshakes per person Handshakes within the team:
1+6=7 ⏟
1 × ⏟
1 =1
Total Handshakes in All 𝑵𝒐.𝒐𝒇 𝑯𝒂𝒏𝒅𝒔𝒉𝒂𝒌𝒆𝒔
7×4 𝑷𝒆𝒐𝒑𝒍𝒆 𝒑𝒆𝒓 𝑷𝒆𝒓𝒔𝒐𝒏
= = 14 Handshakes outside the team: 16
⏟ −2× ⏟
3
2
Part B: Semi-Final 𝑵𝒐.𝒐𝒇 𝑯𝒂𝒏𝒅𝒔𝒉𝒂𝒌𝒆𝒔
𝑷𝒆𝒐𝒑𝒍𝒆 𝒑𝒆𝒓 𝑷𝒆𝒓𝒔𝒐𝒏
Count the number of handshakes for each person:
= 14 × 3 = 42
Handshakes within the team:
Total handshakes per person:

1 × ⏟
1 =1
𝑵𝒐.𝒐𝒇 𝑯𝒂𝒏𝒅𝒔𝒉𝒂𝒌𝒆𝒔
1 + 42 = 43
𝑷𝒆𝒐𝒑𝒍𝒆 𝒑𝒆𝒓 𝑷𝒆𝒓𝒔𝒐𝒏
Handshakes outside the team: Total Handshakes in All:
43 × 16
= = 344
2

§1.1.H Ratios
At a formal western dance party, dances will happen in pairs, with every dance having a male partner and a
female partner. These are often referred to in questions on dances.

Example 1.24
At a party, there were ten men and five women. Each man danced twice. Each woman had the same number of
dances. Dances happened only in man-woman pairs. Find:
A. The total number of dances danced by the men.
B. The total number of dances danced by the women.
C. The number of dances that each woman had.
D. The ratio of the number of dances that each woman had to the number of dances that each man had.
Compare this to the ratio of the number of women to the number of men.
Total Dances for Men
We use the multiplication principle:
𝑇𝑜𝑡𝑎𝑙 𝐷𝑎𝑛𝑐𝑒𝑠 = 10
⏟ × ⏟
2 = 20
𝑵𝒐.𝒐𝒇 𝑫𝒂𝒏𝒄𝒆𝒔
𝑴𝒆𝒏 𝒑𝒆𝒓 𝑴𝒂𝒏

Total Dances for Women


Since we only have men-women dancing pairs, the total number of dances for the women must equal the total
number of dances for the men.
𝐷𝑎𝑛𝑐𝑒𝑠 (𝑊𝑜𝑚𝑒𝑛) = 𝐷𝑎𝑛𝑐𝑒𝑠(𝑀𝑒𝑛) = 20
Total Dances

P a g e 12 | 100
Get all the files at: https://bit.ly/azizhandouts
Aziz Manva (azizmanva@gmail.com)

We use the multiplication principle, and solve for the dances per woman
𝑇𝑜𝑡𝑎𝑙 𝐷𝑎𝑛𝑐𝑒𝑠 20
𝐷𝑎𝑛𝑐𝑒𝑠 𝑝𝑒𝑟 𝑊𝑜𝑚𝑎𝑛 = = =4
𝑁𝑜. 𝑜𝑓 𝑊𝑜𝑚𝑒𝑛 5
Ratio
𝐷𝑎𝑛𝑐𝑒𝑠 𝑝𝑒𝑟 𝑊𝑜𝑚𝑎𝑛 4 2
= =
𝐷𝑎𝑛𝑐𝑒𝑠 𝑝𝑒𝑟 𝑀𝑎𝑛 2 1
𝑁𝑜. 𝑜𝑓 𝑊𝑜𝑚𝑒𝑛 5 1
= =
𝑁𝑜. 𝑜𝑓 𝑀𝑒𝑛 10 2
Note that the first ratio is the reciprocal of the second ratio. This is a general property.

§1.1.I Back Calculations

Example 1.25
At a party, each man danced with exactly three women and each woman danced with exactly two men. Twelve
men attended the party. How many women attended the party? (AMC 10 2004)

Direct Method
𝑁𝑜. 𝑜𝑓 𝐷𝑎𝑛𝑐𝑒(𝑀𝑒𝑛) = 12
⏟ × ⏟
3 = 36
𝑵𝒐.𝒐𝒇 𝑴𝒆𝒏 𝑵𝒐.𝒐𝒇 𝑫𝒂𝒏𝒄𝒆𝒔
𝒂𝒕 𝒕𝒉𝒆 𝑷𝒂𝒓𝒕𝒚 𝒃𝒚 𝒆𝒂𝒄𝒉 𝑴𝒂𝒏
No. of dances for men has to be same as total no. of dances for women
𝑁𝑜. 𝑜𝑓 𝐷𝑎𝑛𝑐𝑒(𝑊𝑜𝑚𝑒𝑛) = 36

𝑁𝑜. 𝑜𝑓 𝐷𝑎𝑛𝑐𝑒𝑠 36
𝑁𝑜. 𝑜𝑓 𝑤𝑜𝑚𝑒𝑛 = = = 18
𝐷𝑎𝑛𝑐𝑒𝑠 𝑝𝑒𝑟 𝑊𝑜𝑚𝑎𝑛 2
Ratio Method
𝐷𝑎𝑛𝑐𝑒𝑠 𝑝𝑒𝑟 𝑀𝑎𝑛 3
=
𝐷𝑎𝑛𝑐𝑒𝑠 𝑝𝑒𝑟 𝑊𝑜𝑚𝑎𝑛 2

𝑁𝑜. 𝑜𝑓 𝑀𝑒𝑛 2 12
= = ⇒ 𝑁𝑜. 𝑜𝑓 𝑊𝑜𝑚𝑒𝑛 = 18
𝑁𝑜. 𝑜𝑓 𝑊𝑜𝑚𝑒𝑛 3 18

Challenge 1.26
At a party, each man danced with exactly three women and each woman danced with the same number of men.
Twelve men attended the party. Let 𝑛 be the number of women at the party. Find the number of values that 𝑛
can take? (AMC 10 2004, Adapted)
As in the earlier example,
𝑁𝑜. 𝑜𝑓 𝐷𝑎𝑛𝑐𝑒(𝑀𝑒𝑛) = 12
⏟ × ⏟
3
𝑵𝒐.𝒐𝒇 𝑴𝒆𝒏 𝑵𝒐.𝒐𝒇 𝑫𝒂𝒏𝒄𝒆𝒔
𝒂𝒕 𝒕𝒉𝒆 𝑷𝒂𝒓𝒕𝒚 𝒃𝒚 𝒆𝒂𝒄𝒉 𝑴𝒂𝒏
Again, as in the earlier example,
𝑁𝑜. 𝑜𝑓 𝐷𝑎𝑛𝑐𝑒𝑠 36
𝑁𝑜. 𝑜𝑓 𝑤𝑜𝑚𝑒𝑛 = = = 𝑛, 𝑛 ∈ 𝑁
𝐷𝑎𝑛𝑐𝑒𝑠 𝑝𝑒𝑟 𝑊𝑜𝑚𝑎𝑛 𝐷𝑎𝑛𝑐𝑒𝑠 𝑝𝑒𝑟 𝑊𝑜𝑚𝑎𝑛

𝐷𝑎𝑛𝑐𝑒𝑠 𝑝𝑒𝑟 𝑊𝑜𝑚𝑎𝑛 can take the values of the positive factors of 36:
36 = 22 × 32 ⇒ (2 + 1)(2 + 1) = 3 × 3 = 9

But 𝑛 ≥ 3:

P a g e 13 | 100
Get all the files at: https://bit.ly/azizhandouts
Aziz Manva (azizmanva@gmail.com)

𝑁𝑜. 𝑜𝑓 𝑉𝑎𝑙𝑢𝑒𝑠 𝑜𝑓 𝑛 = 9 − 2 = 7

§1.1.J Tournaments

Example 1.27: Back Calculations


In a Go Tournament played in a Round Robin format, X withdrew after playing a single match, and Y withdrew
after playing ten matches. The total number of matches played was 55.
A. What is the number of people in the tournament?
B. What will be the number of matches, if the tournament is played again, in single elimination format,
without the people who withdrew?

Next number after 55 satisfying:


n(n + 1) = 66
n = 12
Every person was supposed to have played 11 matches.
X would have missed 10 matches
Y would have missed 1 match.
Total Matches = 55 + 10 + 1 = 66

Hence, people in 1st Tournament = 12

Number of people in 2nd Tournament = 10


Number of Matches (10 People, Single Elimination Format)
=n–1=9

§1.1.K Mixing Concepts / Extra Information

Example 1.28
What is the positive difference in the number of matches that two tournaments with 𝑛 competitors (the first
one being round-robin, the second one being single elimination) have?
𝑛(𝑛 − 1) 𝑛(𝑛 − 3) + 2
− (𝑛 − 1) =
2 2
§1.1.L Combinations
We will solve these questions using logic, and also using the concept of combinations which are covered in the
next chapter. You should come back and redo these questions using combinations after going through that
chapter.

Example 1.29
How many triangles can be formed from the vertices of a heptagon?

Overcounting
To make a triangle, you need to select three of the seven vertices. You can do this in

7 × ⏟ 6 × ⏟ 5 𝑤𝑎𝑦𝑠
𝑭𝒊𝒓𝒔𝒕 𝑺𝒆𝒄𝒐𝒏𝒅 𝑻𝒉𝒊𝒓𝒅
𝑽𝒆𝒓𝒕𝒆𝒙 𝑽𝒆𝒓𝒕𝒆𝒙 𝑽𝒆𝒓𝒕𝒆𝒙
But, this overcounts by the number of ways the vertices can be arranged among themselves. So, correct for this
by dividing by 3!:

P a g e 14 | 100
Get all the files at: https://bit.ly/azizhandouts
Aziz Manva (azizmanva@gmail.com)

7×6×5 7×𝟔×5
= = 35
3! 𝟔
Combinations
From the seven vertices, we need to choose any three:
7 7×6×5 7×𝟔×5
7 𝐶ℎ𝑜𝑜𝑠𝑒 3 = ( ) = = = 35
3 3! 𝟔

Example 1.30
Ten distinct points are identified on the circumference of a circle. How many different convex quadrilaterals can
be formed if each vertex must be one of these 10 points? (MathCounts 2005 Warm-Up 7)

Overcounting
To make a quadrilateral, you need to select four of the ten vertices. You can do this in
10
⏟ × ⏟ 9 × ⏟ 8 × ⏟ 7 𝑤𝑎𝑦𝑠
𝑭𝒊𝒓𝒔𝒕 𝑺𝒆𝒄𝒐𝒏𝒅 𝑻𝒉𝒊𝒓𝒅 𝑻𝒉𝒊𝒓𝒅
𝑽𝒆𝒓𝒕𝒆𝒙 𝑽𝒆𝒓𝒕𝒆𝒙 𝑽𝒆𝒓𝒕𝒆𝒙 𝑽𝒆𝒓𝒕𝒆𝒙
But, this overcounts by the number of ways the vertices can be arranged among themselves. So, correct for this
by dividing 4!:
10 × 9 × 8 × 7
4!
Combinations
From the ten vertices, we need to choose any four:
10 10! 10 × 9 × 8 × 7
( )= =
4 4! 6! 4!

Example 1.31
A teacher has made ten statements for a True-False test. Four statements are true and six are false. How many
distinct answer keys could there be for the test? (MathCounts 2003 Counting/Combinatorics Stretch)

Overcounting
We need to decide the position of the True answers among the ten answers. For this, we have:
10
⏟ × ⏟ 9 × ⏟ 8 × ⏟ 7 𝑤𝑎𝑦𝑠
𝐹𝑖𝑟𝑠𝑡 𝑆𝑒𝑐𝑜𝑛𝑑 𝑇ℎ𝑖𝑟𝑑 𝑇ℎ𝑖𝑟𝑑
𝐴𝑛𝑠𝑤𝑒𝑟 𝐴𝑛𝑠𝑤𝑒𝑟 𝐴𝑛𝑠𝑤𝑒𝑟 𝐴𝑛𝑠𝑤𝑒𝑟
However, we are not interested in the way that these four answers can be arranged among themselves, since we
cannot distinguish one False answer from another. Hence, we are overcounting by a factor of 4!;
Combinations
Imagine that True is represented by T, and False is represented by F.
We need arrange the T’s and the F’s in the answer key. Once we choose the position of the F’s, there is no choice
left for the T’s.

Hence, we need to count the number of ways to select the position of four F’s among ten places:
10 10! 10 × 9 × 8 × 7 10 × 9 × 8 × 7
( )= = = 210
4 4! 6! 4! 4!

§1.1.M Review and Challenge

Challenge 1.32
A mathematical organization is producing a set of commemorative license plates. Each plate contains a

P a g e 15 | 100
Get all the files at: https://bit.ly/azizhandouts
Aziz Manva (azizmanva@gmail.com)

sequence of five characters chosen from the four letters in AIME and the four digits in 2007. No character may
appear in a sequence more times than it appears among the four letters in AIME or the four digits in 2007. A set
of plates in which each possible sequence appears exactly once contains N license plates. Find N/10. (AIME
2007/II/1)

We will do this using casework. There are two possible cases:


➢ Either no or one zero is used in the license plate
✓ This can be done using the multiplication principle
➢ Two zeros are used. In this case, we will place the zeros first.

Case I: No zero or One Zero is Used


We have seven alphanumeric digits to choose from, out of which we want to arrange five digits, which can be
done, using the multiplication principle in

7 × ⏟ 6 × ⏟ 5 × ⏟ 4 × ⏟ 3 = 2,520 𝑊𝑎𝑦𝑠
𝐹𝑖𝑟𝑠𝑡 𝑆𝑒𝑐𝑜𝑛𝑑 𝑆𝑒𝑐𝑜𝑛𝑑 𝑆𝑒𝑐𝑜𝑛𝑑 𝑆𝑒𝑐𝑜𝑛𝑑
𝐷𝑖𝑔𝑖𝑡 𝐷𝑖𝑔𝑖𝑡 𝐷𝑖𝑔𝑖𝑡 𝐷𝑖𝑔𝑖𝑡 𝐷𝑖𝑔𝑖𝑡

Case II: Two Zeros Used


There are five spots
⏟ ⏟ ⏟ ⏟ ⏟

Of which we need to pick a pair for placing our two zeroes. This can be done in
5×4
= 10 𝑊𝑎𝑦𝑠
2
The remaining three digits can be filled in
6 × 5 × 4 = 120 𝑊𝑎𝑦𝑠
Giving us a total of
10 × 120 = 1200 𝑊𝑎𝑦𝑠
Final Answer
The total ways for both the cases are:
𝑁 3720
𝑁 = 2520 + 1200 = 3720 ⇒ = = 372
10 10

§1.2 Rotational Symmetry


§1.2.A Shaking hands with yourself
In this section, we will count the number of tiles in a domino set. A domino question is similar to the handshake
problem, since you have to make pairs of numbers. But, it is also different since in a domino you can pair a
number with itself, whereas in handshakes, a person will not shake hands with himself.

Example 1.33
You go to a party with 10 people, including yourself. Everyone shakes hands with everyone else (they are highly
sociable). They also shake hands with themselves (you are very confused, and you 𝑑𝑜 𝑛𝑜𝑡 shake hands with
yourself). What is the number of handshakes that take place?

10 × 9
+ 10 − 1 = 45 + 10 − 1 = 54
2

P a g e 16 | 100
Get all the files at: https://bit.ly/azizhandouts
Aziz Manva (azizmanva@gmail.com)

§1.2.B Background Information: Dominos


What is a domino
Dominoes is a game played with two squares, joined end to end. We will not be
interested in the rules of the game. Rather, we will be interested in counting the
number of tiles needed to form a complete set.
For our purposes, we will consider that that the back of the looks the same for all
tiles. That is, the dominoes are identical when viewed from the back.
Dots on Dominos
Each of the squares on a domino has a number of dots on it. The number of dots
starts at 0, and goes up to different numbers in different sets. A standard
𝑆𝑖𝑛𝑜 𝐸𝑢𝑟𝑜𝑝𝑒𝑎𝑛 set of dominoes has 0 to 6 dots, and 28 dominoes. The
illustration alongside shows all 28 dominoes in the set.
The next example provides a method of counting the number of tiles in a set of
dominos.
Rotational Symmetry
Dominoes show rotational symmetry, as can be seen form the illustration on the
right.

Example 1.34
What is the order of rotational symmetry for a domino?

§1.2.C Sino European Set

Example 1.35: Tabulation Method


The illustration alongside shows the dominoes in a traditional Sino European set.
A. How many dominoes does it have?
B. Tabulate the values as well.

First Valid Pairs No. of


Dot Pairs
6 (6,6)(6,5)(6,4)(6,3)(6,2)(6,1)(6,0) 7
5 (5,5)(5,4)(5,3)(5,2)(5,1)(5,0) 6
4 (4,4)(4,3)(4,2)(4,1)(4,0) 5
3 (3,3)(3,2)(3,1)(3,0) 4
Example 1.36: 2 (2,2)(2,1)(2,0) 3 Forming Pairs
In a traditional Sino- 1 (1,1)(1,0) 2 European domino set,
each square in the front 0 (0,0) 1 of the domino has a
number of dots ranging from zero to six. It is valid to pair a number on the left square with the same number on
the right square. Find the number of distinct dominos in the set.

Since dominos have rotational symmetry, the dominoes


(0, 7) 𝑎𝑛𝑑 (7,0)
Are the same, by rotation.

Hence, we need to find the number of ways that we can make pairs of the numbers from
7×6
{0,1,2,3,4,5,6} ⇒ 7 𝑁𝑢𝑚𝑏𝑒𝑟𝑠 ⇒ = 21 𝐷𝑜𝑚𝑖𝑛𝑜𝑠
2

P a g e 17 | 100
Get all the files at: https://bit.ly/azizhandouts
Aziz Manva (azizmanva@gmail.com)

But, doubles are also allowed in dominoes:


(0,0), (1,1), … (6,6) ⇒ 7 𝐷𝑜𝑚𝑖𝑛𝑜𝑒𝑠

And hence the total is


21 + 28

Example 1.37: Alternate Solution


We redo the example above using a table method that helps visualize the number of dominoes.

If we had to count just handshakes, we 0 1 2 3 4 5 6 Handshakes Dominoes


would get: 0 0 1
7×6 1 1 2
= 21
2 2 2 3
7×6 3 3 4
𝑆ℎ𝑜𝑟𝑡𝑐𝑢𝑡: + 7 = 21 + 7 = 28 𝑊𝑎𝑦𝑠
2 4 4 5
5 5 6
6 6 7

Total 7×6 28
= 21
2

1.38: Number of Dominoes


The number of distinct dominoes where the dots are from 0 to n is:
(𝑛 + 1)(𝑛 + 2)
2

(𝑛 + 1)(𝑛) 𝑛2 − 𝑛 + 2𝑛 𝑛2 + 𝑛 𝑛(𝑛 + 1)
+𝑛= = =
2 2 2 2

§1.2.D Other Sets

Example 1.39
Domino sets that have all possible combinations of dots from 0 to 9 are commercially available. What is the
number of distinct dominos in such sets?

The number of possible dots are:


0,1, … 9 ⇒ 10 𝑂𝑝𝑡𝑖𝑜𝑛𝑠
The number of pairs of distinct number of dots that can be formed is
10 × 9
𝑁𝑜. 𝑜𝑓 𝑃𝑎𝑖𝑟𝑠 = = 45 𝑇𝑖𝑙𝑒𝑠
2
And, the number of doubles is:
(0,0)(1,1), … (9,9) ⇒ 10 𝑇𝑖𝑙𝑒𝑠

And, hence the total number of tiles is:


45 + 10 = 55

Example 1.40
Like Sino European dominoes, Chinese dominoes also have two adjacent identical squares. Find the number of
tiles in a Chinese domino set given that:

P a g e 18 | 100
Get all the files at: https://bit.ly/azizhandouts
Aziz Manva (azizmanva@gmail.com)

A. Sino European dominos have 0 to 6 dots on a square. Chinese dominoes have dots from 1 to 6.
Dominoes exist for every pair/combination that can be made using the dots from 1 to 6. (Order is not
important).
B. 11 tiles are repeated (occur twice) in a Chinese set. No repetitions occur in a Sino European set.

The number of possible dots are:


1,2, … ,6 ⇒ 6 𝑂𝑝𝑡𝑖𝑜𝑛𝑠
The number of pairs of distinct number of dots that can be formed is
6×5
𝑁𝑜. 𝑜𝑓 𝑃𝑎𝑖𝑟𝑠 = = 15 𝑇𝑖𝑙𝑒𝑠
2
And, the number of doubles is:
(1,1), (2,2) … (6,6) ⇒ 6 𝑇𝑖𝑙𝑒𝑠

And, hence the total number of tiles should be:


15 + 6 = 21

But, we have 11 repeated tiles, and hence the final answer is:
21 + 11 = 32

§1.2.E Using the Multiplication Principle

Example 1.41
A domino consists of two squares of the same size joined end to end, each of which has numbers from zero to
nine. The dots on the domino come in three colours (black, green and red). All dots representing a number are
the same color. The back of the domino is either striped or plain. A set of dominos consists of all the possible
distinct dominoes. Each domino costs $1, and the seller wants to make a 20% profit on his cost. Find the selling
price of a set of dominoes.

9 × 10

2 × ⏟
3 × + 11 = 6 × 55 = 330
𝑶𝒑𝒕𝒊𝒐𝒏𝒔 𝑵𝒐.𝒐𝒇
⏟2
𝒇𝒐𝒓 𝑩𝒂𝒄𝒌 𝑪𝒐𝒍𝒐𝒖𝒓𝒔 𝑵𝒖𝒎𝒃𝒆𝒓 𝒐𝒇
𝑫𝒐𝒎𝒊𝒏𝒐𝒆𝒔
𝑆𝑒𝑙𝑙𝑖𝑛𝑔 𝑃𝑟𝑖𝑐𝑒 = 330
⏟ × ⏟
1 × 1.2
⏟ = 396
𝑵𝒐.𝒐𝒇 𝑪𝒐𝒔𝒕 𝒑𝒆𝒓 𝑨𝒅𝒅 𝟐𝟎%
𝑫𝒐𝒎𝒊𝒏𝒐𝒆𝒔 𝑫𝒐𝒎𝒊𝒏𝒐𝒆𝒔

§1.2.F Trominoes

Example 1.42

§1.3 2D Geometry
Our objective in this section is to apply counting techniques to geometrical objects. Along the way, we will
learn/revise a few geometrical concepts, especially when dealing with three dimensional objects. The focus in
this section is as much on the geometry as it is on the counting. The ability to visualize complex three-
dimensional shapes is both useful and necessary for this section.
§1.3.A Counting with lines and line segments

1.43: Number of Line Segments formed by 𝒏 distinct points


The number of line segments that can be formed by 𝑛 distinct points on a line is:

P a g e 19 | 100
Get all the files at: https://bit.ly/azizhandouts
Aziz Manva (azizmanva@gmail.com)

𝑛(𝑛 − 1)
2

This is the same as the number of handshakes that can take place 𝑛 people, since
➢ Every point can get to remaining 𝑛 −1 points giving us
𝑛 − 1 𝑙𝑖𝑛𝑒 𝑠𝑒𝑔𝑚𝑒𝑛𝑡𝑠
➢ Line AB is the same as line segment BA. Hence, to find the final answer, we need to divide by 2,
giving us:
𝑛(𝑛 − 1)
2

Example 1.44: Counting Line Segments


A. Find the number of line segments that can be formed in a line that distinct three points on it.
B. Find the number of distinct line segments that can be formed in a line with ten points on it. A single
point can be used in more than one line segment.
C. I have 𝑛 points, which form 21 distinct line segments when they are connected to each other. Find the
range of values that 𝑛 can take.

Part A
Method I: Enumeration
Let the points on the line be
𝐴, 𝐵, 𝐶
Then the line segments which can be formed are:
AB AC BC
Repeat BA CA CB
Method II: Counting
Each point can form a line segment with any of the other two points. By the multiplication principle this gives
us
3 × 2 = 6 𝑝𝑜𝑖𝑛𝑡𝑠
But, this overcounts the number of line segments by a factor of two, since line segment AB is the same as line
segment BA. Hence, the actual number of line segments is:
6
=3
2
Part B
Method I: Enumeration
Let the points on the line be 𝐴, 𝐵, 𝐶, … , 𝐽. Then the line segments which can be formed are:
𝐴𝐵, 𝐴𝐶, … , 𝐴𝐽 𝐵𝐶, 𝐵𝐷 … , 𝐵𝐽 . . . 𝐼𝐽
9 8 . . . 1

9 × 10
1 + 2 + ⋯+ 9 = = 45
2
Method II: Counting
Every point can be connected to every other point. Each of the ten points can be connected to remaining nine
points:
9 × 10 = 90
But, this overcounts the number of line segments by a factor of two, since line segment AB is the same as line
segment BA. Hence, the actual number of line segments is
90
= 45
2

P a g e 20 | 100
Get all the files at: https://bit.ly/azizhandouts
Aziz Manva (azizmanva@gmail.com)

Part C
The minimum number of points needed to form 21 line segments is:
𝑛(𝑛 − 1)
= 21 ⇒ 𝑛(𝑛 − 1) = 42 ⇒ 𝑛 = 7
2
If the points are not distinct, then you can have more than 7 points. Hence, the range of values that n can take is:
{7,8,9, … }

1.45: Number of Intersections formed by 𝒏 lines


Maximum intersections among 𝑛 lines is:
𝑛(𝑛 − 1)
2

The intersections will be maximum when:


➢ The lines are distinct
➢ No three lines are concurrent

Example 1.46
Arnav draws five lines in a plane. Find, at each stage of the drawing, the maximum number of intersections of
these lines.

Finding a Pattern

Handshakes /
Intersections
Additional Total
1st Line: One Person: Cannot shake hands with anyone. 0 0
Cannot intersect with any other Hence, there are no handshakes
line ⇒ No points of intersection
2nd Line: Two People: The second person can shake 1 1
Can intersect with the first line in hands with the first person, and we are done,
a maximum of one place. giving us one handshake
3rd Line: Three People: 2 3
Can intersect with the earlier two The third person can shake hands with the
lines in a maximum of two places. earlier two people, and we are done, giving us
two additional handshakes
4th Line: Four People: 3 6
Can intersect with the earlier The fourth person can shake hands with the
three lines in a maximum of three earlier three people, and we are done, giving us
places. three additional handshakes

Getting a Formula
𝑛(𝑛 − 1)
1 + 2 + 3 + ⋯ + (𝑛 − 1) =
2
Comparing with Handshakes

P a g e 21 | 100
Get all the files at: https://bit.ly/azizhandouts
Aziz Manva (azizmanva@gmail.com)

Lines Intersections People Handshakes


1 0 1 0
2 1 2 1
3 3 3 3
4 6 4 6
5 10 5 10

Example 1.47
(For this question, if two lines are the same line, they will overlap completely. Do not count it as an
intersection).
A. I have 10 lines. What is the number of maximum number of intersections of these lines? What is the
minimum number of intersections of these lines?
B. Math City has eight streets, all of which are straight. No street is parallel to another street. One police
officer is stationed at each intersection. What is the greatest number of police officers needed?
(MathCounts 2008 National Countdown)
C. I have 𝑛 lines, which intersect at exactly 4 distinct places. What is the range of values that 𝑛 can take?
Draw a diagram illustrating one way this is possible?

Part A
10 × 9
𝑀𝑎𝑥 = = 45
2
𝑀𝑖𝑛 = 0
Part B
We need a police officer at each intersection, so we need to count the number of intersections.
At max, each street can intersect the remaining seven streets.
8×7
But, Street A intersecting Street B is the same Street B intersecting Street A. Hence, we are overcounting by a
factor of 2:
8×7
= 28
2
Part C
𝑛(𝑛 − 1) 3 × 2
𝑛=3⇒ = =3
2 2
𝑛(𝑛 − 1) 4 × 3
𝑛=4⇒ = =6
2 2

1.48: Number of Regions that 𝒏 lines divide a plane into


The maximum number of regions that 𝑛 lines can divide a plane into is given by:
𝑛(𝑛 + 1)
+ 1 = 𝑛𝑡ℎ 𝑇𝑟𝑖𝑎𝑛𝑔𝑢𝑙𝑎𝑟 𝑁𝑢𝑚𝑏𝑒𝑟 + 1
2

Example 1.49
Arnav draws five lines in a plane. Find, at each stage of the drawing, the maximum number of areas that the
plane will be divided into.

P a g e 22 | 100
Get all the files at: https://bit.ly/azizhandouts
Aziz Manva (azizmanva@gmail.com)

First Line
If you draw a single line, it divides the plane region into two distinct areas, numbered 1 and
2 in the diagram.

Second Line
If the second line is the same as the first line, then the number of regions remains two.
However, if the line is distinct from the first line, then we get four regions.

Third Line
There are two possibilities here.
➢ The third line intersects at the
point of intersection of the first two
lines. This gives us six distinct areas.
(Diagram on the left side).
➢ The third line does not
intersect at the intersection point of
the other two lines. This gives us
seven distinct areas. (Diagram on the
right side).

Fourth Line
Similar to the third line, we draw the fourth line so that it does not cross any
of the intersections of the other two lines. This gives us four additional
regions that the plane is divided into.

Getting a Formula
We can now form a pattern with what we have seen so far:

Number of Areas
Lines Additional Total
0 1 1
1 1 2
2 2 4
3 3 7
4 4 11
5 5 16
𝑛(𝑛 + 1)
1 + (1 + 2 + 3 + 4 + ⋯ + 𝑛) = +1
2

Example 1.50
A. Find the maximum number of regions that can be formed by seven lines cutting a circle.
B. Find the difference between the minimum and the maximum number of regions formed by five lines in a
plane.
C. 𝑛 lines in a Cartesian plane divide it into 56 regions. Find the minimum and the maximum value of 𝑛.

Part A

P a g e 23 | 100
Get all the files at: https://bit.ly/azizhandouts
Aziz Manva (azizmanva@gmail.com)

𝑛(𝑛 + 1) 7×8
𝑛=7⇒ +1= + 1 = 28 + 1 = 29
2 2

Part B
𝑛(𝑛 + 1) 5×6
𝑀𝑎𝑥: 𝑛 = 5 ⇒ +1= + 1 = 15 + 1 = 16
2 2
The minimum number of regions will be formed when the five lines are all the same line, and hence, we get:
𝑀𝑖𝑛: 2 𝑅𝑒𝑔𝑖𝑜𝑛𝑠
𝐷𝑖𝑓𝑓𝑒𝑟𝑒𝑛𝑐𝑒 = 16 − 2 = 14
Part C
Minimum number of lines:
𝑛(𝑛 + 1)
+ 1 = 56 ⇒ 𝑛(𝑛 + 1) = 110 ⇒ 𝑛 = 10
2
Maximum number of lines
=∞
Example 1.51: Sides
A. (𝐿𝑒𝑓𝑡 𝐷𝑖𝑎𝑔𝑟𝑎𝑚) Three
squares of side 1 m are joined
end to end. Find the perimeter
of the resulting polygon if,
instead of three squares (but
in the same pattern), there are
25 squares
B. (𝑀𝑖𝑑𝑑𝑙𝑒 𝑑𝑖𝑎𝑔𝑟𝑎𝑚) Triangles of side 1 𝑚 are joined end to end, from left to right, as shown in the
diagram. Find the number of sides in the resulting polygon if there are 30 triangles
C. (𝑅𝑖𝑔ℎ𝑡 𝑑𝑖𝑎𝑔𝑟𝑎𝑚) Construct a square on one side of an equilateral triangle. On one non-adjacent side of
the square, construct a regular pentagon, as shown. On a non-adjacent side of the pentagon, construct a
hexagon. Continue to construct regular polygons in the same way, until you construct an octagon. How
many sides does the resulting polygon have? (AMC 8 2009/9)

Part A
25
⏟+ 25
⏟ + ⏟
2 = 52
𝑇𝑜𝑝 𝐵𝑜𝑡𝑡𝑜𝑚 𝑆𝑖𝑑𝑒𝑠

Part B
The top and the bottom triangle each have two sides which are not “inside” the diagram:
2×2=4

The other triangles each have one side:


1 × 28 = 28

Total
= 28 + 4 = 32
Part C
The Triangle and the Octagon have one side less. And the other shapes each have two sides less, giving a total of:

1 + ⏟ 1 + 2×4
⏟ = 10
𝑇𝑟𝑖𝑎𝑛𝑔𝑙𝑒 𝑂𝑐𝑡𝑎𝑔𝑜𝑛 𝑅𝑒𝑚𝑎𝑖𝑛𝑖𝑛𝑔 𝐹𝑜𝑢𝑟 𝑆ℎ𝑎𝑝𝑒𝑠

We can now subtract the 10 missing sides from the total sides, giving us
33 − 10 = 23

P a g e 24 | 100
Get all the files at: https://bit.ly/azizhandouts
Aziz Manva (azizmanva@gmail.com)

Triangle Square Pentagon Hexagon Heptagon Octagon Total Repeated Answer


3 4 5 6 7 8 33 10 23

Example 1.52: Cuts


A. Find the number of sides, number of vertices, and the shape that remains when, from an equilateral
𝑠
triangle with length 𝑠, an equilateral triangle with length 3 is chopped off (𝐼) from a corner, (𝐼𝐼) from
each corner
B. Sally is playing with a cardboard square of side 12 inches. She cuts off four isosceles right-angled
triangles, each with legs 12 cm, from each corner of the square. Find the number of sides, the number of
vertices, and the name of shape that remains. (For this question, you may use: 1 inch = 2.54 cm).

Part A-I
Draw equilateral triangle 𝐴𝐵𝐶. Cut off equilateral CDE from the top. The remaining
shape is DABE. It has 4 𝑐𝑜𝑟𝑛𝑒𝑟𝑠 and 4 𝑠𝑖𝑑𝑒𝑠.
DE is parallel to AB. So, it is a 𝑇𝑟𝑎𝑝𝑒𝑧𝑖𝑢𝑚.
Also, DA is equal to EB. Hence, it is an
𝐼𝑠𝑜𝑠𝑐𝑒𝑙𝑒𝑠 𝑇𝑟𝑎𝑝𝑒𝑧𝑖𝑢𝑚

Part A-II
6 𝑐𝑜𝑟𝑛𝑒𝑟𝑠, 6 𝑠𝑖𝑑𝑒𝑠, 𝑅𝑒𝑔𝑢𝑙𝑎𝑟 𝐻𝑒𝑥𝑎𝑔𝑜𝑛

Part B
See the figure. The original square is in black, while the triangles which
are cut are in red.
From the figure, we see that
𝑁𝑜. 𝑜𝑓 𝑆𝑖𝑑𝑒𝑠 = 𝑁𝑜. 𝑜𝑓 𝑉𝑒𝑟𝑡𝑖𝑐𝑒𝑠 = 8
𝑆ℎ𝑎𝑝𝑒 = 𝑂𝑐𝑡𝑎𝑔𝑜𝑛

§1.3.B Points on a Circle

1.53: Points on a Circle


The maximum number of connecting line segments that can be formed by points on a circle is:
𝐶𝑜𝑛𝑛𝑒𝑐𝑡𝑖𝑜𝑛𝑠
𝑁𝑜. 𝑜𝑓 𝑃𝑜𝑖𝑛𝑡𝑠 × 𝑁𝑜. 𝑜𝑓 𝑃𝑜𝑖𝑛𝑡 𝑛(𝑛 − 1)
=
2 2

Example 1.54
Draw a circle, and count the maximum number of line segments that can be formed if the circle has:
A. 3 distinct points
B. 4 distinct points
C. 5 distinct points

P a g e 25 | 100
Get all the files at: https://bit.ly/azizhandouts
Aziz Manva (azizmanva@gmail.com)

1.55: Rays on a Circle


𝑛(𝑛 − 1)

Rays have a direction. Hence, the number of line segments is exactly half the
number of rays.

Example 1.56
A circle has seven points on it. Find the maximum number of
A. line segments that can be formed by joining any two points
B. rays that can be formed by joining any two points

Line Segments
Each of the seven points can be connected to any of the remaining six points. Total number of line segments
should be
7×6
But, this overcounts the number of line segments by two, since a line segment connecting Point A with Point B is
the same as the line segment connecting Point B with Point A.
Therefore, actual number of line segments is
7×6
= 7 × 3 = 21
2

In general: ⏟
𝐴𝐵 = 𝐵𝐴

𝐿𝑖𝑛𝑒 𝑆𝑒𝑔𝑚𝑒𝑛𝑡 𝐿𝑖𝑛𝑒 𝑆𝑒𝑔𝑚𝑒𝑛𝑡

Rays
Rays are directed line segments. They have a start point, and another point which lies on the ray, and the ray
continues till infinity.
In general: ⏟𝐴𝐵 ≠ 𝐵𝐴

𝑅𝑎𝑦 𝑅𝑎𝑦
Total number of rays

7 × ⏟
6 = 42
𝑃𝑜𝑖𝑛𝑡𝑠 𝐸𝑎𝑐ℎ 𝑐𝑜𝑛𝑛𝑒𝑐𝑡𝑒𝑑 𝑡𝑜
𝑟𝑒𝑚𝑎𝑖𝑛𝑖𝑛𝑔 6 𝑝𝑜𝑖𝑛𝑡𝑠

Example 1.57
A mystic rose is a pattern made by connecting dots in a circle via line segments. How many line segments will
there be in a Mystic Rose with 9 distinct dots?

P a g e 26 | 100
Get all the files at: https://bit.ly/azizhandouts
Aziz Manva (azizmanva@gmail.com)

𝑛(𝑛 − 1) 9 × 8
= = 36
2 2

1.58 Lines Connecting 𝒏 vertices of a Polygon


𝒏(𝒏 − 𝟏)
𝐍𝐮𝐦𝐛𝐞𝐫 𝐨𝐟 𝐋𝐢𝐧𝐞𝐬 𝐜𝐨𝐧𝐧𝐞𝐜𝐭𝐢𝐧𝐠 𝒏 𝐯𝐞𝐫𝐭𝐢𝐜𝐞𝐬 𝐨𝐟 𝐚 𝐩𝐨𝐥𝐲𝐠𝐨𝐧 = 𝟐

Each vertex (of which there are n) can have a line drawn connecting to the
remaining vertices (of which there are n – 1).
Proposed Number of Lines = n(n - 1).
However, connecting the 1st point with the 2nd is the same as connecting the 2nd
point with the first. Hence, we are overcounting the actual number of lines by a factor of two.

Example 1.59
What is the number of ways to choose two line segments from the 15 segments connecting the six vertices of
regular hexagon ABCDEF such that the endpoints include each of A, B, C, D, E and F?

From points A, B, C, D, E and F, we need to choose three pairs that cover all the points.

Number of ways to choose the first pair:


6 Choose 2 = 6!/4!2! = 15

Number of ways to choose the second pair:


4 Choose 2 = 4!/2!2! = 6

Number of ways to choose the third pair:


2 Choose 2 = 1

15*6=90

1.60 Diagonals of a Polygon


A polygon with 𝑛 sides has
𝑛(𝑛 − 3)
𝑑𝑖𝑎𝑔𝑜𝑛𝑎𝑙𝑠
2

Using the formula above


𝑛(𝑛−1)
The number of lines segment joining 𝑛 vertices is 2 .
The number of sides is 𝑛.
Using complementary counting, the number of diagonals is:
𝑛(𝑛 − 1) 𝑛2 − 𝑛 − 2𝑛 𝑛2 − 3𝑛 𝑛(𝑛 − 3)
−𝑛 = = =
2 2 2 2
Logic
When counting diagonals, we do not want to count sides. From any vertex of a
polygon, we cannot get a diagonal by drawing a line segment to either vertex
adjacent to it.
Hence, the number of diagonals is:

P a g e 27 | 100
Get all the files at: https://bit.ly/azizhandouts
Aziz Manva (azizmanva@gmail.com)


𝑛 × (𝑛 − 3)
⏟ = 𝑛(𝑛 − 3)
𝑵𝒐.𝒐𝒇 𝑽𝒆𝒓𝒕𝒊𝒄𝒆𝒔 𝑵𝒐.𝒐𝒇 𝑷𝒐𝒊𝒏𝒕𝒔
𝒕𝒐 𝒅𝒓𝒂𝒘 𝑫𝒊𝒂𝒈𝒐𝒏𝒂𝒍𝒔 𝒕𝒐
But, as usual, we need to divide by 2 to account for the overcounting:
𝑛(𝑛 − 3)
2

Example 1.61
A. A diagonal of a polygon is a line segment that connect two vertices of the polygon, but is not a side of the
polygon. What is the number of diagonals of a regular polygon with 12 sides?
B. 10 people are standing in a circle. They pass a ball to each other in every way possible, except that no
one tosses the ball to the person to the right and the left of them in the circle. If the order in which the
tosses is not important, count the number of distinct tosses.
C. How many diagonals does a Septagon have?
D. The number of diagonals that can be drawn in a polygon of 100 sides is (AHMSE 1950/45)
E. A polygon has the same number of diagonals as it has sides. Find the number of sides of the polygon.
F. A polygon has 𝑥 vertices, and another polygon has 𝑦 = 𝑥 − 2 vertices. What is the difference in the
number of diagonals that the two polygons have? What is the minimum value of 𝑥 + 𝑦?

Part A
Imagine that we put all the points on a circle. The total number of ways of obtaining line segments from these
12 points is
12 × 11
= 6 × 11 = 66
2
But, this overcounts the number of diagonals by the number of sides. (Since no side is a diagonal).
Hence, the number of diagonals
66
⏟ − 12
⏟ = 54
𝑵𝒐.𝒐𝒇 𝑳𝒊𝒏𝒆 𝑺𝒆𝒈𝒎𝒆𝒏𝒕𝒔 𝑵𝒐.𝒐𝒇 𝑺𝒊𝒅𝒆𝒔
Part B
10 × 9
− 10 = 45 − 10 = 35
2
Part C
A heptagon has seven vertices. We count diagonals as follows:
𝑛(𝑛 − 3) 7 × 4
n = 7 ⇒ Diagonals = = = 14
2 2
Part D
4850
Part E
𝑛(𝑛 − 3)
=𝑛 ⇒𝑛−3=2⇒𝑛 =5
2
Part F
𝑥(𝑥 − 3) (𝑥 − 2)(𝑥 − 5) 𝑥 2 − 3𝑥 − (𝑥 2 − 7𝑥 + 10) 4𝑥 − 10
− = = = 2𝑥 − 5
2 2 2 2
𝑀𝑖𝑛(𝑦 = 𝑥 − 2) = 3 ⇒ 𝑀𝑖𝑛(𝑥) = 5 ⇒ 𝑦 + 𝑥 = 3 + 5 = 8

𝑀𝑖𝑛.𝑉𝑒𝑟𝑡𝑖𝑐𝑒𝑠 𝑜𝑓
𝑃𝑜𝑙𝑦𝑔𝑜𝑛=3

§1.3.C Tiling: Squares

1.62: Squares

P a g e 28 | 100
Get all the files at: https://bit.ly/azizhandouts
Aziz Manva (azizmanva@gmail.com)

Example 1.63
Consider squares of integer side length made by joining 1 × 1 tiles. For squares of length 1 to 5:
A. count the number of tiles that a diagonal passes through.
B. count the number of number of intersection points (where 4 tiles join together) that a diagonal passes
through
C. Repeat Parts A and B for both diagonals

Single Diagonal Both Diagonals


Side Tiles Intersection Tiles Intersection
Length Points Points
1 1 0 1 0
2 2 1 4 1
3 3 2 5 4
4 4 3 8 5
5 5 4 9 8

Odd 𝑛 𝑛 𝑛−1 2𝑛 − 1 2𝑛 − 2
Even 𝑛 𝑛 𝑛−1 2𝑛 2𝑛 − 3

Example 1.64
A dance floor is 100 feet long by 100 feet wide. It is tiled using tiles that are each 2 feet in length and width. A
blue line is drawn from the top left corner of the floor to the bottom right corner. A red line is drawn from the
top right corner of the floor to the bottom left corner.
A. How many tiles will both lines together pass through
B. How many corner points (intersection of 4 tiles) will both lines together

The dance floor has


50
⏟ × 50
⏟ = 2500 𝑇𝑖𝑙𝑒𝑠
𝐿𝑒𝑛𝑔𝑡ℎ 𝑊𝑖𝑑𝑡ℎ
Part A
𝐵𝑜𝑡ℎ 𝐿𝑖𝑛𝑒𝑠 = 50
⏟ + 50
⏟ = 100
𝐵𝑙𝑢𝑒 𝑅𝑒𝑑
Part B
50
⏟ − 1 + 50
⏟−1− ⏟
1 = 49
⏟ + 49
⏟− ⏟
1 = 98 − 1 = 97
𝐵𝑙𝑢𝑒 𝑅𝑒𝑑 𝑂𝑣𝑒𝑟𝑙𝑎𝑝 𝐵𝑙𝑢𝑒 𝑅𝑒𝑑 𝑂𝑣𝑒𝑟𝑙𝑎𝑝

Answer the above question if the dance floor is 102 feet long by 102 wide instead (and everything else remains
the same).

P a g e 29 | 100
Get all the files at: https://bit.ly/azizhandouts
Aziz Manva (azizmanva@gmail.com)

The number of tiles


102
= = 51
2
Part A
51
⏟ + 51
⏟ − 1 = 51 + 50 = 101
𝐵𝑙𝑢𝑒 𝑅𝑒𝑑
Part B
51
⏟ − 1 + 51
⏟ − 1 = 50 + 50 = 100
𝐵𝑙𝑢𝑒 𝑅𝑒𝑑
Example 1.65
A square is divided into 𝑛2 congruent squares by drawing lines parallel/perpendicular to the sides. Both
diagonals of the squares pass through 𝑥 of the smaller squares. Find the possible value of 𝑛 if:
A. 𝑥 = 28
B. 𝑥 = 30
We do not know whether 𝑛 is even or odd.
𝑛 𝑖𝑠 𝑒𝑣𝑒𝑛 ⇒ 𝑁𝑜. 𝑜𝑓 𝑡𝑖𝑙𝑒𝑠 = 2𝑛
𝑛 𝑖𝑠 𝑜𝑑𝑑 ⇒ 𝑁𝑜. 𝑜𝑓 𝑡𝑖𝑙𝑒𝑠 = 2𝑛 − 1
Part A
2𝑛 = 28 ⇒ 𝑛 = 14 ⇒ 𝐸𝑣𝑒𝑛 ⇒ 𝑉𝑎𝑙𝑖𝑑
2𝑛 − 1 = 28 ⇒ 2𝑛 = 29 ⇒ 𝑛 = 14.5 ⇒ 𝑁𝑜𝑡 𝑎𝑛 𝐼𝑛𝑡𝑒𝑔𝑒𝑟 ⇒ 𝑁𝑜𝑡 𝑉𝑎𝑙𝑖𝑑
Part B
2𝑛 = 30 ⇒ 𝑛 = 15 ⇒ 𝑂𝑑𝑑 ⇒ 𝑁𝑜𝑡 𝑉𝑎𝑙𝑖𝑑
2𝑛 − 1 = 30 ⇒ 2𝑛 = 31 ⇒ 𝑛 = 15.5 ⇒ 𝑁𝑜𝑡 𝑎𝑛 𝐼𝑛𝑡𝑒𝑔𝑒𝑟 ⇒ 𝑁𝑜𝑡 𝑉𝑎𝑙𝑖𝑑
There is no square that meets these conditions.

Example 1.66
A square-shaped floor is covered with congruent square tiles. If the total number of tiles that lie on the two
diagonals is 37, how many tiles cover the floor? (AMC 8 2017/11)

2𝑛 − 1 = 37
2𝑛 = 38
𝑛 = 19

Example 1.67
The floor of a square room is covered with congruent square tiles. The diagonals of the room are drawn across
the floor, and two diagonals intersect a total of 9 tiles. How many tiles are on the floor? (MathCounts 2000
Chapter #8)

Let 𝑛 be the side length of the room.


9
𝐸𝑣𝑒𝑛: 2𝑛 = 9 ⇒ 𝑛 = ⇒ 𝑁𝑜𝑡 𝑉𝑎𝑙𝑖𝑑
2
𝑂𝑑𝑑: 2𝑛 − 1 = 9 ⇒ 𝑛 = 5 ⇒ 𝑛2 = 25

Example 1.68
A square is divided into 𝑛2 congruent squares by drawing lines parallel/perpendicular to the sides. Both
diagonals of the squares pass through 𝑥 intersections of 4 tiles. Find the possible value of 𝑛 if:
A. 𝑥 = 28
B. 𝑥 = 30

P a g e 30 | 100
Get all the files at: https://bit.ly/azizhandouts
Aziz Manva (azizmanva@gmail.com)

We do not know whether 𝑛 is even or odd.


𝑛 𝑖𝑠 𝑒𝑣𝑒𝑛 ⇒ 𝑁𝑜. 𝑜𝑓 𝑡𝑖𝑙𝑒𝑠 = (𝑛 − 1) + (𝑛 − 1) − 1 = 2𝑛 − 3
𝑛 𝑖𝑠 𝑜𝑑𝑑 ⇒ 𝑁𝑜. 𝑜𝑓 𝑡𝑖𝑙𝑒𝑠 = (𝑛 − 1) + (𝑛 − 1) = 2𝑛 − 2
Part A
2𝑛 − 2 = 28 ⇒ 2𝑛 = 30 ⇒ 𝑛 = 15 ⇒ 𝑂𝑑𝑑 ⇒ 𝑉𝑎𝑙𝑖𝑑

Example 1.69
(CEMC Pascal 2022/21)

§1.3.D Tiling: Rectangles

1.70: Number of Tiles

P a g e 31 | 100
Get all the files at: https://bit.ly/azizhandouts
Aziz Manva (azizmanva@gmail.com)

The number of tiles crossed by the diagonal of a rectangular floor with integral length 𝑙 and integral width 𝑤 is
𝑙 + 𝑤 − HCF(𝑙, 𝑤)

The rectangle to the right has:


𝑙𝑒𝑛𝑔𝑡ℎ = 4, 𝑤𝑖𝑑𝑡ℎ = 3 ⇒ 𝐻𝐶𝐹(3,4) = 1
The diagonal crosses 3 vertical lines 2 horizontal lines:

3 + ⏟
2 + ⏟ 1 =6
𝑉𝑒𝑟𝑡𝑖𝑐𝑎𝑙 𝐻𝑜𝑟𝑖𝑧𝑜𝑛𝑡𝑎𝑙 𝑆𝑡𝑎𝑟𝑡
𝐿𝑖𝑛𝑒𝑠 𝐿𝑖𝑛𝑒𝑠 𝑇𝑖𝑙𝑒


7 + ⏟
5 + ⏟
1 − 2−1
⏟ = 12
𝑉𝑒𝑟𝑡𝑖𝑐𝑎𝑙 𝐻𝑜𝑟𝑖𝑧𝑜𝑛𝑡𝑎𝑙 𝑆𝑡𝑎𝑟𝑡 𝐼𝑛𝑡𝑒𝑟𝑠𝑒𝑐𝑡𝑖𝑜𝑛
𝐿𝑖𝑛𝑒𝑠 𝐿𝑖𝑛𝑒𝑠 𝑇𝑖𝑙𝑒 𝑃𝑜𝑖𝑛𝑡𝑠

7 + ⏟
5 + ⏟
1 − ⏟
2 + 1 = 12
𝑉𝑒𝑟𝑡𝑖𝑐𝑎𝑙 𝐻𝑜𝑟𝑖𝑧𝑜𝑛𝑡𝑎𝑙 𝑆𝑡𝑎𝑟𝑡 𝐻𝐶𝐹(6,8)
𝐿𝑖𝑛𝑒𝑠 𝐿𝑖𝑛𝑒𝑠 𝑇𝑖𝑙𝑒

7 + ⏟
5 +2− ⏟
2 = 12
𝑉𝑒𝑟𝑡𝑖𝑐𝑎𝑙 𝐻𝑜𝑟𝑖𝑧𝑜𝑛𝑡𝑎𝑙 𝐻𝐶𝐹(6,8)
𝐿𝑖𝑛𝑒𝑠 𝐿𝑖𝑛𝑒𝑠

8 + ⏟
6 − ⏟
2 = 12
𝐿𝑒𝑛𝑔𝑡ℎ 𝑊𝑖𝑑𝑡ℎ 𝐻𝐶𝐹(6,8)

(𝑙 − 1) + (𝑤 − 1) + 1 − [𝐻𝐶𝐹(𝑙, 𝑤) − 1]

Example 1.71
A. A rectangular floor that is 10 feet wide and 17 feet long is tiled with 170 one-foot square tiles. A bug
walks from one corner to the opposite corner in a straight line. Including the first and the last tile, how
many tiles does the bug visit? (AMC 10A 2019/10)
B. A 24-foot by 72-foot rectangular dance floor is completely tiled with 1-foot by 1-foot square tiles. Two
opposite corners of the dance floor are connected by a diagonal. This diagonal passes through the
interior of exactly how many tiles? (2002 MathCounts Chapter Sprint #27)

Part A
𝑙 + 𝑤 − HCF(𝑙, 𝑤) = 10 + 17 − 1 = 27 − 1 = 26
Part B
𝑙 + 𝑤 − gcd(𝑙, 𝑤) = 72 + 24 − 24 = 72

Example 1.72
The diagonal of a rectangular floor tiled using 1 × 1 tiles passes through exactly 12 tiles. Find the sum of the
possible values of the perimeter.

𝑙 + 𝑤 − HCF(𝑙, 𝑤)
The minimum value of 𝐻𝐶𝐹(𝑙, 𝑤) is 1.

Perimeter Length Width 𝑙+𝑤 𝐻𝐶𝐹(𝑙, 𝑤) No. of Tiles


26 12 1 13 1 12
28 8 6 14 2 12
30 9 6 15 3 12
32 12 4 16 4 12
17 5 12

P a g e 32 | 100
Get all the files at: https://bit.ly/azizhandouts
Aziz Manva (azizmanva@gmail.com)

36 12 6 18 6 12
19 7 12
20 8 12

152 Total

1.73: Number of Tiles crossed by both diagonals

Both are even:


Overlap of 0

Both length and width are odd and equal:


Overlap of 1

One is odd, and the other is even:


Overlap of 2

Both length and width are odd and distinct:


Overlap of 3

Example 1.74
Julian tiled a 15 feet by 21 feet rectangular ballroom with one-foot square tiles. When he finished, he drew both
diagonals on the floor connecting the opposite corners of the room. What is the total number of tiles that the
diagonals pass through? (2001 MathCounts Handbook Warm up 12 #6)

𝑙 + 𝑤 − gcd(𝑙, 𝑤) = 21 + 15 − 3 = 33
33 + 33 − 3 = 63

1.75: Number of Intersection Points


The number of 4-corner intersection points crossed by the diagonal of a rectangular floor with integral length 𝑙
and integral width 𝑤 is
HCF(𝑙, 𝑤) + 1(𝐸𝑛𝑑𝑝𝑜𝑖𝑛𝑡𝑠 𝑖𝑛𝑐𝑙𝑢𝑑𝑒𝑑)
HCF(𝑙, 𝑤) − 1(𝐸𝑛𝑑𝑝𝑜𝑖𝑛𝑡𝑠 𝑒𝑥𝑐𝑙𝑢𝑑𝑒𝑑)

Example 1.76
A rectangular floor is tiled in a chalkboard fashion with square tiles of edge length 1. The dimensions of the
floor are 321 by 123 units. An insect walks along the diagonal of the rectangle.

P a g e 33 | 100
Get all the files at: https://bit.ly/azizhandouts
Aziz Manva (azizmanva@gmail.com)

A. How many 4-corner intersection points does the insect cross on its walk between diagonally opposite
corners? (Michigan Mathematics Prize Competition 1998 #18)
B. What are the coordinates of those intersection points?
Part A
gcd(321,123) − 1 = 3 − 1 = 2
Part B
(107,41), (214,82)
§1.3.E Tiling: Cuboids

1.77: Cuboids
For a cuboid with integer side lengths and length 𝑙, width 𝑤, and height ℎ, the number of 1 × 1 × 1 cuboids a
diagonal passes through the interior is:
𝑤 + 𝑙 + ℎ − 𝐻𝐶𝐹(𝑤, 𝑙) − 𝐻𝐶𝐹(𝑙, ℎ) − 𝐻𝐶𝐹(ℎ, 𝑤) + 𝐻𝐶𝐹(𝑤, 𝑙, ℎ)

Example 1.78
A 150 × 324 × 375 rectangular solid is made by gluing together 1 × 1 × 1 cubes. An internal diagonal of this
solid passes through the interiors of how many of the 1 × 1 × 1 cubes. (AIME 1996/14)

𝑤 = 150 = 2 × 3 × 52
𝑙 = 324 = 22 × 34
ℎ = 375 = 3 × 52
= 150
⏟ + 324
⏟ + 375
⏟− ⏟ 6 − ⏟ 3 − 75 ⏟ + ⏟
3 = 768
𝑤 𝑙 ℎ 𝐻𝐶𝐹(𝑤,𝑙) 𝐻𝐶𝐹(𝑙,ℎ) 𝐻𝐶𝐹(ℎ,𝑤) 𝐻𝐶𝐹(𝑤,𝑙,ℎ)

§1.4 3D Geometry
§1.4.A Manual Counting

Example 1.79
Jamie counted the number of edges of a cube, Jimmy counted the numbers of corners, and Judy counted the
number of faces. They then added the three numbers. What was the resulting sum? (AMC 8 2003/1)

A cube has a face each at the base, at the top, in front, on the left, on the right and
on the back, giving

2 + ⏟
2 + ⏟ 2 =6
𝑇𝑜𝑝 𝑎𝑛𝑑 𝐹𝑟𝑜𝑛𝑡 𝑎𝑛𝑑 𝐿𝑒𝑓𝑡 𝑎𝑛𝑑
𝐵𝑜𝑡𝑡𝑜𝑚 𝐵𝑎𝑐𝑘 𝑅𝑖𝑔ℎ𝑡
A cube has four vertices at the base. And four vertices at the top, giving us a total
of
4 + 4 = 8 𝑉𝑒𝑟𝑡𝑖𝑐𝑒𝑠

A cube has four edges at the base, four edges at the top, and four edges
connecting the top with the base giving
4 + 4 + 4 = 12 𝑒𝑑𝑔𝑒𝑠

𝑇𝑜𝑡𝑎𝑙 = ⏟
6 + ⏟
8 + 12
⏟ = 26
𝑭𝒂𝒄𝒆𝒔 𝑽𝒆𝒓𝒕𝒊𝒄𝒆𝒔 𝑬𝒅𝒈𝒆𝒔

1.80: Euler’s Characteristic


Euler’s Characteristic connects the number of faces, the number of vertices, and the number of edges of a

P a g e 34 | 100
Get all the files at: https://bit.ly/azizhandouts
Aziz Manva (azizmanva@gmail.com)

𝑐𝑜𝑛𝑣𝑒𝑥 𝑝𝑜𝑙𝑦𝑔𝑜𝑛, or ⏟
⏟ 𝑐𝑜𝑛𝑣𝑒𝑥 𝑝𝑜𝑙𝑦ℎ𝑒𝑑𝑟𝑜𝑛.
𝑻𝒘𝒐 𝒅𝒊𝒎𝒆𝒏𝒔𝒊𝒐𝒏𝒔 𝑻𝒉𝒓𝒆𝒆 𝑫𝒊𝒎𝒆𝒏𝒔𝒊𝒐𝒏𝒔
𝐹
⏟ + 𝑉
⏟ = 𝐸
⏟ +2
𝑭𝒂𝒄𝒆𝒔 𝑽𝒆𝒓𝒕𝒊𝒄𝒆𝒔 𝑬𝒅𝒈𝒆𝒔

Convex Polyhedron: A polyhedron is convex if no angle in the polyhedron is more than 180 degrees.
Some common convex polyhedra are given below:

Example 1.81
Verify that the following shapes satisfy Euler’s Formula. A diagram of (most) of the shapes is above. Make a
summary table once you have collated all the answers.
A. A cube is the three-dimensional version of a square. It has all sides the same.
B. Pyramids are of different types. A pyramid is named according to the base that it has. A triangular
pyramid (tetrahedron) is the simplest pyramidal shape. It has the same shape for its base as for the rest
of its faces.
C. A square pyramid has a square for a base. And the rest of the faces are triangles.
D. A rectangular pyramid has a rectangle for a base. And the rest of the faces are triangles.
E. A pentagonal pyramid has a pentagon for a base. And the rest of the faces are triangles.
F. A triangular prism has a triangle for a base, and the remaining sides are at right angles to the base.

Part A 𝑅𝐻𝑆 = 𝐸 + 2 = 8 + 2 = 10

6 + ⏟
8 = 12
⏟ + 2 = 14 ⇒ 𝑉𝑒𝑟𝑖𝑓𝑖𝑒𝑑 Part D
𝑭𝒂𝒄𝒆𝒔 𝑽𝒆𝒓𝒕𝒊𝒄𝒆𝒔 𝑬𝒅𝒈𝒆𝒔
Part B 𝐹+𝑉 =𝐸+2
The base is a triangle, and each side of the base 𝐿𝐻𝑆 = 𝐹 + 𝑉 = 5 + 5 = 10
triangle is attached to a slant face leading towards 𝑅𝐻𝑆 = 𝐸 + 2 = 8 + 2 = 10
the apex of the pyramid. Part E
𝑇𝑜𝑡𝑎𝑙 𝐹𝑎𝑐𝑒𝑠 = ⏟ 1 + ⏟
3 =4
𝑩𝒂𝒔𝒆 𝑺𝒍𝒂𝒏𝒕 𝑭𝒂𝒄𝒆𝒔
𝐹+𝑉 =𝐸+2
The base has three vertices from the triangle, and
𝐿𝐻𝑆 = 𝐹 + 𝑉 = 6 + 6 = 12
one vertex for apex:
𝑅𝐻𝑆 = 𝐸 + 2 = 10 + 2 = 12
𝑇𝑜𝑡𝑎𝑙 𝑉𝑒𝑟𝑡𝑖𝑐𝑒𝑠 = ⏟ 3 + ⏟ 1 =4
𝑩𝒂𝒔𝒆 𝑨𝒑𝒆𝒙
Part F: Triangular Prism
Three edges from the base and three edges from the 𝐹+𝑉 =𝐸+2
slant faces: 𝐿𝐻𝑆 = 𝐹 + 𝑉 = 5 + 6 = 11

3 + ⏟ 3 =6 𝑅𝐻𝑆 = 𝐸 + 2 = 9 + 2 = 11
𝑩𝒂𝒔𝒆 𝑬𝒅𝒈𝒆𝒔 Part G: Pentagonal Prism
𝐹+𝑉 =𝐸+2 𝐹𝑎𝑐𝑒𝑠 = ⏟ 1 + ⏟ 1 + ⏟5 =7
𝐿𝐻𝑆 = 𝐹 + 𝑉 = 4 + 4 = 8 𝐵𝑎𝑠𝑒 𝑇𝑜𝑝 𝑆𝑖𝑑𝑒𝑠

𝑅𝐻𝑆 = 𝐸 + 2 = 6 + 2 = 8 𝑉𝑒𝑟𝑡𝑖𝑐𝑒𝑠 = ⏟
5 + ⏟
5 = 10
𝐵𝑎𝑠𝑒 𝑇𝑜𝑝
Part C
𝐸𝑑𝑔𝑒𝑠 = 5 + 5 + 5 = 15
𝐹+𝑉 =𝐸+2
𝐿𝐻𝑆 = 𝐹 + 𝑉 = 10 + 7 = 17
𝐿𝐻𝑆 = 𝐹 + 𝑉 = 5 + 5 = 10
𝑅𝐻𝑆 = 𝐸 + 2 = 15 + 2 = 17

P a g e 35 | 100
Get all the files at: https://bit.ly/azizhandouts
Aziz Manva (azizmanva@gmail.com)

Summary Table
Faces Vertices Edges 𝐹+𝑉
=𝐸+2
Prisms Cube/Cuboid/ 6 8 12 14
Rectangular Prism
Pentagonal Prism 7 10 15 17

Octahedron 8 6 12 14
Pyramids Triangular Pyramid 4 4 6 8
(Tetrahedron)
Square Pyramid 5 5 8 10

Example 1.82
Show that the following shapes do not satisfy Euler’s Characteristic given above.
A. Cylinder
B. Sphere

Part A
𝐿𝐻𝑆 = ⏟
3 + ⏟
0 =3≠4= ⏟
2 + 2 = 𝑅𝐻𝑆
𝑭𝒂𝒄𝒆𝒔 𝑽𝒆𝒓𝒕𝒊𝒄𝒆𝒔 𝑬𝒅𝒈𝒆𝒔
Hence, a cylinder does not satisfy Euler’s Characteristic.

Part B
𝐿𝐻𝑆 = ⏟
1 + ⏟
0 =1≠2= ⏟
0 + 2 = 𝑅𝐻𝑆
𝑭𝒂𝒄𝒆𝒔 𝑽𝒆𝒓𝒕𝒊𝒄𝒆𝒔 𝑬𝒅𝒈𝒆𝒔

Example 1.83
A. What is the number of edges that an octahedron (a convex three dimensional
solid with 8 faces, and 6 vertices) must have?

Part A
Substitute 𝐹 = 8, 𝑉 = 6 in Euler’s Formula:
𝐹 + 𝑉 = 𝐸 + 2 ⇒ 8 + 6 = 𝐸 + 2 ⇒ 𝐸 = 12

Example 1.84
Each corner of a rectangular prism is cut off. Two (of the eight) cuts are shown.
A. How many edges does the new figure have? Assume that the planes cutting the
prism do not intersect anywhere in or on the prism. (AMC 8 1990/18)
B. How many vertices does the new figure have?

Part A
The number of edges of a cube is 12. Each cut adds three edges, but does not take away any
edge. Hence, the number of additional edges

3 × ⏟ 8 = 24
𝐴𝑑𝑑𝑖𝑡𝑖𝑜𝑛𝑎𝑙 𝑁𝑜.𝑜𝑓
𝐸𝑑𝑔𝑒𝑠 𝑉𝑒𝑟𝑡𝑖𝑐𝑒𝑠
Hence, the total number of edges of the figure is

P a g e 36 | 100
Get all the files at: https://bit.ly/azizhandouts
Aziz Manva (azizmanva@gmail.com)

12 + 24 = 36
Part B
8 + 16 = 24

Example 1.85
A cube of edge 3 cm is cut into 𝑁 smaller cubes, not all the same size. If the edge of each of the smaller cubes is a
whole number of centimeters, then 𝑁 = (AMC 8 1991/24)

Volume of the original cube


= 3 × 3 × 3 = 27
Cubes with edge smaller than 3 cm = {2 cm, 1 cm}

Since all the cubes are not of the same size, there has to be at least one cube of edge length 2 cm, which has
volume
8 𝑐𝑚
The only possible cubes which can be accommodated now are cubes of edge length 1.

The number of such cubes will be


27 − 8 = 19 𝐶𝑢𝑏𝑒𝑠
𝑇𝑜𝑡𝑎𝑙 𝐶𝑢𝑏𝑒𝑠 = 1 + 19 = 20

§1.4.B Counting Formulas

1.86: Vertices
If a fix number of faces meet at each vertex of a polyhedron, then:
𝑁𝑜. 𝑜𝑓 𝐹𝑎𝑐𝑒𝑠 × 𝑁𝑜. 𝑜𝑓𝑆𝑖𝑑𝑒𝑠 𝑝𝑒𝑟 𝐹𝑎𝑐𝑒
𝑁𝑜. 𝑜𝑓 𝑉𝑒𝑟𝑡𝑖𝑐𝑒𝑠 =
𝑁𝑜. 𝑜𝑓 𝐹𝑎𝑐𝑒𝑠 𝑎𝑡 𝑒𝑎𝑐ℎ 𝑣𝑒𝑟𝑡𝑒𝑥

We divide by the number of faces meeting at each vertex because that many faces share a vertex.

Example 1.87
Use the formula above to calculate the number of vertices for:
A. A cube

Part A: Cube
𝑁𝑜. 𝑜𝑓 𝐹𝑎𝑐𝑒𝑠 × 𝑁𝑜. 𝑜𝑓𝑆𝑖𝑑𝑒𝑠 𝑝𝑒𝑟 𝐹𝑎𝑐𝑒 6(4) 24
𝑁𝑜. 𝑜𝑓 𝑉𝑒𝑟𝑡𝑖𝑐𝑒𝑠 = = = =8
𝑁𝑜. 𝑜𝑓 𝐹𝑎𝑐𝑒𝑠 𝑎𝑡 𝑒𝑎𝑐ℎ 𝑣𝑒𝑟𝑡𝑒𝑥 3 3

Octahedron
Number of Edges
𝑁𝑜. 𝑜𝑓 𝐹𝑎𝑐𝑒𝑠 × 𝑁𝑜. 𝑜𝑓 𝑆𝑖𝑑𝑒𝑠 𝑝𝑒𝑟 𝐹𝑎𝑐𝑒 8 × 3 24
= = = = 12
2 2 2
Number of Vertices
𝑁𝑜. 𝑜𝑓 𝐹𝑎𝑐𝑒𝑠 × 𝑁𝑜. 𝑜𝑓𝑆𝑖𝑑𝑒𝑠 𝑝𝑒𝑟 𝐹𝑎𝑐𝑒 8 × 3 24
= = = =6
𝑁𝑜. 𝑜𝑓 𝐹𝑎𝑐𝑒𝑠 𝑚𝑒𝑒𝑡𝑖𝑛𝑔 𝑎𝑡 𝑒𝑎𝑐ℎ 𝑣𝑒𝑟𝑡𝑒𝑥 4 4

P a g e 37 | 100
Get all the files at: https://bit.ly/azizhandouts
Aziz Manva (azizmanva@gmail.com)

1.88: Edges-I
If a polyhedron has the same two-dimensional shape for each of its faces, then:
𝑁𝑜. 𝑜𝑓 𝐹𝑎𝑐𝑒𝑠 × 𝑁𝑜. 𝑜𝑓 𝑆𝑖𝑑𝑒𝑠 𝑝𝑒𝑟 𝐹𝑎𝑐𝑒
𝑁𝑜. 𝑜𝑓 𝐸𝑑𝑔𝑒𝑠 =
2

For edges, we divide by two because each edge is shared by two faces.

Example 1.89
Use the formula above to calculate the number of vertices and number of edges for:
A. Octahedron

Octahedron
Number of Edges
𝑁𝑜. 𝑜𝑓 𝐹𝑎𝑐𝑒𝑠 × 𝑁𝑜. 𝑜𝑓 𝑆𝑖𝑑𝑒𝑠 𝑝𝑒𝑟 𝐹𝑎𝑐𝑒 8 × 3 24
= = = = 12
2 2 2

Number of Vertices
𝑁𝑜. 𝑜𝑓 𝐹𝑎𝑐𝑒𝑠 × 𝑁𝑜. 𝑜𝑓𝑆𝑖𝑑𝑒𝑠 𝑝𝑒𝑟 𝐹𝑎𝑐𝑒 8 × 3 24
= = = =6
𝑁𝑜. 𝑜𝑓 𝐹𝑎𝑐𝑒𝑠 𝑚𝑒𝑒𝑡𝑖𝑛𝑔 𝑎𝑡 𝑒𝑎𝑐ℎ 𝑣𝑒𝑟𝑡𝑒𝑥 4 4

Example 1.90
A regular dodecahedron is made up of twelve regular pentagons. Find the number of edges.

𝑁𝑜. 𝑜𝑓 𝐹𝑎𝑐𝑒𝑠 × 𝑁𝑜. 𝑜𝑓 𝑆𝑖𝑑𝑒𝑠 𝑝𝑒𝑟 𝐹𝑎𝑐𝑒 12 × 5 60


𝑁𝑜. 𝑜𝑓 𝐸𝑑𝑔𝑒𝑠 = = = = 30
2 2 2

1.91: Edges-II
If the faces of a polyhedron are not all the same type, then we account for each type of
face separately.
𝑓1 × 𝑠1 + 𝑓2 × 𝑠2 + ⋯
𝐸𝑑𝑔𝑒𝑠 =
2
Where
𝑓1 = 𝑁𝑜. 𝑜𝑓 𝐹𝑎𝑐𝑒𝑠 𝑜𝑓 𝑇𝑦𝑝𝑒 1
𝑠1 = 𝑁𝑜. 𝑜𝑓 𝑆𝑖𝑑𝑒𝑠 𝑖𝑛 𝑒𝑎𝑐ℎ 𝑜𝑓 𝑓1
𝑓2 = 𝑁𝑜. 𝑜𝑓 𝐹𝑎𝑐𝑒𝑠 𝑜𝑓 𝑇𝑦𝑝𝑒 2
𝑠2 = 𝑁𝑜. 𝑜𝑓 𝑆𝑖𝑑𝑒𝑠 𝑖𝑛 𝑒𝑎𝑐ℎ 𝑜𝑓 𝑓2

Example 1.92
Using the formulas above, count the number of edges, and the number of vertices of
A. A triangular pyramid has four faces, with each face being a triangle, and three faces meeting at each
vertex.
B. A square pyramid has five faces, and the faces are of two types. The first type of face is a single square,
each with four sides. The second type of face is a triangle (of which there are four), each with three sides

P a g e 38 | 100
Get all the files at: https://bit.ly/azizhandouts
Aziz Manva (azizmanva@gmail.com)

Triangular Pyramid

4 × ⏟
3
𝑁𝑜.𝑜𝑓 𝑆𝑖𝑑𝑒𝑠
𝐹𝑎𝑐𝑒𝑠 𝑝𝑒𝑟 𝐹𝑎𝑐𝑒 12
𝐸𝑑𝑔𝑒𝑠 = = =6
2 2

4 × ⏟
3
𝑁𝑜.𝑜𝑓 𝑆𝑖𝑑𝑒𝑠
𝑝𝑒𝑟 𝐹𝑎𝑐𝑒 12
𝑉𝑒𝑟𝑡𝑖𝑐𝑒𝑠 = 𝐹𝑎𝑐𝑒𝑠 = =4
3 3
Square Pyramid


1 × ⏟
4 + ⏟
4 × ⏟
3
𝑁𝑜.𝑜𝑓 𝑆𝑖𝑑𝑒𝑠 𝑁𝑜.𝑜𝑓 𝑆𝑖𝑑𝑒𝑠
𝑆𝑞𝑢𝑎𝑟𝑒𝑠 𝑝𝑒𝑟 𝑆𝑞𝑢𝑎𝑟𝑒 𝑇𝑟𝑖𝑎𝑛𝑔𝑙𝑒𝑠 𝑝𝑒𝑟 𝑇𝑟𝑖𝑎𝑛𝑔𝑙𝑒 4 + 12 16
𝐸𝑑𝑔𝑒𝑠 = = = =8
2 2 2

Shape of Faces Sides per No. of No. of Faces at


Face Face Sides Edges Vertex
𝑓 𝑠 𝑓𝑠 𝑓𝑠 𝑛
2
Cuboid Rectangle 6 4 24 12
Triangular Triangle 4 3 12 6
Pyramid
Octahedron Triangle 8 3 24 12
Square Triangle 4 3 12 8
Pyramid Square 1 4 4

§1.4.C Diagonals of a Polyhedron

1.93 Polyhedron
➢ A polyhedron is a three-dimensional figure.
➢ It is the three-dimensional equivalent of a polygon.
➢ Examples of polyhedrons: Cubes, cuboids, pyramids, and prisms

1.94 Types of Line Segments in a Polyhedron


A line connecting two vertices of a polyhedron can be classified into one of three types:
➢ Edge: A line which is part of the boundary of the polyhedron, and part of two faces is
an edge.
➢ Face Diagonals: A line lying completely on a face is called a face diagonal.
➢ Space Diagonals: A line which is not a part of any face is called a space diagonal.

Example 1.95
Find the number of edges, vertices, faces, space diagonals, and face diagonals for a cube.

P a g e 39 | 100
Get all the files at: https://bit.ly/azizhandouts
Aziz Manva (azizmanva@gmail.com)

𝐸𝑑𝑔𝑒𝑠 = 12
𝑉𝑒𝑟𝑡𝑖𝑐𝑒𝑠 = 8
𝐹𝑎𝑐𝑒𝑠 = 6
𝑆𝑝𝑎𝑐𝑒 𝐷𝑖𝑎𝑔𝑜𝑛𝑎𝑙𝑠 = 4

Face Diagonals
The number of face diagonals is
= 2 × 6 = 12

1.96 Classifying Line Segments in a Polyhedron


A polyhedron with 𝑛 vertices has
𝑛(𝑛 − 1)
𝐿𝑖𝑛𝑒 𝑆𝑒𝑔𝑚𝑒𝑛𝑡𝑠
2
These line segments can be classified as:
𝐿𝑖𝑛𝑒𝑠 𝑆𝑒𝑔𝑚𝑒𝑛𝑡𝑠 𝑗𝑜𝑖𝑛𝑖𝑛𝑔 𝑛 𝑣𝑒𝑟𝑡𝑖𝑐𝑒𝑠 = 𝐸𝑑𝑔𝑒𝑠 + 𝐹𝑎𝑐𝑒 𝐷𝑖𝑎𝑔𝑜𝑛𝑎𝑙𝑠 + 𝑆𝑝𝑎𝑐𝑒 𝐷𝑖𝑎𝑔𝑜𝑛𝑎𝑙𝑠

Example 1.97
What is the number of line segments joining the vertices of a cube?

8×7
= 28
2
12
⏟ + ⏟
4 + 12
⏟ = 28
𝐸𝑑𝑔𝑒𝑠 𝑆𝑝𝑎𝑐𝑒 𝐹𝑎𝑐𝑒
𝐷𝑖𝑎𝑔𝑜𝑛𝑎𝑙𝑠 𝐷𝑖𝑎𝑔𝑜𝑛𝑎𝑙𝑠

1.98 No. of Diagonals in a Polyhedron


𝑁𝑜. 𝑜𝑓 𝐹𝑎𝑐𝑒 𝐷𝑖𝑎𝑔𝑜𝑛𝑎𝑙𝑠 = 𝑁𝑜. 𝑜𝑓 𝐹𝑎𝑐𝑒𝑠 × 𝐷𝑖𝑎𝑔𝑜𝑛𝑎𝑙𝑠 𝑝𝑒𝑟 𝐹𝑎𝑐𝑒

This assumes that all faces are of the same type, and hence have the same number of
diagonals.

Example 1.99
A. What is the number of face diagonals of an octahedron?
B. A regular dodecahedron has 12 faces, each of 12 congruent regular pentagon
faces. Find the number of face diagonals of a regular dodecahedron.
C. Find the number of face diagonals of a heptagonal prism.

𝑂𝑐𝑡𝑎ℎ𝑒𝑑𝑟𝑜𝑛: 8 × 0 = 0
𝑅𝑒𝑔𝑢𝑙𝑎𝑟 𝐷𝑜𝑑𝑒𝑐𝑎ℎ𝑒𝑑𝑟𝑜𝑛: 5 × 12 = 60

Heptagonal Prism
The number of diagonals in the two bases is:
7×4
2× = 28
2
7 × 2 = 14
28 + 14 = 42

1.100 Diagonals of a Polyhedron


P a g e 40 | 100
Get all the files at: https://bit.ly/azizhandouts
Aziz Manva (azizmanva@gmail.com)

A polyhedron with 𝒏 vertices has



𝐿 − 𝐸
⏟ 𝑑𝑖𝑎𝑔𝑜𝑛𝑎𝑙𝑠
𝐿𝑖𝑛𝑒 𝐸𝑑𝑔𝑒𝑠
𝑆𝑒𝑔𝑚𝑒𝑛𝑡𝑠

𝑉(𝑉 − 1)
𝐿= , 𝑉 = 𝑁𝑜. 𝑜𝑓 𝑉𝑒𝑟𝑡𝑖𝑐𝑒𝑠
2

The logical method that we used for counting number of diagonals of a polygon does not easily generalize to
polyhedrons. But the other formula does.

Example 1.101
A cube has eight vertices (corners) and twelve edges. A segment, such as 𝑥, which joins two vertices not joined
by an edge is called a diagonal. Segment 𝑦 is also a diagonal. How many diagonals does a cube have? (AMC 8
1997/17)

8×7
− 12
⏟ = 28 − 12 = 16
⏟2
𝑬𝒅𝒈𝒆𝒔
𝑳𝒊𝒏𝒆 𝑺𝒆𝒈𝒎𝒆𝒏𝒕𝒔

Example 1.102
A diagonal of a polyhedron is a line segment connecting two non-adjacent vertices. How many diagonals does a
pentagonal prism have? (MathCounts 2001 National Sprint)

10 × 9
𝐿−𝐸 = − 15 = 45 − 15 = 30
2

We can also count directly:


2×5
⏟ + 5×2
⏟ = 10 + 10 = 20 𝐹𝑎𝑐𝑒 𝐷𝑖𝑎𝑔𝑜𝑛𝑎𝑙𝑠
𝐷𝑖𝑎𝑔𝑜𝑛𝑎𝑙𝑠 𝐷𝑖𝑎𝑔𝑜𝑛𝑎𝑙𝑠
𝑓𝑟𝑜𝑚 𝑅𝑒𝑐𝑡𝑎𝑛𝑔𝑙𝑒𝑠 𝑓𝑟𝑜𝑚 𝑃𝑒𝑛𝑡𝑎𝑔𝑜𝑛𝑠


5 × (5 − 3)
⏟ = 5 × 2 = 10 𝑆𝑝𝑎𝑐𝑒 𝐷𝑖𝑎𝑔𝑜𝑛𝑎𝑙𝑠
𝑉𝑒𝑟𝑡𝑖𝑐𝑒𝑠 𝑉𝑒𝑟𝑡𝑖𝑐𝑒𝑠
𝑎𝑡 𝑡ℎ𝑒 𝑡𝑜𝑝 𝑎𝑡 𝑡ℎ𝑒 𝑏𝑜𝑡𝑡𝑜𝑚
20
⏟ + 10
⏟ = 30
𝐹𝑎𝑐𝑒 𝑆𝑝𝑎𝑐𝑒
𝐷𝑖𝑎𝑔𝑜𝑛𝑎𝑙𝑠 𝐷𝑖𝑎𝑔𝑜𝑛𝑎𝑙𝑠

1.103 Space Diagonals


A polyhedron with 𝑛 vertices has number of:
𝑆𝐷
⏟ = ⏟ 𝐿𝑖𝑛𝑒 𝑆𝑒𝑔𝑚𝑒𝑛𝑡𝑠 − 𝐸
⏟ − 𝐹𝐷

𝑺𝒑𝒂𝒄𝒆 𝑫𝒊𝒂𝒈𝒐𝒏𝒂𝒍𝒔 𝑳𝒊𝒏𝒆𝒔 𝒋𝒐𝒊𝒏𝒊𝒏𝒈 𝒏 𝑷𝒐𝒊𝒏𝒕𝒔 𝑬𝒅𝒈𝒆𝒔 𝑭𝒂𝒄𝒆 𝑫𝒊𝒂𝒈𝒐𝒏𝒂𝒍𝒔

Example 1.104
An octahedron is formed by joining two congruent square pyramids at their base. Find the number of space

P a g e 41 | 100
Get all the files at: https://bit.ly/azizhandouts
Aziz Manva (azizmanva@gmail.com)

diagonals of an octahedron.

Visualization
An octahedron is formed by joining two tetrahedrons.
Face Diagonals: Each face of the octahedron is a triangle. A triangle does not diagonals. Hence, there are no face
diagonals in a tetrahedron.
Space Diagonals: Two diagonals will be formed by the four points forming a square that were a part of the base
of both the tetrahedrons. One diagonal will be formed by a line going from one tip of the octahedron to the
other tip of the octahedron.
Formula
6×5
𝐷𝑖𝑎𝑔𝑜𝑛𝑎𝑙𝑠 = − 12 − ( 8 × 0 ) = 15 − 12 = 3
2 𝐹𝑎𝑐𝑒𝑠 𝐷𝑖𝑎𝑔𝑜𝑛𝑎𝑙𝑠
𝐹𝑎𝑐𝑒

Example 1.105
Find the number of space diagonals in each case. Keep the following definitions in mind. A dodecahedron has 12
faces. An icosahedron has 20 faces.
A. A cuboid has six rectangular faces, three of which meet at each vertex.
B. A regular icosahedron with 20 congruent equilateral triangle faces.
C. A regular dodecahedron with 12 congruent regular pentagonal faces.
D. A hexarhombic dodecahedron with 8 rhombic and 4 hexagonal faces
E. A convex polyhedron 𝑃 has 36 faces, 24 triangular and 12 quadrilaterals. (AIME I 2004/3, Modified)

Part A: Cuboid 5(4)


6 × 4 24 = − 5 = 10 − 5 = 5
2
𝑁𝑜. 𝑜𝑓 𝐸𝑑𝑔𝑒𝑠 = = = 12
2 2 Total number of face diagonals:
24 = 12 × 5 = 60
𝑁𝑜. 𝑜𝑓 𝑉𝑒𝑟𝑡𝑖𝑐𝑒𝑠 = 𝑉 = =8
3 The number of space diagonals
Number of Face Diagonals 20 × 19
= 6 × 2 = 12 = − 30
⏟ − 60
⏟ = 190 − 90 = 100
⏟ 2 𝑭𝒂𝒄𝒆
𝑬𝒅𝒈𝒆𝒔
Number of Space Diagonals 𝑳𝒊𝒏𝒆 𝑫𝒊𝒂𝒈𝒐𝒏𝒂𝒍𝒔
𝑺𝒆𝒈𝒎𝒆𝒏𝒕𝒔
8×7
= − 12 − 12 = 4 Part D: Hexarhombic Dodecahedron
2 8 × 4 + 4 × 6 56
Part B: Regular Icosahedron 𝐸𝑑𝑔𝑒𝑠 = = = 28
20 × 3 60 2 2
𝐸𝑑𝑔𝑒𝑠 = = = 30 Substitute 𝐸 = 30, 𝐹 = 12 in Euler’s Formula:
2 2
Substitute 𝐸 = 30, 𝐹 = 20 in Euler’s Formula: 12 + 𝑉 = 28 + 2 ⇒ 𝑉 = 18
20 + 𝑉 = 30 + 2 ⇒ 𝑉 = 12 The number of diagonals of a hexagon
Number of space diagonals: 6(5)
= − 6 = 15 − 6 = 9
12 × 11 2
= − 30⏟ − ⏟
0 = 66 − 30 = 36 Total number of face diagonals:
⏟ 2 𝑭𝒂𝒄𝒆
𝑬𝒅𝒈𝒆𝒔 = 2 × 8 + 4 × 9 = 52
𝑳𝒊𝒏𝒆 𝑫𝒊𝒂𝒈𝒐𝒏𝒂𝒍𝒔
𝑺𝒆𝒈𝒎𝒆𝒏𝒕𝒔 The number of space diagonals
Part C: Regular Dodecahedron 18 × 17
12 × 5 60 = − 28⏟ − 52
⏟ = 73
𝐸𝑑𝑔𝑒𝑠 = = = 30 ⏟ 2 𝑭𝒂𝒄𝒆
𝑬𝒅𝒈𝒆𝒔
2 2 𝑳𝒊𝒏𝒆 𝑫𝒊𝒂𝒈𝒐𝒏𝒂𝒍𝒔
Substitute 𝐸 = 30, 𝐹 = 12 in Euler’s Formula: 𝑺𝒆𝒈𝒎𝒆𝒏𝒕𝒔

12 + 𝑉 = 30 + 2 ⇒ 𝑉 = 20 Part E: Unknown AIME Polygon


The number of face diagonals of a pentagon 24 × 3 + 12 × 4 120
𝐸𝑑𝑔𝑒𝑠 = = = 60
2 2

P a g e 42 | 100
Get all the files at: https://bit.ly/azizhandouts
Aziz Manva (azizmanva@gmail.com)

Substitute 𝐸 = 60, 𝐹 = 36 in Euler’s Formula: Number of Space Diagonals


36 + 𝑉 = 60 + 2 ⇒ 𝑉 = 62 − 36 = 26 26 × 25
= − 60 − 24 = 241
Number of Face Diagonals 2
= 0⏟× 24 + 2⏟× 12 = 0 + 24 = 24
𝑻𝒓𝒊𝒂𝒏𝒈𝒖𝒍𝒂𝒓 𝑸𝒖𝒂𝒅𝒓𝒊𝒍𝒂𝒕𝒆𝒓𝒂𝒍
𝑭𝒂𝒄𝒆𝒔 𝑭𝒂𝒄𝒆𝒔

§1.4.D Cuts in 3D Objects

Example 1.106: Making Cuts


A. A wooden cube is colored red. It has edge length 3. It is cut using 6 cuts to get a shape that looks like a
Rubik’s cube. It has 27 smaller cubes, each of edge length 1. Count the number of cubes with exactly 𝑛
sides colored. 𝑛 = {0,1,2, … }. Answer separately for each value of 𝑛.
B. The remaining sides which are not colored are now colored. Find the number of sides that need to be
colored.

Part A
First, find the maximum value of 𝑛.
𝑀𝑎𝑥(𝑛) = 3
The cubes which have three sides colored will be the corner cubes:
𝑛3 = 𝑁𝑜. 𝑜𝑓 𝑉𝑒𝑟𝑡𝑖𝑐𝑒𝑠 = 8
The cubes which have two sides colored will in the middle position in the edge:
𝑛2 = 4 × 3 = 12
The cubes which have one sides colored will in the middle position in the face:
𝑛1 = 1 × 6 = 6
There is one cube inside which has no face colored:
𝑛0 = 1
And we can verify by adding to check
8 + 12 + 6 + 1 = 27
Part B
Total Number of Sides
= 27 × 6 − 9 × 6 = 6(27 − 9) = 6(18) = 108

Example 1.107: Making Cuts


A wooden cuboid of dimensions 3 × 4 × 5 is colored red. It is cut to form 60 smaller cubes with edge length 1.
Count the number of cubes with exactly 𝑛 sides colored. 𝑛 = {0,1,2, … }. Answer separately for each value of 𝑛.

P a g e 43 | 100
Get all the files at: https://bit.ly/azizhandouts
Aziz Manva (azizmanva@gmail.com)

The cubes which have three sides colored will be the corner
cubes:
𝑛3 = 𝑁𝑜. 𝑜𝑓 𝑉𝑒𝑟𝑡𝑖𝑐𝑒𝑠 = 8

Cubes with two sides colored will be in the middle position in the
edge:
𝑛2 = 4[(3 − 2) + (4 − 2) + (5 − 2)] = 4(1 + 2 + 3) = 4(6) = 24

The cubes which have one side colored will in the middle position
in the face:

𝑛1 = 2 ((1 (1 × 3) + ⏟
⏟ × 2) + ⏟ (2 × 3) ) = 2(2 + 3 + 6) = 2(11)
3×4 𝐹𝑎𝑐𝑒 3×5 𝐹𝑎𝑐𝑒 4×5 𝐹𝑎𝑐𝑒
= 22

Cube inside which has no face colored:


𝑛0 = (3 − 2)(4 − 2)(5 − 2) = (1)(2)(3) = 6
And we can verify by adding to check:
8 + 24 + 22 + 6 = 60

Example 1.108: Making Cuts


A wooden cuboid is colored red. The cuboid has dimensions 𝑙 × 𝑤 × ℎ where 𝑙, 𝑤, ℎ are natural numbers each
greater than two. It is then cut to form 𝑙𝑤ℎ smaller cubes each with edge length 1. Count the number of cubes
with exactly 𝑛 sides colored. 𝑛 = {0,1,2, … }. Answer separately for each value of 𝑛.

The cubes which have three sides colored will be the corner cubes:
𝑛3 = 𝑁𝑜. 𝑜𝑓 𝑉𝑒𝑟𝑡𝑖𝑐𝑒𝑠 = 8

The cubes which have two sides colored will in the middle position in the edge:
𝑛2 = 4[(𝑙 − 2) + (𝑤 − 2) + (ℎ − 2)] = 4(𝑙 + 𝑤 + ℎ − 6) = 4𝑙 + 4𝑤 + 4ℎ − 24

The cubes which have one sides colored will in the middle position in the face:

⏟ − 2)(𝑤 − 2) + (𝑤
𝑛1 = 2 ((𝑙 ⏟ − 2)(ℎ − 2) + (ℎ
⏟ − 2)(𝑙 − 2))
𝑙𝑤 𝐹𝑎𝑐𝑒 𝑤ℎ 𝐹𝑎𝑐𝑒 ℎ𝑙 𝐹𝑎𝑐𝑒
= 2[(𝑙𝑤 − 2𝑙 − 2𝑤 + 4) + (𝑤ℎ − 2𝑤 − 2ℎ + 4) + (ℎ𝑙 − 2ℎ − 2𝑙 + 4)]
= 2[𝑙𝑤 + 𝑤ℎ + ℎ𝑙 − 4𝑙 − 4𝑤 − 4ℎ + 12]
= 2𝑙𝑤 + 2𝑤ℎ + 2ℎ𝑙 − 8𝑙 − 8𝑤 − 8ℎ + 24

There is one cube inside which has no face colored:


𝑛0 = (𝑙 − 2)(𝑤 − 2)(ℎ − 2)
= (𝑙𝑤 − 2𝑙 − 2𝑤 + 4)(ℎ − 2)
= 𝑙𝑤ℎ − 2ℎ𝑙 − 2𝑤ℎ + 4ℎ − 2𝑙𝑤 + 4𝑙 + 4𝑤 − 8
Verification:
Add the numbers first:
8 − 24 + 24 − 8
Take terms which have exactly have one variable. Note that 𝑛3 has no variable terms:
4𝑙 − 8𝑙 + 4𝑙 = 0
4𝑤 − 8𝑤 + 4𝑤 = 0
4ℎ − 8ℎ + 4ℎ = 0
Take terms which have exactly two variables. Note that 𝑛3 and 𝑛2 have no two-variable terms:

P a g e 44 | 100
Get all the files at: https://bit.ly/azizhandouts
Aziz Manva (azizmanva@gmail.com)

2𝑙𝑤 + 2𝑤ℎ + 2ℎ𝑙 − 2ℎ𝑙 − 2𝑤ℎ − 2𝑙𝑤 = 0


And only 𝑛1 has a three-variable term:
𝑙𝑤ℎ
Which is what it should be.

Example 1.109: Maximizing number of cuboids


I have a cuboid. I need to cut the cuboid into smaller cuboids by making cuts such that each cut is parallel to a
face of the cuboid. Find the maximum number of cuboids that I can make if I make
A. 6 Cuts
B. 7 Cuts
C. 9 Cuts
D. 11 Cuts
E. 𝑛 𝐶𝑢𝑡𝑠, 𝑛 ∈ ℕ

Part A
(1 + 1)
⏟ (1 + 1)
⏟ (4 + 1) = 2 × 2 × 5 = 20

1 𝐶𝑢𝑡 1 𝐶𝑢𝑡 4 𝐶𝑢𝑡𝑠
𝐿𝑒𝑛𝑔𝑡ℎ𝑤𝑖𝑠𝑒 𝑊𝑖𝑑𝑡ℎ𝑊𝑖𝑠𝑒 𝐻𝑒𝑖𝑔ℎ𝑡𝑤𝑖𝑠𝑒
(2 + 1)(2 + 1)(2 + 1) = 33 = 27
Part B
(2 + 1)(2 + 1)(3 + 1) = 32 × 4 = 36
Part C
(3 + 1)(3 + 1)(3 + 1) = 43 = 64
Part D
(3 + 1)(3 + 1)(5 + 1) = 42 × 6 = 96
(3 + 1)(4 + 1)(4 + 1) = 4 × 52 = 100
Part E
𝑛 is a multiple of 3:𝑛 = 3𝑥
(𝑥 + 1)(𝑥 + 1)(𝑥 + 1)
𝑛 is one more than a multiple of 3:𝑛 = 3𝑥 + 1
(𝑥 + 1)(𝑥 + 1)(𝑥 + 2)
𝑛 is two more than a multiple of 3:𝑛 = 3𝑥 + 2
(𝑥 + 1)(𝑥 + 2)(𝑥 + 2)

Example 1.110
Each face of a cube is painted with exactly one color. What is the smallest number of colours needed to paint a
cube so that no two faces that share an edge are the same color? (Gauss Grade 7 2014/22)

Suppose that the top of the cube is colored blue.


Then, the front shares an edge with the top, and must be colored a
different color, say purple.
The left shares an edge with both the top and the front and must be
colored a third colour, say pink.
(Hence, we now know that two colors are not sufficient to color the cube).

Now, color the


right as pink
back as purple
bottom as blue

P a g e 45 | 100
Get all the files at: https://bit.ly/azizhandouts
Aziz Manva (azizmanva@gmail.com)

And now this meets all the conditions given in the question.
Hence, the final answer is 3.

Example 1.111
A. Each face of an octahedron is numbered from 1 to 8. The top four faces are numbered from 1 to 4. The
face below 1 is numbered 5, the face below 2 is numbered 6. What is the smallest number of colours
needed to paint the faces so that no two faces that share an edge are the same color? Give the color
scheme: 𝐹𝑎𝑐𝑒 1: 𝐶𝑜𝑙𝑜𝑟 𝑋, 𝐹𝑎𝑐𝑒 2: 𝐶𝑜𝑙𝑜𝑟 𝑌.
B. Pyramid 𝑃1 has a regular dodecagon for its base. It is joined to identical pyramid 𝑃2 at the base. Each
face of 𝑃1 is numbered from 1 to 12. Each face of 𝑃2 is numbered from 13 𝑡𝑜 24, with 13 below 1, 14
below 2 and so on. The shape is colored with the minimum number of colors such that no two faces that
share an edge have the same color. If red is one of the colors used, find the sum of the face numbers that
are colored red.

Hints: Dodecagon is a 12-sided polygon

Part A
𝑅𝑒𝑑: 1,3,6,8
𝐵𝑙𝑢𝑒: 2,4,5,7
Part B
𝐵𝑙𝑢𝑒: 1,3,5,7,9,11,14,16,18,20,22,24
𝑅𝑒𝑑: 2,4,6,8,10,12,13,15,17,19,21,23

2 + 4 + 6 + 8 + 10 + 12 + 13 + 15 + 17 + 19 + 21 + 23
25 × 12 300
= = = 150
2 2
Shortcut:
25 × 24
1 + 2 + ⋯ + 24 2
= = 25 × 6 = 150
2 2

Example 1.112
The large cube shown is made up of 27 identical sized smaller cubes. For each face of the large cube, the
opposite face is shaded the same way. The total number of smaller cubes that must have at least one face shaded
is (AMC 8 1987/7)

None of the cubes have more than one face painted. We can see
10 𝐹𝑎𝑐𝑒𝑠
And we can see half the cube.

So, the total of number of cubes with exactly one face painted
10 × 2 = 20

P a g e 46 | 100
Get all the files at: https://bit.ly/azizhandouts
Aziz Manva (azizmanva@gmail.com)

Example 1.113
Akash's birthday cake is in the form of a 4 × 4 × 4 inch cube. The cake has icing on the top and the four side
faces, and no icing on the bottom. Suppose the cake is cut into 64 smaller cubes, each measuring 1 × 1 × 1
inch, as shown below. How many small pieces will have icing on exactly two sides? (AMC 8 2020/9)

From the top layer


2+2+2+2=8
𝑛𝑑 𝑟𝑑
From the 2 and 3 layer:
2𝑛𝑑: 1 + 1 + 1 + 1 = 4
3𝑟𝑑: 1 + 1 + 1 + 1 = 4
From the bottom layer, we get the four corner cubes:
1+1+1+1=4

8 + 4 + 4 + 4 = 20

Example 1.114
A wooden cube 𝑛 units on a side is painted red on all six faces and then cut into 𝑛3 unit cubes. Exactly one-
fourth of the total number of faces of the unit cubes are red. What is 𝑛? (AMC 10A 2005/11)

Surface area of all faces of the unit cubes


= ⏟
6 ⏟3 = 6𝑛3
× 𝑛
𝑆𝑢𝑟𝑓𝑎𝑐𝑒 𝐴𝑟𝑒𝑎 𝑁𝑜. 𝑜𝑓
𝑜𝑓 1 𝐶𝑢𝑏𝑒 𝐶𝑢𝑏𝑒𝑠

Since the cubes are unit cubes, area of each face is 1. Surface area of all faces of the original cube (which is also
equal to the number of faces of the unit cubes which are colored red):
= 6𝑛2

1
6𝑛2 = (6𝑛3 ) ⇒ 4𝑛2 = 𝑛3 ⇒ 4 = 𝑛
4

Example 1.115
Fourteen white cubes are put together to form the figure on the right. The
complete surface of the figure, including the bottom, is painted red. The figure is
then separated into individual cubes.
A. How many of the individual cubes have exactly four red faces? (AMC 8
2003/13)
B. How many of the individual cubes have exactly five red faces?
C. How many of the individual cubes have exactly three red faces?

P a g e 47 | 100
Get all the files at: https://bit.ly/azizhandouts
Aziz Manva (azizmanva@gmail.com)

𝐸𝑥𝑎𝑐𝑡𝑙𝑦 4 𝑅𝑒𝑑 𝐹𝑎𝑐𝑒𝑠: 6


𝐸𝑥𝑎𝑐𝑡𝑙𝑦 5 𝑅𝑒𝑑 𝐹𝑎𝑐𝑒𝑠: 4
𝐸𝑥𝑎𝑐𝑡𝑙𝑦 3 𝑅𝑒𝑑 𝐹𝑎𝑐𝑒𝑠: 4

Example 1.116
A 4 × 4 × 4 cubical box contains 64 identical small cubes that exactly fill the box. How many of these small
cubes touch a side or the bottom of the box? (AMC 8 1998/21)

Cubes that we do not want:


𝑇𝑜𝑝 𝐿𝑎𝑦𝑒𝑟: 4 𝐶𝑢𝑏𝑒𝑠
2𝑛𝑑 𝐿𝑎𝑦𝑒𝑟: 4 𝐶𝑢𝑏𝑒𝑠
3𝑟𝑑 𝐿𝑎𝑦𝑒𝑟: 4 𝐶𝑢𝑏𝑒𝑠

𝑇𝑜𝑡𝑎𝑙 = 4 + 4 + 4 = 12
Cubes that we do want
= 64 − 12 = 52

Example 1.117
Three dice with faces numbered 1 through 6 are stacked as shown. Seven of the eighteen faces
are visible, leaving eleven faces hidden (back, bottom, between). The total number of dots
NOT visible in this view is (AMC 8 2000/8)

The total number of dots on a die is


1 + 2 + 3 + 4 + 5 + 6 = 21
The total dots visible
= 1 + 2 + 3 + 4 + 5 + 6 + 1 = 22
The non visible dots
= 21 × 3 − 22 = 41

Example 1.118
How many pairs of parallel edges, such as 𝐴𝐵 and 𝐺𝐻 or 𝐸𝐻 and 𝐹𝐺, does a cube have?
(AMC 8 2015/12)

Note that every edge is parallel to three other edges.


The total number of pairs is
1 1
12
⏟ × ⏟
3 × = 36 × = 18
𝑁𝑜.𝑜𝑓 𝐸𝑑𝑔𝑒𝑠 𝑡𝑜

2 2
𝐸𝑑𝑔𝑒𝑠 𝑝𝑎𝑖𝑟 𝑤𝑖𝑡ℎ 𝑂𝑣𝑒𝑟𝑐𝑜𝑢𝑛𝑡𝑖𝑛𝑔
𝐹𝑎𝑐𝑡𝑜𝑟

§1.4.E Eulerian Paths

1.119 Eulerian Path


An Eulerian path is a path that travel along each edge of an object exactly once.
➢ If the start point and end point are different, it is called an Eulerian trail.
➢ If the start point and end point are the same, it is called an Eulerian cycle.

Example 1.120
P a g e 48 | 100
Get all the files at: https://bit.ly/azizhandouts
Aziz Manva (azizmanva@gmail.com)

A. An ant at vertex A of the cube wants to travel along the edges without repeating any. What is the
maximum length of path possible? Give one such path.

It is possible to get a path of length nine edges. For example:


𝐴𝐵𝐶𝐷𝐴𝐸𝐹𝐺𝐻𝐸 ⇒ 9 𝐸𝑑𝑔𝑒𝑠
The edges that remain unutilized are:
𝐹𝐵, 𝐷𝐻, 𝐶𝐺

It remains to demonstrate that you cannot have a path of more than nine edges.
Note that each vertex has exactly three edges joined together.

➢ To travel along an edge, without repeating an edge, you must arrive at a vertex via an edge, and leave via
a different edge.

Consider two types of vertices:


➢ Type I: Where the path starts (and also ends).
➢ Type II: Which is not of Type I

𝑨𝑩𝐶𝑫𝑨𝑬𝐹𝐺𝐻𝐸 ⇒ 9 𝐸𝑑𝑔𝑒𝑠
Consider the vertex A, which is of Type I:
𝐴𝐵, 𝐷𝐴, 𝐴𝐸 ⇒ 𝐴𝑙𝑙 𝐸𝑑𝑔𝑒𝑠 𝑐𝑜𝑛𝑛𝑒𝑐𝑡𝑒𝑑 𝑡𝑜 𝐴 𝑤𝑒𝑟𝑒 𝑢𝑡𝑖𝑙𝑖𝑧𝑒𝑑

Consider any vertex, which is of Type II:


➢ Any path that passes through a vertex of Type II must utilize an even number of edges connected to the
vertex.

Hence, the maximum number of edges that can be traversed is



2 × ⏟
3 + ⏟ 6 × ⏟
2 = 6 + 12 = 18
𝑇𝑦𝑝𝑒 𝐼 𝑈𝑡𝑖𝑙𝑖𝑧𝑎𝑏𝑙𝑒 𝑇𝑦𝑝𝑒 𝐼𝐼 𝑈𝑡𝑖𝑙𝑖𝑧𝑎𝑏𝑙𝑒
𝑉𝑒𝑟𝑡𝑒𝑥 𝐸𝑑𝑔𝑒𝑠 𝑉𝑒𝑟𝑡𝑒𝑥 𝐸𝑑𝑔𝑒𝑠
However, this overcounts by a factor of 2, since an edge from A to B is the same as an edge from B to A.
Hence, the maximum length of path is:
18
=9
2

1.121 Degree of A Vertex


The number of edges connected to a vertex is called the degree
of the vertex.

1.122 Condition for Eulerian Path


➢ Any graph with an Eulerian path must have exactly zero or two vertices of odd degree.

Example 1.123: Seven Bridges of Konigsberg


This is a famous problem solved by Leonard Euler. Consider the graph alongside. Show that there is no path
traverses all the edges exactly once.

Count the number of edges connected to each vertex:


𝐴: 5, 𝐵: 3, 𝐶: 3, 𝐷: 3

P a g e 49 | 100
Get all the files at: https://bit.ly/azizhandouts
Aziz Manva (azizmanva@gmail.com)

All the vertices have an odd number of edges. In a path that goes through the edges without repeating any edge,
the maximum number of vertices that can have an odd number of edges is 2.
Hence, such a path is not possible.

1 × 5 + 1 × 3 + 2 × 2 = 5 + 3 + 4 = 12
12
=6
2

𝐴𝐷, 𝐷𝐴, 𝐴𝐶, 𝐶𝐴, 𝐴𝐵, 𝐵𝐷

Example 1.124
Show that there is no path that travels through each of the edges of the
adjoining graph exactly once.

𝐸, 𝐹, 𝐺, 𝐻: 𝐴𝑙𝑙 5 ℎ𝑎𝑣𝑒 5 𝑒𝑑𝑔𝑒𝑠 𝑒𝑎𝑐ℎ

To have a path that travels through each edge exactly once, the number of vertices with an odd number of edges
must be exactly zero or two.
In this case, it is 5. Hence, such a path is not possible.

§1.4.F Paths along Faces

Example 1.125
For each geometrical object below, how many ways are there to move from the top face to the bottom face via a
sequence of adjacent faces so that each face is visited at most once and moves are permitted down, left, and
right, but not up.
A. Cube
B. Hexagonal Prism standing on its base

Part A
Any sequence of moves that meets the above condition must have the
following parts:
➢ Step I: Move from top face to a side face:
4 𝑊𝑎𝑦𝑠
➢ Step II: Move along the side faces:
For example, if you pick 2 as your side face, then the options are:
2,23,234,2345,25,254,2543 ⇒ 7 𝑊𝑎𝑦𝑠
➢ Step III: From side face to bottom face:
1 𝑊𝑎𝑦𝑠
Hence, the final answer, by the multiplication principle
= 4 × 7 = 28
Part B
Any sequence of moves that meets the above condition must have the following parts:
➢ Step I: Move from top face to a side face:
6 𝑊𝑎𝑦𝑠
➢ Step II: Move along the side faces:
1 + 5 + 5 = 11 𝑊𝑎𝑦𝑠
➢ Step III: From side face to bottom face:

P a g e 50 | 100
Get all the files at: https://bit.ly/azizhandouts
Aziz Manva (azizmanva@gmail.com)

1 𝑊𝑎𝑦𝑠
Hence, the final answer, by the multiplication principle
= 6 × 11 = 66 𝑊𝑎𝑦𝑠
Part C

Example 1.126
A hexagonal Prism is kept so that one of its six rectangular faces is
the base. How many ways are there to move from the top face to the
bottom face via a sequence of adjacent faces so that each face is
visited at most once?

Example 1.127
As shown in the figure below, a regular dodecahedron (the polyhedron
consisting of 12 congruent regular pentagonal faces) floats in space with two
horizontal faces. Note that there is a ring of five slanted faces adjacent to the top
face, and a ring of five slanted faces adjacent to the bottom face. How many ways
are there to move from the top face to the bottom face via a sequence of
adjacent faces so that each face is visited at most once and moves are not
permitted from the bottom ring to the top ring? (AMC 10A 2020/19)

Any sequence of moves that meets the above


condition must have the following parts:
➢ Part A: Move from top face to top ring
5 𝑊𝑎𝑦𝑠
➢ Part B: Move along the top ring
1, 12,123,1234,12345 ⇒ 5 𝑂𝑝𝑡𝑖𝑜𝑛𝑠
15,154,1543,15432 ⇒ 4 𝑂𝑝𝑡𝑖𝑜𝑛𝑠
𝑇𝑜𝑡𝑎𝑙 𝑂𝑝𝑡𝑖𝑜𝑛𝑠 = 9 𝑊𝑎𝑦𝑠
➢ Part C: From top ring to bottom ring
2 𝑊𝑎𝑦𝑠
➢ Part D: Move along the bottom ring
The bottom ring is symmetrical to the top ring,
and hence the number of ways to travel along
the bottom is exactly the same as the number of ways to travel along the top ring:
9 𝑊𝑎𝑦𝑠
➢ Part E: Move to bottom face
1 𝑊𝑎𝑦

In order to move from top face to bottom face, each of the above five parts, must be done, in sequence, and any
of the choices can be combined in any way that you want.
Hence, the final answer, by the multiplication principle
= 5 × 9 × 2 × 9 = 10 × 81 = 810

P a g e 51 | 100
Get all the files at: https://bit.ly/azizhandouts
Aziz Manva (azizmanva@gmail.com)

§2. COMBINATIONS
§2.1 Combinations
§2.1.A Basics

2.1: Factorial
𝑛! (read n factorial) is the product of the first 𝑛 natural numbers.
𝑛! = 1 × 2 × … × 𝑛, 𝑛 ≥ 1
𝑛! = 1, 𝑛 = 0

2.2: Combination Formula: Choosing 𝒓 objects out of 𝒏 objects


The number of ways to choose 𝑟 objects out of 𝑛 objects is given by:
𝑛 𝑛!
( )=
𝑟 𝑟! (𝑛 − 𝑟)!

𝑛
( ) 𝑖𝑠 𝑟𝑒𝑎𝑑 𝒏 𝑐ℎ𝑜𝑜𝑠𝑒 𝒓
𝑟

The number of ways of arranging 𝑟 distinct objects in 𝑛 positions is


𝑛!
𝑛 × (𝑛 − 1) × (𝑛 − 2) × … × (𝑛 − 𝑟 + 1) =
(𝑛 − 𝑟)!

If we are arranging 𝑟 identical objects in 𝑛 positions, then we are only choosing the positions, not the objects
that go into the positions.
Hence, we are overcounting by the number of ways to arrange the 𝑟 objects among themselves. And hence, we
need to divide by 𝑟!
𝑛! 𝑛! 1 𝑛!
÷ 𝑟! = × =
(𝑛 − 𝑟)! (𝑛 − 𝑟)! 𝑟! 𝑟! (𝑛 − 𝑟)!

2.3: Choosing 𝒓 objects out of 𝒏 objects


𝑛 𝑛
𝐶𝑟 = ( )
𝑟

Example 2.4

Calculate:
A. (42)
B. (64), (62)
C. (75)
D. (10
3
)
E. (85)
F. (52)
G. (73)
H. (69)

Part A

P a g e 52 | 100
Get all the files at: https://bit.ly/azizhandouts
Aziz Manva (azizmanva@gmail.com)

𝒏!
Substitute 𝑛 = 4, 𝑟 = 2 in (𝒏𝒓) = :
𝒓!(𝒏−𝒓)!
4 4! 4!
( )= =
2 2! (4 − 2)! 2! 2!
Expand and simplify:
4×3×𝟐×𝟏 4×3
= =2×3=6
(2 × 1)(𝟐 × 𝟏) 2
Part B
6 6! 6 × 5 × 𝟒! 6 × 5
( )= = = = 15
4 4! 2! 𝟒! 2! 2
6 6! 6
( )= = ( ) = 15
2 2! 4! 4
Part C
7 7! 7 × 6 × 𝟓! 7 × 6
( )= = = = 21
5 5! 2! 𝟓! 2! 2×1
Part D
10 10! 10 × 9 × 8 × 𝟕! 10 × 9 × 8
( )= = = = 120
3 3! 7! 3! 𝟕! 3!

Part E
8 8! 8×7×6
( )= = = 56
5 5! 3! 3!
Part F
5 5! 5×4
( )= = = 10
2 2! 3! 2
Part G
7 7! 7×6×5
( )= = = 35
3 3! 4! 6
Part H
9 9! 9×8×7
( )= = = 84
6 3! 6! 2×3

2.5: Interpreting 𝒏 choose 𝒓


𝑛
Given ( ) we can interpret it as choosing 𝑟 objects out of 𝑛 objects.
𝑟

This may seem quite trivial, but is very useful in some difficult questions.

Example 2.6
Interpret the following as choosing 𝒓 objects out of 𝒏.
10
A. ( )
4
7
B. ( )
5
45
C. ( )
23

Part A
The total number of objects is 10.
The number of objects that we are choosing is 4.
10
Hence, ( ) is the number of ways of choosing 4 objects out of 10.
4

P a g e 53 | 100
Get all the files at: https://bit.ly/azizhandouts
Aziz Manva (azizmanva@gmail.com)

Part B
Number of ways of choosing 5 objects out of 7.

Part K
Number of ways of choosing 23 objects out of 45.

2.7: Properties of Combinations


The individual properties are not very difficult. But recognizing which property is applicable is a key skill that is
needed to solve difficult questions. Each of the questions below uses a property from the ones just discussed.
Identify which one, and use it.
𝑛 𝑛
( )=( )=1
0 𝑛
𝑛
( )=𝑛
1
𝑛 𝑛(𝑛 − 1)
( )=
2 2

Choosing zero objects


Number of ways of choosing zero objects out of 𝑛 objects is:
𝑛 𝑛! 𝑛! 𝑛!
( )= = = =1
0 0! (𝑛)! 1 × 𝑛! 𝑛!
Recall that, by definition, zero factorial is one. This helps in simplifying a lot of calculations.
Logically speaking, there is only one way of not choosing 𝑛 objects, which is to not choose all of them.

Choosing all objects


Number of ways of choosing 𝑛 objects out of 𝑛 objects is:
𝑛 𝑛! 𝑛! 𝑛!
( )= = = =1
𝑛 0! (𝑛)! 1 × 𝑛! 𝑛!

Choosing/Rejecting one object out of 𝒏


Choosing one out of 𝑛 objects will be:
𝑛 𝑛!
( )= =𝑛
1 1! (𝑛 − 1)!
Logically speaking if you have 𝑛 objects, and you want to choose one of them, you can do it in 𝑛 ways.

Choosing/Rejecting two objects out of n


Choosing two out of 𝑛 objects will be
𝑛 𝑛! 𝑛 × (𝑛 − 1) × (𝑛 − 2)! 𝑛(𝑛 − 1)
( )= = =
2 2! (𝑛 − 2)! 2! (𝒏 − 𝟐)! 2

Example 2.8
A. I have five Chinese restaurants in my locality, and my family has decided to go to one of them for dinner.
I don’t like Chinese, and hence I will not visit any of the restaurants. In how many ways can I do this?
B. I have nine favorite 𝐻𝑖𝑛𝑑𝑖 action movies. Today is the last day of vacation, and I want to maximize my
day. I have decided to watch one movie out of the nine. In how many ways can I do this?
C. I have nine favorite 𝐻𝑖𝑛𝑑𝑖 action movies. Every movie last exactly three hours. Today is the last day of
vacation, and I want to maximize my day. I have twenty-four hours to watch the maximum number of
movies that I can. In how many ways can I do this?
D. I have a fancy-dress party. I want to select exactly one mystery pair in the party: both of whom will play

P a g e 54 | 100
Get all the files at: https://bit.ly/azizhandouts
Aziz Manva (azizmanva@gmail.com)

the role of the heroes. If there are 4 people in the party, in how many ways can I do this?
E. If the above question is changed so that one person in the pair plays the role of hero, and the other
becomes a villain, then how many pairs can I make?

Part A
I can do it in 1 way only. Not go to any of them.

Part B
Logically, you are going to watch one movie out of 9, hence, you can do this in
9 𝑊𝑎𝑦𝑠

9 9!
( )= = 9 𝑊𝑎𝑦𝑠
1 1! 8!
Part C
Number of movies that he can watch is:
24
= 8 𝑀𝑜𝑣𝑖𝑒𝑠
3

Logically, you are going to watch eight movies out of 9. Choosing 8 movies is the same as rejecting 1 movie.
Hence, you can do this in
9 𝑊𝑎𝑦𝑠

Choosing 8 Movies out of 9:


9 9!
( )= = 9 𝑊𝑎𝑦𝑠
8 8! 1!

Choosing 8 movies is the same as rejecting 1 movie. Hence, we need to decide 1 movie to reject which can be
done in:
9 9!
( )= = 9 𝑊𝑎𝑦𝑠
1 1! 8!
Part D
Method I: Enumeration
Suppose the people in the party are
𝐴𝐵𝐶𝐷

Then the number of ways of choosing the mystery pair is:


𝐴𝐵, 𝐴𝐶, 𝐴𝐷
𝐵𝐶, 𝐵𝐷
𝐶𝐷

Note that choosing AB as your mystery pair is the same as choosing BA as your mystery, because both of these
are going to play the role of hero.

Method II: Formula


There are four people in the party, of whom we want to choose 2.
Hence, the number of ways of doing this is:
4 4! 4×3
4 𝑐ℎ𝑜𝑜𝑠𝑒 2 = ( ) = = =6
2 2! 2! 2
Part E
Since order is important here the number of pairs will exactly double, giving us:

P a g e 55 | 100
Get all the files at: https://bit.ly/azizhandouts
Aziz Manva (azizmanva@gmail.com)

6 × 2 = 12

Example 2.9
I am planning a trip to Europe. I have shortlisted ten cities to visit. If the order in which I visit the cities is not
important, what is the number of ways in which I can visit (answer each question independently):
A. All of them
B. One of them
C. Two of them
D. Eight of them
E. Nine of them
F. None of them

Since the order of visiting the cities is not important, we are choosing, not arranging. Hence, we need to use
combinations.
Part A
There are ten cities, and we are choosing all ten. This can be done in:
10
( ) = 1 𝑊𝑎𝑦
10
Part B
There are ten cities, and we are choosing one of them to visit. This can be done in:
10
( ) = 10 𝑊𝑎𝑦𝑠
1
Part C
There are ten cities, and we are choosing two to visit. This can be done in:
10 10 × 9
( )= = 45 𝑊𝑎𝑦𝑠
2 2
Part D
There are ten cities, and we are choosing eight to visit. This is the same as rejecting two cities to visit, and hence
it can be done in:
10 10 10 × 9
( )=( )= = 45 𝑊𝑎𝑦𝑠
8 2 2
Part E
There are ten cities, and we are choosing nine of them to visit. This is the same as rejecting a single city, and this
can be done in:
10 10
( ) = ( ) = 10 𝑊𝑎𝑦𝑠
9 1
Part F
There are ten cities, and we are choosing zero cities, which is the same as rejecting all ten cities. This can be
done in:
10 10
( ) = ( ) = 1 𝑊𝑎𝑦
0 10

2.10: Symmetry in Combinations


There is a powerful symmetry in the formula for combinations, which can be seen in the following relation.
𝑛 𝑛
( )=( )
𝑟 𝑛−𝑟

Note that the expression below is an identity since:

P a g e 56 | 100
Get all the files at: https://bit.ly/azizhandouts
Aziz Manva (azizmanva@gmail.com)

➢ the numerator of the LHS is equal to the numerator of the RHS.


➢ The denominator has the same terms, but in a different order, and since the terms are multiplied, the
value is the same.
𝑛! 𝑛!
=
𝑟! (𝑛 − 𝑟)! (𝑛 − 𝑟)! 𝑟!
But note that
𝑛! 𝑛
𝐿𝐻𝑆 = =( )
𝑟! (𝑛 − 𝑟)! 𝑟
𝑛! 𝑛
𝑅𝐻𝑆 = =( )
(𝑛 − 𝑟)! 𝑟! 𝑛−𝑟
Hence
𝑛 𝑛
( )=( )
𝑟 𝑛−𝑟

Example 2.11
Determine the value of 𝑥 in each case:
17 17
A. ( ) = ( ) , 𝑥 ≠ 3
3 𝑥
12 12
B. ( ) = ( ) , 𝑥 ≠ 5
5 𝑥
14 14
C. ( ) = ( ) , 𝑥 ≠ 0
0 𝑥

17 17
( )=( )
3 14
12 12
( )=( )
5 7
14 14
( )=( )
0 14

2.12: Examples of Symmetry in Combinations


The most use of symmetry is encountered by substituting the first three positive values of 𝑟:
𝑛 𝑛
𝑟 = 0: ( ) = ( ) = 1
0 𝑛
𝑛 𝑛
𝑟 = 1: ( ) = ( )=𝑛
1 𝑛−1
𝑛 𝑛 𝑛(𝑛 − 1)
𝑟 = 2: ( ) = ( )=
2 𝑛−2 2

While symmetry is very powerful, the above three examples of symmetry are encountered very often.
Hence, it is important to recognize and recall them instantly.

Example 2.13
20
Calculate ( )
18

20 20 20 × 19
( )=( )= = 10 × 19 = 190
18 2 2
§2.1.B Scenarios

Example 2.14
A. I have a team of eleven basketball players, of whom I must choose the starting five. What is the number

P a g e 57 | 100
Get all the files at: https://bit.ly/azizhandouts
Aziz Manva (azizmanva@gmail.com)

of ways in which I can do this?


B. I have a team of soccer players that has ten people who practice. I need to select three of the best players
for extra practice. What is the number of ways in which I can make the selection? What if I want to rank
the three best players, not just select them?
C. I must choose four out of six friends to invite for a party. In how many ways can I do this?
D. I am applying for college to the US, and need to list my three favorite books. I have made a shortlist of six
books. In how many ways can I do this, if the order does not matter? If the order does matter?
E. There are nine applicants applying for three positions of Software Engineer. If the candidates are equally
capable, in how many ways can the selection be made?
F. Saloni has shortlisted seven activities that she wants to do during the vacation. If she only has time for
four of them, in how many ways can she choose the activities?
G. Election for Officers are taking place at my favorite fraternity. There are 9 candidates of whom six are to
be selected. In how many ways can the winners be selected?

Part A
Number of ways to arrange five players out of eleven players is:
11!
11 × 10 × 9 × 8 × 7 =
6!
We want to choose, not arrange. So, we need to divide by the number of ways of arranging five players among
themselves.
Hence, the final number of ways is:
11 11!
( )=
5 5! 6!
Part B
Formula
Number of ways to choose three players out of ten:
10 10! 10 × 9 × 8
=( )= = = 10 × 3 × 4 = 120
3 3! 7! 2×3
Method II: Selecting using Permutations
Number of ways of selecting and arranging three players out of ten is:
10
⏟ × ⏟ 9 × ⏟ 8 = 10 × 9 × 8
𝐹𝑖𝑟𝑠𝑡 𝑆𝑒𝑐𝑜𝑛𝑑 𝑇ℎ𝑖𝑟𝑑
𝑃𝑙𝑎𝑦𝑒𝑟 𝑃𝑙𝑎𝑦𝑒𝑟 𝑃𝑙𝑎𝑦𝑒𝑟
However, you wish to only select, and not arrange. Hence, we are overcounting by the number of ways that three
players can be arranged in a row.
10 × 9 × 8
∴ 𝑁𝑜. 𝑜𝑓 𝑤𝑎𝑦𝑠 𝑡𝑜 𝑠𝑒𝑙𝑒𝑐𝑡 = = 120
3!
Ranking
10
⏟ × ⏟ 9 × ⏟ 8 = 10 × 9 × 8
𝐹𝑖𝑟𝑠𝑡 𝑆𝑒𝑐𝑜𝑛𝑑 𝑇ℎ𝑖𝑟𝑑
𝑃𝑙𝑎𝑦𝑒𝑟 𝑃𝑙𝑎𝑦𝑒𝑟 𝑃𝑙𝑎𝑦𝑒𝑟

Part C
Using Combinations
We need to choose 4 objects out of 6 objects, which can be done is
6 6! 6×5
( )= = = 15
4 4! 2! 2
Using Overcounting
The number of ways to arrange four friends out of six is:
6×5×4×3
We need to only select the friends, not arrange them, and hence we need to divide by the number of ways in

P a g e 58 | 100
Get all the files at: https://bit.ly/azizhandouts
Aziz Manva (azizmanva@gmail.com)

which four friends can be arranged


6×5×𝟒×𝟑 6×5
= = 15
𝟒×𝟑×2×1 2
Part D
Using Combinations
We need to choose 3 objects out of 6 objects, which can be done is
6 6! 6×5×4
( )= = = 20
3 3! 3! 6
Using Overcounting
The number of ways to arrange three objects out of six is:
6×5×4
We need to only select the objects, and not arrange them, and hence we need to divide by the number of ways in
which three objects can be arranged:
6×5×4
= 20
3×2×1
Part E
We need to choose 3 objects out of 9 objects, which can be done is
9 9! 9×8×7
( )= = = 84
3 3! 6! 6
Part F
We need to choose 4 objects out of 7 objects, which can be done is
7 7! 7×6×5
( )= = = 35
4 4! 3! 6
Part G
We need to choose 6 objects out of 9 objects, which can be done in
9 9! 9×8×7
( )= = = 84 𝑊𝑎𝑦𝑠
6 6! 3! 6

Example 2.15
I am planning a hiking trail. I have eight populated towns that are near enough to forested areas to be visited as
part of the hike. I need to choose three towns for this purpose. In how many ways can I do this, if:
A. the order of visiting the towns is not important?
B. the order of visiting the towns is important?

Using Combinations
We need to choose 3 objects out of 8 objects, which can be done is
8 8! 8×7×6
( )= = = 56
3 3! 5! 6
Using Overcounting
The number of ways to arrange three objects out of eight is:
8×7×6
We need to only select the objects, and not arrange them, and hence we need to divide by the number of ways in
which three objects can be arranged:
8×7×6
= 56
6
Part B
We can visit the towns in
8!
8𝑃3 = = 8 × 7 × 6 = 336
5!

P a g e 59 | 100
Get all the files at: https://bit.ly/azizhandouts
Aziz Manva (azizmanva@gmail.com)

§2.1.C Choosing between Permutations and Combinations

2.16: Choices with Repetition


If I have to make 𝑟 independent choices, for each of which I have 𝑛 options, then the number of ways in which I
can do it is:
𝑛 × 𝑛 × … × 𝑛 = 𝑛𝑟

𝒓 𝒕𝒊𝒎𝒆𝒔

2.17: Examples of Choices with Repetitions


➢ Ten people each of whom will pass or fail a course = 210 ≠ 2 × 10
➢ Ordering Continental, Indian or Chinese cuisine for each day from Monday to Friday, with repetition
allowed (= 35 ≠ 3 × 5)

2.18: Examples of Arrangements (Permutations)


Some of the common/classic situations asked in exam questions which 𝑜𝑓𝑡𝑒𝑛1 lead to permutations are:
➢ Arranging books in a row
➢ Seating people in a row
➢ Selecting top three ranks in a class to receive gold, silver, bronze medals
➢ Arranging letters in a license plate
➢ Forming options to break a code or a lock
➢ Ranking a team of players

In all of the above situations, order is important. Remember that arrangements make use of permutations
where
𝑛!
No. of ways of arranging 𝑟 out of 𝑛 distinct objects is:
(𝑛 − 𝑟)!

2.19: Examples of Selections (Combinations)


➢ Selecting books to read
➢ Choosing people to sit
➢ Selecting three people in a class to receive identical achievement medals
➢ Forming a team of players
➢ Forming a committee from a set of people
➢ Forming Subsets

In all of the above situations, order is 𝑛𝑜𝑡 important. Remember that selections make use of combinations
where
𝑛!
No. of ways of choosing 𝑟 out of 𝑛 distinct objects is:
𝑟! (𝑛 − 𝑟)!

Example 2.20

1The emphasis is on often, since you should not memorize keywords. It is possible to create questions that have keywords
associated with certain concepts, but because of the phrasing of the question, use a different concept.

P a g e 60 | 100
Get all the files at: https://bit.ly/azizhandouts
Aziz Manva (azizmanva@gmail.com)

In how many ways can I order pizza over four days, if


A. every day I will order a different topping out of tomatoes, bell peppers, chicken, and green chillies?
B. every day I will order one topping out of tomatoes, bell peppers, chicken, and green chillies?

Part A
I need to select a different topping each day of the week.
Order is important, since having chicken on Tuesday is not the same as having on Wednesday.
Total Ways
= 𝑊𝑎𝑦𝑠 𝑡𝑜 𝑎𝑟𝑟𝑎𝑛𝑔𝑒 𝑓𝑜𝑢𝑟 𝑖𝑡𝑒𝑚𝑠 = 4!

Part B
44 = 28 = 256

Example 2.21
In how many ways can I choose three of five main courses, two desserts of four, and add-on an (optional)
entrée?

The number of ways to choose three of five main courses is:


5
( ) = 10 𝐶ℎ𝑜𝑖𝑐𝑒𝑠
3
The number of ways to choose two desserts out of four is:
4
( ) = 6 𝐶ℎ𝑜𝑖𝑐𝑒𝑠
2
With respect to the entrée, you have two choices (add, or don’t add):
2 𝐶ℎ𝑜𝑖𝑐𝑒𝑠

And, by the multiplication rule (since each of the choices can be mixed in whichever way you want)
10
⏟ × ⏟ 6 × ⏟ 2 = 120
𝑀𝑎𝑖𝑛 𝐷𝑒𝑠𝑠𝑒𝑟𝑡𝑠 𝐸𝑛𝑡𝑟𝑒𝑒
𝐶𝑜𝑢𝑟𝑠𝑒

Example 2.22
A. In how many ways can I get the three chemistry toppers to sit in the three front row seats, the five
physics toppers to sit in the five second row seats, and the seven math toppers to sit in the seven third
row seats?
B. Answer the above question if the chemistry toppers sit in the third row, the physics toppers sit in the
first row, and the math toppers sit in the second row. (Only focus on seating arrangements, and keep in
mind that one seat can sit a maximum of one person).

Part A
3! × 5! × 7!
Part B
Number of ways to seat the chemistry toppers:
7
= ( ) × 3!
3

Number of ways to seat the physics toppers:


5
= ( ) × 3!
3

Number of ways to seat the math toppers:

P a g e 61 | 100
Get all the files at: https://bit.ly/azizhandouts
Aziz Manva (azizmanva@gmail.com)

7
= ( ) × 5!
5

§2.1.D Contrasting arrangements, selections and repetition

Example 2.23
I have seven novels on my reading list. I need to read three of them. In how many ways can I do this. Answer in
two different ways: if order matters, and if it doesn’t. Also, answer in two different ways: books can be repeated,
and books can’t be.

Order
Matters Does Not Matter
7×7×7=7 3 7 7 7
Allowed ( )+( )+( )
1 2 3
Repetition Not 7 × 6 × 5 = 210 7
( )
Allowed 3

If repetition is allowed and order does not matter


We consider three distinct cases. We can choose
Case I: Three distinct books
7
( )
3
Case II: Two distinct books
7
( )
2
Case III: Only one distinct book
7
( )
1

Example 2.24
My college offers five elective subjects. I need to choose three subjects in the next semester. In how many ways
can I do this if:
A. The sequence in which I take the subjects is important, since I must submit the subjects in order of
priority for me.
B. The order of selection of subjects is not important.
Part A
Since the order does matter, I need to arrange three out of seven books, which can be done in
5 × 4 × 3 = 60 𝑊𝑎𝑦𝑠
Part B
Since the order does not matter, I need to choose three out of five electives, which can be done in
5
( ) = 10 𝑊𝑎𝑦𝑠
3

Example 2.25
I need to display five of seven books on a bookshelf. In how many ways can I do this if:
A. The order of the books does not matter
B. The order of the books does matter

Part A

P a g e 62 | 100
Get all the files at: https://bit.ly/azizhandouts
Aziz Manva (azizmanva@gmail.com)

Since the order does not matter, I need to choose five out of seven books, which can be done in
7 7 7×6
( )=( )= = 21 𝑊𝑎𝑦𝑠
5 2 2
Part B
Since the order does matter, I need to arrange five out of seven books, which can be done in
7 7! 5040
𝑃5 = = = 2520 𝑊𝑎𝑦𝑠
2! 2

Example 2.26 Date Friend 𝑅𝑒𝑠𝑡𝑎𝑢𝑟𝑎𝑛𝑡


My approved list of restaurants has ten names. Four of 𝑡ℎ
7 𝐽𝑢𝑙𝑦 Ilaiyaraja 𝐺𝑜𝑙𝑑𝑒𝑛 𝐷𝑒𝑙𝑖𝑐𝑎𝑐𝑖𝑒𝑠
my friends are visiting me on four distinct (fixed) days. I 8𝑡ℎ 𝐽𝑢𝑙𝑦 Ilanthirayan 𝑀𝑖𝑛𝑔′ 𝑠 𝑃𝑎𝑟𝑎𝑑𝑖𝑠𝑒
am making my schedule for those four days, where on 9𝑡ℎ 𝐽𝑢𝑙𝑦 Chinnamani 𝐹𝑎𝑠𝑡𝑒𝑠𝑡 𝐹𝑜𝑟𝑘 𝐹𝑖𝑟𝑠𝑡
each day I mention where I will go for dinner. A sample 10𝑡ℎ 𝐽𝑢𝑙𝑦 Eelampirai 𝑅𝑎𝑑ℎ𝑎𝑘𝑟𝑖𝑠ℎ𝑛𝑎
schedule is shown in the table alongside.
How many distinct schedules can I make if:
A. I can repeat restaurants. The order of visiting the restaurants matters.
B. I cannot repeat restaurants. The order of visiting the restaurants matters.
C. I cannot repeat restaurants. The order of visiting the restaurants does not matter.
D. I can repeat restaurants. The order of visiting the restaurants does not matter.

Order
Matters Does Not Matter
10 × 10 × 10 × 10 = 104 10 10 10
( )+( )+( )+( )
10
Allowed 𝑷𝒂𝒓𝒕 𝑨 1 2 3 4
𝑷𝒂𝒓𝒕 𝐃
Repetition 10
Not 10 × 9 × 8 × 7 ( )
Allowed 𝑷𝒂𝒓𝒕 𝐁 4
𝑷𝒂𝒓𝒕 𝐂
§2.1.E Review

Example 2.27
Vedika likes four cities in the US: Seattle, New York, Philadelphia and Denver. She like five cities in Europe:
Rome, Milan, London, Istanbul and Madrid. How many choices does she have if she is going to visit:
A. A single city
B. Two cities in USA
C. Two cities in Europe
D. A city in the USA, and a city in Europe
E. All the cities
Answer each question above considering
I. Order is important
II. Order is not important

P a g e 63 | 100
Get all the files at: https://bit.ly/azizhandouts
Aziz Manva (azizmanva@gmail.com)

Part A
9 𝐶ℎ𝑜𝑖𝑐𝑒𝑠
Part B
Location
4×3
4
( )
2 USA Europe
Part C
5×4
5
( ) 4 Choices 5 Choices
2
Part D
5×4×2
5×4
Part E
9!
1

True/False 2.28
I have a counting question. I know Method A will find the answer, but it is very calculation-intensive. I think a
while and find the answer to the original question using Method B. Have I also found the answer to Method A?

Yes, counting in two ways is a powerful technique.


It uses the bijection principle.

True/False 2.29
A. Imposing a restriction in a combination question will always reduce the number of ways in the final
answer.
B. Removing a restriction in a combination question will always increase the number of ways in the final
answer.

False
False

Example 2.30
For 𝑛 > 1, under what conditions does the following hold:
𝑛 𝑛
( )>( )
2 1

𝑛(𝑛 − 1) 𝑛−1
>𝑛⇒ >1⇒𝑛−1>2⇒ 𝑛>3
2 2

Example 2.31
𝑛
Is there a solution in natural numbers 𝑛 and 𝑟 to ( ) = 0, given that 0 ≤ 𝑟 ≤ 𝑛,. Explain why or why not?
𝑟

𝑛 𝑛 𝑛
min (( ) , ( ) , … ( )) = 1 ⇒ 𝑁𝑜 𝑆𝑜𝑙𝑢𝑡𝑖𝑜𝑛𝑠
0 1 𝑛

P a g e 64 | 100
Get all the files at: https://bit.ly/azizhandouts
Aziz Manva (azizmanva@gmail.com)

Example 2.32
What is the difference between an arrangement and a selection?
A. Explain using the definition
B. Explain using a practical example
C. Calculate the numerical difference in your practice example between the arrangement and the selection.

Example 2.33
A. Read the questions in these notes and elsewhere to see the keywords that distinguish questions that
require permutations from those that require combinations?
B. Does the presence of a keyword guarantee the use of permutations or combinations?
C. Write a question which has a “permutation” keyword, but the answer uses combinations.
D. Write a question which has a “combination” keyword, but the answer uses permutation.
E. What do the above parts tell you about a keyword-based approach to questions. Is it advisable?

Example 2.34
The number of ways of arranging r objects out of n is given by 𝑛𝑃𝑟, while the number of ways of selecting r
𝑛
objects out of n is given by ( ).
𝑟
𝑛
A. Write the formulas for 𝑛𝑃𝑟 and ( ). You will find them in the chapter on Permutations, and
𝑟
Combinations, respectively.
𝑛
B. Find the ratio 𝑛𝑃𝑟: ( )
𝑟
𝑛
C. Use your answer to the previous part to determine under what conditions is 𝑛𝑃𝑟 = ( )
𝑟

Investigation 2.35: Pascal’s Triangle


4
A. Write the values of ( ) for 𝑟 = 0 to 𝑟 = 4. Where is it maximum? Where is it minimum? Do you notice a
𝑟
pattern?
5 6 7
B. Repeat the above question for ( ) , ( ) and ( ) (for suitable values of 𝑟). Does the pattern still hold?
𝑟 𝑟 𝑟
C. What observations can you make about the pattern. If you start from 𝑛 = 1 and proceed, you create
𝑃𝑎𝑠𝑐𝑎𝑙’𝑠 𝑇𝑟𝑖𝑎𝑛𝑔𝑙𝑒, which is an important area of mathematics, with many interesting properties and
applications.
𝑛 𝑛
D. When is ( ) greater than ( ). You will find the previous investigation useful.
𝑟+1 𝑟
𝑛
E. What is the maximum value of ( ). Consider two cases. First, when 𝑛 is odd and second, when 𝑛 is even.
𝑟

§2.2 Counting Numbers


§2.2.A Product of Digits

Example 2.36
Find the number of four digit numbers whose product of digits is 18.

abcd = 18 =2*3*3
Product of two factors greater than 1:
(1,18) does not work
(2,9) 1129 arranged in 4C2=6 Ways
(3,6) 1136 arranged in 4C2 = 6 Ways

P a g e 65 | 100
Get all the files at: https://bit.ly/azizhandouts
Aziz Manva (azizmanva@gmail.com)

Product of three factors, each greater than 1:


(2,3,3) 1233 arranged in 4C2 = 6 Ways

§2.2.B Descending and Ascending Numbers

2.37: Descending Numbers


➢ A descending number is a number such that each successive digit is smaller than the one that comes
before it.
➢ A single digit number is also descending since there is only one digit, and it does not violate the
condition imposed.
➢ A descending number cannot repeat digits

A descending number refers to the digits of a number, and not two different numbers.
For example, the numbers 23, 72, 11 when arranged in descending order are
⇒ 72, 23, 11

However, in a descending number, the digits are arranged in descending order.

Some examples of descending numbers are:


91: 9 > 1
541: 5 > 4 > 1
870: 8 > 7 > 0

755: 7 > 5 = 5 ⇒ 𝑁𝑒𝑖𝑡ℎ𝑒𝑟 𝐴𝑠𝑐𝑒𝑛𝑑𝑖𝑛𝑔 𝑛𝑜𝑟 𝑑𝑒𝑠𝑐𝑒𝑛𝑑𝑖𝑛𝑔


99: 9 = 9 ⇒ 𝑁𝑒𝑖𝑡ℎ𝑒𝑟 𝐴𝑠𝑐𝑒𝑛𝑑𝑖𝑛𝑔 𝑛𝑜𝑟 𝑑𝑒𝑠𝑐𝑒𝑛𝑑𝑖𝑛𝑔
437: 4 > 3, 3 < 7 ⇒ 𝑁𝑒𝑖𝑡ℎ𝑒𝑟 𝐴𝑠𝑐𝑒𝑛𝑑𝑖𝑛𝑔 𝑛𝑜𝑟 𝑑𝑒𝑠𝑐𝑒𝑛𝑑𝑖𝑛𝑔

Example 2.38
A. What is the largest descending number?
B. Hence, what is the maximum number of digits that a descending number can have?

9,876,543,210 ⇒ 10 𝐷𝑖𝑔𝑖𝑡𝑠

Example 2.39
I have a five-digit descending number. Answer each question independently.
A. What is the largest digit that can be at the ten thousand’s place?
B. What is the smallest digit that can be at the ten thousand’s place?
C. What is the largest digit that can be at the unit’s place?
D. What is the smallest digit that can be at the unit’s place?

98765 ⇒ 5
43210 ⇒ 0
98765 ⇒ 5
43210 ⇒ 0

Example 2.40
What is the number of four-digit descending numbers?

P a g e 66 | 100
Get all the files at: https://bit.ly/azizhandouts
Aziz Manva (azizmanva@gmail.com)

Note that there are ten digits in the decimal system:


0−9
From above, we know that digits cannot be repeated.

Hence, for a four-digit number, we must choose four distinct digits out of ten possible digits.
Once we choose the digits, they can be arranged in descending order in only one possible way.
10
( ) = 210 𝑊𝑎𝑦𝑠
4

Example 2.41
What is the number of 𝑛 digit descending numbers?

𝑛 = 0: 𝑍𝑒𝑟𝑜 𝑁𝑢𝑚𝑏𝑒𝑟𝑠
10
1 ≤ 𝑛 ≤ 10 ⇒ ( )
𝑛
𝑛 > 10 ⇒ 𝑍𝑒𝑟𝑜 𝑁𝑢𝑚𝑏𝑒𝑟𝑠

2.42: Ascending Numbers


➢ An ascending number is a number such that each successive digit is greater than the one that comes
before it.
➢ A single digit number is also ascending since there is only one digit, and it does not violate the condition
imposed.

Some examples of ascending numbers are:


249: 2 < 4 < 9
28: 2 < 8
189: 1 < 8 < 9

Example 2.43
What is the number of ascending numbers which are two digits or less?

Consider choosing two digits out of ten, which can be done in


10
( ) = 45 𝑊𝑎𝑦𝑠
2
And then arranging them in ascending order. There are two cases:
First Digit is not Zero
This becomes a two-digit ascending number.
First Digit is Zero
This becomes a one-digit ascending number.

Number Zero
The number zero is not included in the above counting, but is nevertheless an ascending number.

Hence, final answer is:


45 + 1 = 46

Example 2.44
What is the number of ascending numbers which are three or four digits?

P a g e 67 | 100
Get all the files at: https://bit.ly/azizhandouts
Aziz Manva (azizmanva@gmail.com)

Consider choosing four digits out of ten, which can be done in


10
( ) = 210 𝑊𝑎𝑦𝑠
4
And then arranging them in ascending order. There are two cases:
First Digit is not Zero
This becomes a four-digit ascending number.
First Digit is Zero
This becomes a three-digit ascending number.

Example 2.45
What is the number of ascending numbers which are four digits or less?

For four-digit numbers, the starting digit cannot be zero. Hence, we only have nine digits to choose from:
9
𝐹𝑜𝑢𝑟 𝐷𝑖𝑔𝑖𝑡 𝑁𝑢𝑚𝑏𝑒𝑟𝑠 = ( )
4
Similarly, number of three-digit ascending numbers
9
=( )
3
And, number of two-digit ascending numbers
9
( )
2
However, for a single digit number, zero is an acceptable ascending number. Hence, the number of single digit
ascending numbers is
10
( ) = 10
1

9 9 9 10
( ) + ( ) + ( ) + ( ) = 256
4 3 2 1

§2.3 Standard Restrictions


§2.3.A Introduction
When choosing objects, we use the formula that the number of ways to choose 𝑟 objects out of 𝑛 objects is given
by
𝑛 𝑛!
𝑛 𝑐ℎ𝑜𝑜𝑠𝑒 𝑟 = ( ) =
𝑟 𝑟! (𝑛 − 𝑟)!
To this choosing, we can apply one or more restrictions that complicate the situation, and reduce our choices.

Some important things to remember in complex problems:


➢ Apply the most restrictive condition first. (This is not a rule, but it often helps in making the rest of the
counting easier).
➢ At no stage will a condition or a restriction increases the number of choices. At best, it will keep the
number of choices the same.
§2.3.B Basic Restrictions

2.46: Never Selected


The number of ways of choosing 𝑟 objects from 𝑛 objects, out of which 𝑎 objects are never to be selected is:
𝑛−𝑎
( )
𝑟

P a g e 68 | 100
Get all the files at: https://bit.ly/azizhandouts
Aziz Manva (azizmanva@gmail.com)

𝑛 𝑜𝑏𝑗𝑒𝑐𝑡𝑠
⏟ (𝑛 − 𝑎) 𝑜𝑏𝑗𝑒𝑐𝑡𝑠
→ 𝑁𝑒𝑣𝑒𝑟 𝑐ℎ𝑜𝑜𝑠𝑒 𝑎 𝑜𝑏𝑗𝑒𝑐𝑡𝑠 → ⏟
𝑆𝑡𝑎𝑟𝑡𝑖𝑛𝑔 𝑆𝑐𝑒𝑛𝑎𝑟𝑖𝑜 𝐸𝑓𝑓𝑒𝑐𝑡𝑖𝑣𝑒 𝑁𝑢𝑚𝑏𝑒𝑟
𝑜𝑓 𝑂𝑏𝑗𝑒𝑐𝑡𝑠

If some objects are never selected, then remove them from the set of objects from which the choice is to be
made.

Example 2.47
A Go Club must choose three team members out of fifteen candidates. In how many ways can the selection be
made
A. With no restrictions
B. If, out of the fifteen, a set of triplets have allergic rhinitis, and cannot go for the tournament.

Part A
15
( )
3
Part B
There are fifteen members, but three of them are not available. Hence, we only have twelve members from
which the team can be formed.
In other words, we need to choose a team of three members from twelve eligible candidates, which can be done
in
12 12! 12 × 11 × 10
( )= = = 220
3 3! 9! 6

2.48: Always Selected


The number of ways of choosing 𝑟 objects from 𝑛 objects, out of which 𝑏 objects are always to be selected is:
𝑛−𝑏
( )
𝑟−𝑏

𝑛 𝑜𝑏𝑗𝑒𝑐𝑡𝑠
⏟ (𝑛 − 𝑎) 𝑜𝑏𝑗𝑒𝑐𝑡𝑠 𝑡𝑜 𝑐ℎ𝑜𝑜𝑠𝑒 𝑓𝑟𝑜𝑚
→ 𝐴𝑙𝑤𝑎𝑦𝑠 𝑐ℎ𝑜𝑜𝑠𝑒 𝑎 𝑜𝑏𝑗𝑒𝑐𝑡𝑠 → ⏟
𝑆𝑡𝑎𝑟𝑡𝑖𝑛𝑔 𝑆𝑐𝑒𝑛𝑎𝑟𝑖𝑜 𝐸𝑓𝑓𝑒𝑐𝑡𝑖𝑣𝑒 𝑁𝑢𝑚𝑏𝑒𝑟
𝑜𝑓 𝑂𝑏𝑗𝑒𝑐𝑡𝑠
𝑁𝑜. 𝑜𝑓 𝑜𝑏𝑗𝑒𝑐𝑡𝑠 𝑡𝑜 𝑎𝑐𝑡𝑢𝑎𝑙𝑙𝑦 𝑐ℎ𝑜𝑜𝑠𝑒 𝑏𝑒𝑐𝑜𝑚𝑒𝑠 (𝑟 − 𝑎)

Example 2.49
A Scrabble group has eight enthusiasts, of which four are to be selected for a Scrabble meet. Two of the
participants are so good that the coach has already confirmed their selection. In how many ways can the team of
four be selected?

We need to select a team of four, out of which two have already been selected, leaving only a pool of six
enthusiasts for the remaining two slots.
Hence, we need to select two more participants out of the remaining six, which can be done in
8−2 6 6! 6×5
( )=( )= = = 15
4−2 2 2! 4! 2

2.50: Combining Restrictions


The number of ways to select 𝑟 objects from 𝑛 objects, out of which 𝑎 objects are never selected, and 𝑏 objects
are always selected is:
𝑛−𝑎−𝑏
( )
𝑟−𝑏

P a g e 69 | 100
Get all the files at: https://bit.ly/azizhandouts
Aziz Manva (azizmanva@gmail.com)

Example 2.51
Five players going for a camp must be chosen from ten Ice Hockey players. There are two good players, who
must be selected, and one player is down with an injury and cannot be selected. What is the number of ways in
which I can make the selection?

There are ten players. But one is injured, so we only have


10 − 1 = 9 𝐹𝑖𝑡 𝑃𝑙𝑎𝑦𝑒𝑟𝑠

Out of the fit players, the already selected players block two spots:
𝑷𝒍𝒂𝒚𝒆𝒓 𝟏 , ⏟
⏟ 𝑷𝒍𝒂𝒚𝒆𝒓 𝟐 , ⏟ 𝑷𝒍𝒂𝒚𝒆𝒓 𝟑 , ⏟
𝑷𝒍𝒂𝒚𝒆𝒓 𝟒 , ⏟
𝑷𝒍𝒂𝒚𝒆𝒓 𝟓
𝑽𝒆𝒓𝒚 𝑮𝒐𝒐𝒅 𝑷𝒍𝒂𝒚𝒆𝒓 𝑽𝒆𝒓𝒚 𝑮𝒐𝒐𝒅 𝑷𝒍𝒂𝒚𝒆𝒓 𝑺𝒍𝒐𝒕 𝑶𝒑𝒆𝒏 𝑺𝒍𝒐𝒕 𝑶𝒑𝒆𝒏 𝑺𝒍𝒐𝒕 𝑶𝒑𝒆𝒏

Hence, we must fill three slots from remaining 9 − 2 = 7 players:


9−2 7 7! 7×6×5
=( )=( )= = = 35
3 3 4! 3! 6

Shortcut
10 − 1 − 2 7
( ) = ( ) = 35
3 3
§2.3.C Always selected together
In certain scenarios, an object can only be selected when other objects in the group are also selected.
Calculating the number of ways this can be done usually requires casework.

We can have a condition where a pair gets selected together, or does not get selected at all. There may be
various real-life scenarios for this:
➢ Two players who play really well together on a team, and hence must be chosen together, or not at all.
➢ A parent who must always stay with a child.

Such kind of restrictions usually require casework.

2.52: Always Selected Together


When two or more objects are always selected together, the number of ways to do so can be calculated using
casework.

Example 2.53
I must select four students out of 9 for my team math contest. The Williams triplets do math fabulously
together, but are miserable without each other, so I will choose them all together, or not at all. In how many
ways can I make the selection?

There are two cases to consider:


Case I: You pick the triplets
Making use of the triplets “always selected” logic:
9−3 6
( )=( )=6
4−3 1
Case II: You don’t pick the triplets
Making use of the triplets “never selected” logic:
9−3 6 6 6×5
( )=( )=( )= = 15
4 4 2 2

P a g e 70 | 100
Get all the files at: https://bit.ly/azizhandouts
Aziz Manva (azizmanva@gmail.com)

And, hence the final number of ways to select is:


= 6 + 15 = 21

Example 2.54
I have to select four team members for my team math contest. The Irving twins do math fabulously together, but
are miserable without each other, so I have to choose between taking both of them, or neither of them. If there
are ten club members to choose from, in how many ways can I make the selection?

There are two cases to consider:


Case I: Twins Always Selected
There are ten people to choose from, out of which since you have already picked the twins, you are left with
10 − 2 = 8 𝑀𝑒𝑚𝑏𝑒𝑟𝑠 𝑡𝑜 𝑐ℎ𝑜𝑜𝑠𝑒 𝑓𝑟𝑜𝑚

There are four team members to be chosen, out which you have already chosen the twins, hence, you are left
with
4 − 2 = 2 𝑀𝑒𝑚𝑏𝑒𝑟𝑠 𝑡𝑜 𝑏𝑒 𝑐ℎ𝑜𝑠𝑒𝑛

Hence, we need to chose the remaining two members out of the remaining 8 members, which is:
10 − 2 8
( ) = ( ) = 28
4−2 2

Case II: Twins Never Selected


There are ten people to choose from, out of which since you not picking the twins, you are left with
10 − 2 = 8 𝑀𝑒𝑚𝑏𝑒𝑟𝑠 𝑡𝑜 𝑐ℎ𝑜𝑜𝑠𝑒 𝑓𝑟𝑜𝑚

There are four team members to be chosen, on which there are no restrictions.
Hence, the number of ways to form a team while not picking the twins is:
10 − 2 8
( ) = ( ) = 70
4 4

Adding the Cases


And, hence the final number of ways to select is:
28 + 70 = 98

Example 2.55
I have to select four team members for my team math contest. The Irving twins do math fabulously together, but
are miserable without each other, so I have to choose between taking both of them, or neither of them.
Similarly, the Jones’ twins also have the same situation. So, I need to choose both of them, or neither of them. If
there are ten club members to choose from, in how many ways can I make the selection?

We have multiple cases, and we need to handle them carefully.

Irving Twins
Yes No
Jones Twins Yes 1 15
No 15 15

Case I: You pick neither the Irving twins, nor the Jones twins
10 − 4 6 6×5
( )=( )= = 15
4 4 2

P a g e 71 | 100
Get all the files at: https://bit.ly/azizhandouts
Aziz Manva (azizmanva@gmail.com)

Case II: You pick the Irving twins, but not the Jones twins
You already picked the Irving twins, so you can’t pick them again. You decided not to pick the Jones twins, so
you can’t do that do either. Hence, the number of choices is
10 − 2 − 2 6
( ) = ( ) = 15
2 2
Case III: You pick the Jones twins, but not the Irving twins
This case, by symmetry is exactly the same as the one above, and hence the answer is also:
15
Case IV: You pick both the Jones twins, and the Irving twins
This can be done in 1 way.
And, hence the final number of ways to select is:
15 + 15 + 15 + 1 = 46

Example 2.56
A mother-toddler club with ten mothers is going for a picnic, and it has six spots booked on a bus. Two of the
mothers have twins, and the rest have a single child. Toddlers can only go along for the picnic when their
mother is present. Also, a mother going for the picnic will take all her children along. A mother occupies exactly
one spot on the bus. A toddler also occupies one spot on the bus (a different one from the one his mother
occupies). In how many ways can the members going for the picnic be selected?

Hint: A spot can go un-utilized if a single set of twins goes for the picnic. This is acceptable. In all other cases, all
spots will be utilized.

Mother Mother Mother Mother Mother Mother Mother Mother Mother Mother
Twin A Twin X Toddler Toddler Toddler Toddler Toddler Toddler Toddler Toddler
Twin B Twin Y

We do this according to casework. We can identify three distinct cases:


➢ Both sets of twins go for the picnic
➢ One set of twins goes for the picnic
➢ No twins go for the picnic
Case I: Two sets of twins go for the picnic
𝑀𝑜𝑡ℎ𝑒𝑟 1 ⏟
⏟ 𝑇𝑤𝑖𝑛 𝐴 ⏟𝑇𝑤𝑖𝑛 𝐵 ⏟
𝑀𝑜𝑡ℎ𝑒𝑟 2 ⏟
𝑇𝑤𝑖𝑛 𝑋 ⏟
𝑇𝑤𝑖𝑛 𝑌 ⇒ 𝐴𝑙𝑙 𝑇𝑖𝑐𝑘𝑒𝑡𝑠 𝑈𝑡𝑖𝑙𝑖𝑧𝑒𝑑

There is no choice here. Every eligible set of twins gets selected. So, we have only
1 𝑊𝑎𝑦

Case II: One set of twins goes for the picnic



𝑀𝑜𝑡ℎ𝑒𝑟 ⏟
𝑇𝑤𝑖𝑛 ⏟
𝑇𝑤𝑖𝑛 ⏟
𝑀𝑜𝑡ℎ𝑒𝑟 ⏟
𝑇𝑜𝑑𝑑𝑙𝑒𝑟 𝐸𝑚𝑝𝑡𝑦 𝑆𝑝𝑜𝑡 ⇒ 𝑂𝑛𝑒 𝑇𝑖𝑐𝑘𝑒𝑡 𝑈𝑛 − 𝑈𝑡𝑖𝑙𝑖𝑧𝑒𝑑

⏟ ⏟
𝑻𝒉𝒓𝒆𝒆 𝑺𝒑𝒐𝒕𝒔 𝑻𝒘𝒐 𝑺𝒑𝒐𝒕𝒔

Choose the mother with the twins, which can be done in


2 𝑊𝑎𝑦𝑠
and the mother with one child, which can be done in:
8 𝑤𝑎𝑦𝑠
Total Ways are
2 × 8 = 16

P a g e 72 | 100
Get all the files at: https://bit.ly/azizhandouts
Aziz Manva (azizmanva@gmail.com)

Case III: None of the twins go for the picnic



𝑀𝑜𝑡ℎ𝑒𝑟 ⏟
𝑇𝑜𝑑𝑑𝑙𝑒𝑟 ⏟
𝑀𝑜𝑡ℎ𝑒𝑟 ⏟
𝑇𝑜𝑑𝑑𝑙𝑒𝑟 ⏟
𝑀𝑜𝑡ℎ𝑒𝑟 ⏟
𝑇𝑜𝑑𝑑𝑙𝑒𝑟 ⇒ 𝐴𝑙𝑙 𝑇𝑖𝑐𝑘𝑒𝑡𝑠 𝑈𝑡𝑖𝑙𝑖𝑧𝑒𝑑
⏟ ⏟ ⏟
𝑻𝒘𝒐 𝑺𝒑𝒐𝒕𝒔 𝑻𝒘𝒐 𝑺𝒑𝒐𝒕𝒔 𝑻𝒘𝒐 𝑺𝒑𝒐𝒕𝒔

Since the twins don’t go, we are left with


10 − 2 = 8 𝑀𝑜𝑡ℎ𝑒𝑟𝑠
From these 8 mothers, we have to choose 3 mothers (toddlers automatically get chosen), which can be done in:
8
( ) = 56
3
Total Number of Ways
56 + 16 + 1 = 73
§2.3.D At Least One Selected

Example 2.57 (Calculator Allowed)


A school movie club is selecting four movies to showcase at its film festival from ten movies proposed by the
ninth grade, and ten movies proposed by the tenth grade. To not hurt anyone feelings at least one movie must
be selected from the tenth-grade proposals, and at least one movie must be selected from the ninth-grade
proposals. In how many ways can the selections be made?

There are two methods to do this. The first one is direct counting. Consider the number of tenth grade movies
that can be chosen, without any restrictions:


0 , 1,2,3, ⏟
4
𝑁𝑜𝑡 𝑁𝑜𝑡
𝑃𝑜𝑠𝑠𝑖𝑏𝑙𝑒 𝑃𝑜𝑠𝑠𝑖𝑏𝑙𝑒
Hence, we are left with three cases to consider:

Case I: Choose 1 10th Grade Movie (and hence 3 9th Grade Movies)
10 10
( ) × ( ) = 10 × 120 = 1200
1 3

Case II: Choose 3 10th Grade Movies (and hence 1 9th Grade Movie)
By symmetry, this is also
10 10
( ) × ( ) = 10 × 120 = 1200
1 3
Case III: Choose 2 Movies of each type
10 10 10 2 10 × 9 2
( ) × ( ) = [( )] = [ ] = 452 = 2025
2 2 2 2

And hence the final answer is:


1200 + 1200 + 2025 = 4425

2.58: At least one selected


Consider 𝑛 objects of Type 1, and 𝑚 objects of Type 2, resulting in a total of 𝑛 + 𝑚 objects.

The number of ways of selecting 𝑟 objects, such that at least one object of each type is selected:
𝑛+𝑚 𝑚 𝑛
( ) − (⏟) − (⏟)
⏟ 𝑟 𝑟 𝑟
𝑻𝒐𝒕𝒂𝒍 𝑨𝒍𝒍 𝒐𝒇 𝑨𝒍𝒍 𝒐𝒇
𝑻𝒚𝒑𝒆 𝟏 𝑻𝒚𝒑𝒆 𝟐

P a g e 73 | 100
Get all the files at: https://bit.ly/azizhandouts
Aziz Manva (azizmanva@gmail.com)

If we want to select at least one of each type from a pool of objects, we can find the ways to make the selection
using complementary counting.
𝑊𝑎𝑦𝑠𝐴𝑡 𝐿𝑒𝑎𝑠𝑡 𝑂𝑛𝑒 = 𝑊𝑎𝑦𝑠𝑇𝑜𝑡𝑎𝑙 − 𝑊𝑎𝑦𝑠𝑂𝑛𝑙𝑦 𝑠𝑖𝑛𝑔𝑙𝑒 𝑡𝑦𝑝𝑒 𝑖𝑠 𝑠𝑒𝑙𝑒𝑐𝑡𝑒𝑑

Example 2.59 (Calculator Allowed)


A school movie club is selecting four movies to showcase at its film festival from ten movies proposed by the
ninth grade, and ten movies proposed by the tenth grade. To not hurt anyone feelings at least one movie must
be selected from the tenth-grade proposals, and at least one movie must be selected from the ninth-grade
proposals. In how many ways can the selections be made?

If there were no restrictions, then we needed to choose 4 movies out of 20 movies available, which can be done
in:
20 20 × 19 × 18 × 17
( )= = 4845
4 4!
But we cannot choose all 4 movies to be 10th grade, or all four movies to be 9th grade, which can be done in:
10
( ) × 2 = 210 × 2 = 420
4
And hence, the final answer is:
4845 − 420 = 4425

Example 2.60 (Calculator Allowed)


In the question above, both of the methods took similar time and calculations. Which of the methods is
preferable if we needed to choose six movies instead of four, and other conditions remained the same? Do the
calculations and confirm.

20 10 10 20 10
( )−( )−( ) =( )−( )×2
6 6 6 6 6

10 10 10 10 10 10 10 10 10 10
( )( ) + ( )( ) + ( )( ) + ( )( ) + ( )( )
1 5 2 4 3 3 4 2 5 1

Example 2.61
A school must send three of its best programmers for a mixed-gender programming competition, which
specifies that at least one member of the team must be male, and one must be female. It must select its team
from a pool of five boys and four girls. In how many ways can the selection be made?

We can do this using complementary counting.


➢ Count the number of ways without restrictions
➢ And subtract both 𝑎𝑙𝑙 − 𝑏𝑜𝑦 and 𝑎𝑙𝑙 − 𝑔𝑖𝑟𝑙 teams from the total
Set up the Logic
Without Restrictions
If there are no restrictions, the selection can be made in:
9
( ) = 84 𝑊𝑎𝑦𝑠
3
Ways to Select All Boys
An all-boy team can be selected in
5
( ) = 10 𝑊𝑎𝑦𝑠
3
Ways to Select All Girls

P a g e 74 | 100
Get all the files at: https://bit.ly/azizhandouts
Aziz Manva (azizmanva@gmail.com)

An all-girl team can be selected in


4
( ) = 4 𝑊𝑎𝑦𝑠
3
Complementary Counting
Total Number of Ways
= 84
⏟ − 10
⏟ − ⏟
4 = 70
𝑁𝑜 𝐴𝑙𝑙 𝐵𝑜𝑦𝑠 𝐴𝑙𝑙 𝐺𝑖𝑟𝑙𝑠
𝑅𝑒𝑠𝑡𝑟𝑖𝑐𝑡𝑖𝑜𝑛𝑠

(Alternate Solution) A school must send three of its best programmers for a mixed-gender programming
competition, which specifies that at least one member of the team must be male, and one must be female. It
must select its team from a pool of five boys and four girls. In how many ways can the selection be made?

There are only two cases possible:


➢ Two boys and one girl
➢ Two girls and one boy

5 4 5 4
( ) ( ) + ( ) ( ) = 10 × 4 + 5 × 6 = 40 + 30 = 70
⏟2 1 ⏟1 2
𝑻𝒘𝒐 𝑩𝒐𝒚𝒔 𝑻𝒘𝒐 𝑮𝒊𝒓𝒍𝒔
𝒂𝒏𝒅 𝒐𝒏𝒆 𝑮𝒊𝒓𝒍 𝒂𝒏𝒅 𝑶𝒏𝒆 𝑩𝒐𝒚

2.62: At least one of a particular type selected


I have 𝑛 objects of a particular type, and 𝑚 objects of another type, and I want to choose 𝑟 (𝑟 < 𝑛, 𝑟 < 𝑚)
objects from all of these objects such that at least one of each type is selected then the number of ways to do so
is:
𝑛+𝑚 𝑛 𝑚
( )−( )−( )
𝑟 𝑟 𝑟

Example 2.63
A syllabus committee consisting of three people must be formed in a college. There are four Sociology
professors and three Anthropology professors eligible for the committee. In how many ways can this be done if
the committee consists of at least one professor from each field.

Strategy
Counting all the ways in which it is possible will result in a lot of nasty casework.
Instead, count the total number of ways, and subtract the number of ways where the condition of one professor
from each field is not met.
Total Ways
I want to select three professors from seven professors, which can be done in
7 7! 7×6×5
=( )= = = 35
3 4! 3! 6
Single Field Committees
A committee consisting of only Anthropology professors can be done in only
3
=( )=1
3
A committee consisting of only Sociology professors can be done in:
4 4
=( )=( )=4
3 1
Complementary Counting

P a g e 75 | 100
Get all the files at: https://bit.ly/azizhandouts
Aziz Manva (azizmanva@gmail.com)

The number of ways we want is


35
⏟ − ⏟
1 − ⏟
4 = 30
𝑻𝒐𝒕𝒂𝒍 𝑾𝒂𝒚𝒔 𝑨𝒍𝒍 𝑨𝒏𝒕𝒉𝒓𝒐𝒑𝒐𝒍𝒐𝒈𝒚 𝑨𝒍𝒍 𝑺𝒐𝒄𝒊𝒐𝒍𝒐𝒈𝒚

Example 2.64
Out of 6 ruling and 5 opposition party members, 4 are to be selected for a delegation. In how many ways can
this be done so as to include at least one opposition member. (JMET 2011/69)

11 6
( )− ( ) = 330 − 15 = 315
⏟4 ⏟4
𝑇𝑜𝑡𝑎𝑙 𝑁𝑜 𝑂𝑝𝑝𝑜𝑠𝑖𝑡𝑖𝑜𝑛
𝑤𝑎𝑦𝑠 𝑀𝑒𝑚𝑏𝑒𝑟

E. Always in a particular order

Example 2.65
Find the number of ways in which four people (𝑃, 𝑄, 𝑅, 𝑆) can be seated such that 𝑅 is always to the right of 𝑃.

Enumeration
We list out the 24 possible ways in which four people can be seated. And we highlight and count the ways in
which R is to the right of P, which gives us:
6 + 3 + 3 = 12 𝑊𝑎𝑦𝑠

Start with P Start with Q Start with R Start with S


PQRS PRQS PSQR QPRS QRPS QSPR RPQS RQPS RSPQ SPQR SQPR SRPQ
PQSR PRSQ PSRQ QPSR QRSP QSRP RPSQ RQSP RSQP SPRQ SQRP SRQP
6 3 0 3

Constructive Counting
We can choose the positions of Q and S in
4
( ) 𝑤𝑎𝑦𝑠
2
And arrange them among themselves in
2! 𝑤𝑎𝑦𝑠
Finally, R and P can only be arranged in 1 way.
Hence, the final answer is:
4
( ) × 2 = 12
2
Symmetry
The number of ways that four people can be arranged without restrictions
4! = 24
But, note that, for every arrangement where R is to the right of P, there is exactly one arrangement where P is to
the right of R (obtained simply by interchanging the positions of P and R).

Hence, the number of required ways is simply:


4!
= 12
2

Example 2.66
A. In how many ways can the letters in 𝑐𝑎𝑠𝑒 be arranged so that the vowels are all in alphabetic order.
B. In how many ways can the letters in 𝑓𝑎𝑐𝑒𝑡𝑖𝑜𝑢𝑠 be arranged so that the vowels are all in alphabetic

P a g e 76 | 100
Get all the files at: https://bit.ly/azizhandouts
Aziz Manva (azizmanva@gmail.com)

order.

Part B
This is the same question as part A.
12
Part C
We can choose the positions of {𝑓, 𝑐, 𝑡, 𝑠} in
9 9!
( )= 𝑤𝑎𝑦𝑠
4 4! 5!
Then the vowels can be arranged in only 1 way.
Finally, {𝑓, 𝑐, 𝑡, 𝑠} can be arranged among themselves in
4! 𝑤𝑎𝑦𝑠
Hence, the final answer is:
9! 9!
× 4! =
4! 5! 5!

§2.3.F Addition Rule (Casework)

Example 2.67: Combinations with the Addition Rule


Bill is sent to a donut shop to purchase exactly six donuts. If the shop has four kinds of donuts and Bill is to get
at least one of each kind, how many combinations will satisfy Bill's order requirements? (MathCounts 2008
State Target)

Consider that the donuts are of Type


𝐴, 𝐵, 𝐶, 𝐷
We need to purchase at least one donut of each type. Hence, even though we are purchasing six donuts, we do
have a choice in the first four donuts. They must be, in some order:
𝐴, 𝐵, 𝐶, 𝐷

So, the choice only lies in the last two donuts to be purchased.
𝐴, 𝐵, 𝐶, 𝐷, ? ?, ??

Now, we can break the choice of purchasing the last two donuts in two ways:

Case I: Both donuts are the same


Here, we can only choose the type of donut, which can be done in:
4 𝑊𝑎𝑦𝑠

𝐴, 𝐵, 𝐶, 𝐷, 𝐴, 𝐴
𝐴, 𝐵, 𝐶, 𝐷, 𝐵, 𝐵
𝐴, 𝐵, 𝐶, 𝐷, 𝐶, 𝐶
𝐴, 𝐵, 𝐶, 𝐷, 𝐷, 𝐷

Case II: Both donuts are different


Here, we need to choose two donuts out of 4, which can be done in:
4 4×3
( )= =6
2 2

𝑇𝑜𝑡𝑎𝑙 𝑊𝑎𝑦𝑠 = 4 + 6 = 10

P a g e 77 | 100
Get all the files at: https://bit.ly/azizhandouts
Aziz Manva (azizmanva@gmail.com)

(Alternate Solution)
The question above can be solved using Diophantine Equations, which we do next.

Let’s say the number of donuts purchases of different varieties is:


𝑑1 = 𝑁𝑜. 𝑜𝑓 𝐷𝑜𝑛𝑢𝑡𝑠 𝑜𝑓 𝐹𝑖𝑟𝑠𝑡 𝑇𝑦𝑝𝑒 𝑃𝑢𝑟𝑐ℎ𝑎𝑠𝑒𝑑
𝑑2 = 𝑁𝑜. 𝑜𝑓 𝐷𝑜𝑛𝑢𝑡𝑠 𝑜𝑓 𝑆𝑒𝑐𝑜𝑛𝑑 𝑇𝑦𝑝𝑒 𝑃𝑢𝑟𝑐ℎ𝑎𝑠𝑒𝑑
𝑑3 = 𝑁𝑜. 𝑜𝑓 𝐷𝑜𝑛𝑢𝑡𝑠 𝑜𝑓 𝑇ℎ𝑖𝑟𝑑 𝑇𝑦𝑝𝑒 𝑃𝑢𝑟𝑐ℎ𝑎𝑠𝑒𝑑
𝑑4 = 𝑁𝑜. 𝑜𝑓 𝐷𝑜𝑛𝑢𝑡𝑠 𝑜𝑓 𝐹𝑜𝑢𝑟 𝑇𝑦𝑝𝑒 𝑃𝑢𝑟𝑐ℎ𝑎𝑠𝑒𝑑

We need to purchase six donuts, and hence:


𝑑1 + 𝑑2 + 𝑑3 + 𝑑4 = 6, 𝑑𝑛 > 0, 𝑑𝑛 ∈ ℤ
But, we need to purchase one of each kind. So, we do not have any choice there. What we are left with is
𝑑′1 + 𝑑′2 + 𝑑′3 + 𝑑′4 = 2, 𝑑𝑛′ ≥ 0, , 𝑑𝑛 ∈ ℤ
There are two ways we can do the allocation:
➢ Assign the value of two to a single variable. This can be done in four ways.
2+0+0+0=2
0+2+0+0=2
0+0+2+0=2
0+0+0+2=2
➢ Assign a value of one each to two of the four variables. This amounts to choosing two out of four
objects, which can be done in
4
( ) = 6 𝑊𝑎𝑦𝑠
2

𝑇𝑜𝑡𝑎𝑙 𝑊𝑎𝑦𝑠 = 4 + 6 = 10

Example 2.68
A bakery sells three kinds of rolls. How many different combinations of rolls could Jack purchase if he buys a
total of six rolls and includes at least one of each kind? (MathCounts 2010 National Countdown)

Let the three types be A, B and C. Then, Jack must purchase one of each, and has three empty spots that could be
filled with any rolls of his choice:
𝐴𝐵𝐶 𝑋𝑋𝑋
We do this using casework, and focus only on the last rolls, since that is where we have a choice
Case I: All three of a different type
𝐴𝐵𝐶 ⇒ 1 𝑊𝑎𝑦
Case II: All three of the same type
𝐴𝐴𝐴, 𝐵𝐵𝐵, 𝐶𝐶𝐶 ⇒ 3 𝑊𝑎𝑦𝑠
Case III: Two of a type, and the third of a different type
𝐴𝐴𝐵, 𝐴𝐴𝐶, 𝐵𝐵𝐴, 𝐵𝐵𝐶, 𝐶𝐶𝐴, 𝐶𝐶𝐵
Use the multiplication principle here:
3 𝐶ℎ𝑜𝑖𝑐𝑒𝑠 × ⏟
⏟ 2 𝐶ℎ𝑜𝑖𝑐𝑒𝑠 = 3 × 2 = 6
𝑆𝑎𝑚𝑒 𝑇𝑦𝑝𝑒 𝐷𝑖𝑓𝑓𝑒𝑟𝑒𝑛𝑡 𝑇𝑦𝑝𝑒
Total
= 1 + 3 + 6 = 10

Example 2.69
Let 𝑆 = {1,2,3, . . . ,9}. For 𝑘 = 1,2, . . .5, let 𝑁𝑘 be the number of subsets of 𝑆, each containing 5 elements out of
which exactly 𝑘 are odd. Then 𝑁1 + 𝑁2 + 𝑁3 + 𝑁4 + 𝑁5 = (JEE Adv. 2017)

P a g e 78 | 100
Get all the files at: https://bit.ly/azizhandouts
Aziz Manva (azizmanva@gmail.com)

4 5
0 𝐸𝑣𝑒𝑛, 5 𝑂𝑑𝑑 ⇒ ( ) ( ) = 1
0 5
4 5
1 𝐸𝑣𝑒𝑛, 4 𝑂𝑑𝑑 ⇒ ( ) ( ) = 20
1 4
4 5
2 𝐸𝑣𝑒𝑛, 3 𝑂𝑑𝑑 ⇒ ( ) ( ) = 60
2 3
4 5
3 𝐸𝑣𝑒𝑛, 2 𝑂𝑑𝑑 ⇒ ( ) ( ) = 40
3 2
4 5
4 𝐸𝑣𝑒𝑛, 1 𝑂𝑑𝑑 ⇒ ( ) ( ) = 5
4 1
Total
= 1 + 20 + 60 + 40 + 5 = 126
§2.3.G Multiplication Rule

Example 2.70
A class has 12 boys and 9 girls. In how many ways can you pick two monitors, such that each is of a different
gender.

Without using Combinations


We have done this question without combinations (by using the multiplication rule) as follows:
12
⏟ × ⏟
9 = 108
𝑪𝒉𝒐𝒊𝒄𝒆𝒔 𝒇𝒐𝒓 𝒂 𝑪𝒉𝒐𝒊𝒄𝒆𝒔 𝒇𝒐𝒓 𝒂
𝒎𝒂𝒍𝒆 𝒎𝒐𝒏𝒊𝒕𝒐𝒓 𝒇𝒆𝒎𝒂𝒍𝒆 𝒎𝒐𝒏𝒊𝒕𝒐𝒓

With Combinations
The number of choices for the male monitor and the female monitor can be written in the language of
combinations, without changing the final answer:
12 9
( ) × ( ) = 12 × 9 = 108
⏟1 ⏟1
𝑪𝒉𝒐𝒊𝒄𝒆𝒔 𝒇𝒐𝒓 𝒂 𝑪𝒉𝒐𝒊𝒄𝒆𝒔 𝒇𝒐𝒓 𝒂
𝒎𝒂𝒍𝒆 𝒎𝒐𝒏𝒊𝒕𝒐𝒓 𝒇𝒆𝒎𝒂𝒍𝒆 𝒎𝒐𝒏𝒊𝒕𝒐𝒓

Example 2.71
Out of 6 ruling and 5 opposition party members, 4 are to be selected for a delegation. In how many ways can
this be done so as to include exactly one ruling party member. (JMET 2011/68)

6 5
( ) × ( ) = 6 × 10 = 60
⏟1 ⏟3
𝑅𝑢𝑙𝑖𝑛𝑔 𝑂𝑝𝑝𝑜𝑠𝑖𝑡𝑖𝑜𝑛

Example 2.72
I have five books and seven movies. I am planning to watch two movies and read two books. In how many ways
can I do this?

5 7 5×4 7×6
( )×( ) = × = 10 × 21 = 210
2 2 2 2

Example 2.73
Two subsets of the set 𝑆 = {𝑎, 𝑏, 𝑐, 𝑑, 𝑒} are to be chosen so that their union is 𝑆 and their intersection contains
exactly two elements. In how many ways can this be done, assuming that the order in which the subsets are
chosen does not matter? (AMC 10B 2008/23)

P a g e 79 | 100
Get all the files at: https://bit.ly/azizhandouts
Aziz Manva (azizmanva@gmail.com)

Let the sets be 𝑆1 and 𝑆2 . We can choose the 10 × 8 = 80 𝑊𝑎𝑦𝑠


elements of 𝑆1 ∩ 𝑆2 (the intersection of the two But note that since the order of the subsets does not
sets) in matter, the following two are the same:
5 5×4 𝑆1 = {𝑎, 𝑏, 𝑐, 𝑑, 𝑒}, 𝑆2 = {𝑑, 𝑒}
( )= = 10 𝑊𝑎𝑦𝑠
2 2 𝑆2 = {𝑎, 𝑏, 𝑐, 𝑑, 𝑒}, 𝑆1 = {𝑑, 𝑒}
The remaining three elements can be assigned to And hence, to account for the overcounting, we
one of the two sets in divide by 2:
23 = 8 𝑊𝑎𝑦𝑠 80
The total number of ways in which this can be done, = 40 𝑊𝑎𝑦𝑠
2
by the multiplication principle is:

2.74: Selecting from distinct types


Sometimes, when selecting we may have multiple groups, and specific conditions attached to each group.
If specific seats are reserved for specific groups, then the question becomes:
Select 𝑥 objects from group 𝑦
𝑦
( ) 𝑊𝑎𝑦𝑠
𝑥
Select 𝑝 objects from group 𝑞
𝑞
(𝑝) 𝑊𝑎𝑦𝑠

Combine the two using the multiplication principle.


𝑦 𝑞
( ) × (𝑝)
𝑥

Example 2.75
A college has seven Maths professors, three Statistics professors, two Computer Science (CS) professors. What
is the number of ways in which a committee can be formed consisting of:
A. six professors.
B. six professors with an equal number of professors of each background.
Part A
We have
7 + 3 + 2 = 12 𝑃𝑟𝑜𝑓𝑒𝑠𝑠𝑜𝑟𝑠
out of which we need to choose six professors for the committee.

This can be done in:


12 12! 12 × 11 × 10 × 9 × 8 × 7
𝐶6 = = = 11 × 2 × 6 × 7 = 924
6! 6! 6×5×4×3×2

Part B
Since, we need to choose an equal number of each background, we need
2 + 2 + 2 = 6 𝑃𝑟𝑜𝑓𝑒𝑠𝑠𝑜𝑟𝑠

And, the choice of professors is independent of each other. Hence, by the multiplication rule:
7 3 2 7! 3! 2!
⏟ 𝐶2 × ⏟ 𝐶2 × ⏟ 𝐶2 = × × = (7 × 3) × 3 × 1 = 63
2! 5! 2! 1! 2! 0!
𝑀𝑎𝑡ℎ𝑠 𝑆𝑡𝑎𝑡𝑠 𝐶𝑆

Example 2.76

P a g e 80 | 100
Get all the files at: https://bit.ly/azizhandouts
Aziz Manva (azizmanva@gmail.com)

Amit has 11 friends: 7 boys and 4 girls. In how many ways, can Amit invite them, if there have to be exactly 4
boys in the invitees? (JMET 2011/81)

Ways of inviting exactly 4 boys out of 7


7
= ( ) = 35
4
For each girl, we have a choice of inviting or not inviting, which us total choices as:
24 = 16
Total Ways
= 16 × 35 = 560

§2.3.H Distinguishability
When objects cannot be identified, the number of choices will reduce.
If objects can be identified, the number of choices will increase.

In both cases above, in some special cases, the number of choices can remain same.

Example 2.77
I have four identical red balls, and three identical black balls. In how many ways can I place two red balls and
two black balls in a decorative vase, if the order in which I place the balls:
A. Is not important
B. Is important

Since the balls are identical having more balls than the required number does not increase the number of ways.
Part A
Since I cannot distinguish between the balls, there is only one way to do it.

Part B
If order is important, then I need to choose the positions of the red balls among the four balls.
⏟ ⏟ ⏟ ⏟
𝐵𝑎𝑙𝑙 1 𝐵𝑎𝑙𝑙 2 𝐵𝑎𝑙𝑙 3 𝐵𝑎𝑙𝑙 4
And hence the number of ways in which I can place the balls is:
4
( )=6
2

Example 2.78
I have four red balls, numbered one to four, and three black balls, numbered one to three. In how many ways can
I place two red balls and two black balls in a decorative vase, if the order in which I place the balls:
A. Is not important
B. Is important

Strategy
Because the balls are numbered, therefore they are distinct or identifiable. Hence, we have multiple ways to
choose among them.

Part A
Here, we only need to select the balls, since the order of placing them in the vase is not important. However, we
need to break into parts.

P a g e 81 | 100
Get all the files at: https://bit.ly/azizhandouts
Aziz Manva (azizmanva@gmail.com)

Red Balls
Out of the four red balls, we want to select two, which can be done in
4 4!
( )= = 6 𝑊𝑎𝑦𝑠
2 2! 2!
Black Balls
Out of the three black balls, we want to select one, which can be done in
3 3!
( )= = 3 𝑊𝑎𝑦𝑠
2 1! 2!
Multiplication Principle
By the multiplication principle, since choosing the black balls is independent of the choosing the red balls, the
total number of ways of choosing the black and red balls:
4 3 4! 3!
( ) × ( ) = × = 6 × 3 = 18
⏟2 ⏟ 2 2! 2! 1! 2!
𝑺𝒆𝒍𝒆𝒄𝒕𝒊𝒏𝒈 𝑺𝒆𝒍𝒆𝒄𝒕𝒊𝒏𝒈
𝑹𝒆𝒅 𝑩𝒂𝒍𝒍𝒔 𝑩𝒍𝒂𝒄𝒌 𝑩𝒂𝒍𝒍𝒔

Part B
Break the process into two parts:
➢ Select the balls = 18 𝑊𝑎𝑦𝑠
➢ Arrange the balls = 4! = 24 𝑊𝑎𝑦𝑠

Then, the total ways to do the entire process is:


18
⏟ × 24
⏟ = 432
𝑺𝒆𝒍𝒆𝒄𝒕 𝑨𝒓𝒓𝒂𝒏𝒈𝒆
𝑭𝒐𝒖𝒓 𝑩𝒂𝒍𝒍𝒔 𝑭𝒐𝒖𝒓 𝑩𝒂𝒍𝒍𝒔

§2.3.I Complementary Counting

Example 2.79
A team needs to choose the colours of its uniform, which consists of a jersey and shorts. The coach has chosen
turquoise as one of the colors, and the players need to choose the second color out of fifteen colours (of which
turquoise is one colour). The color of the jersey can be different or same as the color of the shorts. In how many
ways can this selection be done?

Number of Ways of Choosing the second color is:


15
( ) = 15
1
We also need to choose which color is the jersey, and which color are the shorts.
𝐽𝑒𝑟𝑠𝑒𝑦
⏟ + 𝑆ℎ𝑜𝑟𝑡𝑠
⏟ , 𝑆ℎ𝑜𝑟𝑡𝑠
⏟ + 𝐽𝑒𝑟𝑠𝑒𝑦
⏟ ⇒ 2 𝑊𝑎𝑦𝑠
𝟏𝒔𝒕 𝑪𝒐𝒍𝒐𝒖𝒓 𝟐𝒏𝒅 𝑪𝒐𝒍𝒐𝒖𝒓 𝟏𝒔𝒕 𝑪𝒐𝒍𝒐𝒖𝒓 𝟐𝒏𝒅 𝑪𝒐𝒍𝒐𝒖𝒓
Total Ways
15 × 2 = 30
However, turquoise is one of the 15 colours. So, if you pick turquoise as one of the colors, then you have only
one choice, not two.
Hence, the final answer is
30 − 1 = 29

Example 2.80
I have eight boxes in a row. In how many ways can I place two identical red balls, one per box, in the boxes so
that no two adjacent boxes have red balls.

If there are no restrictions, then the number of ways to place two red balls, one per box in eight boxes is

P a g e 82 | 100
Get all the files at: https://bit.ly/azizhandouts
Aziz Manva (azizmanva@gmail.com)

8 8×7
( )= = 28
2 2

Number the boxes 1 to 8. The two red balls can go in box numbers given below if they are adjacent:
{12, 23, 34, 45, 56, 67, 78} ⇒ 7 𝑊𝑎𝑦𝑠

The final answer is:


28 − 7 = 21 𝑊𝑎𝑦𝑠

Example 2.81
I have eight boxes in a row. In how many ways can I place three identical red balls, one per box, in eight boxes so
that no two adjacent boxes have red balls.

Total Ways
We can place three identical balls in eight boxes in a row in:
8
( ) = 56 𝑤𝑎𝑦𝑠
3
Balls Together
Two Balls Together
Number the boxes 1 to 8. The number of ways to put two red balls in eight boxes so that they are adjacent is:
{12, 23, 34, 45, 56, 67, 78} ⇒ 7 𝑊𝑎𝑦𝑠
For each arrangement above, the third ball can go in any of the remaining six positions:
6 𝑊𝑎𝑦𝑠

The final number of ways to have at least two red balls together is:
7 × 6 = 42 𝑊𝑎𝑦𝑠

But the above arrangement counts each way that three red balls can be together twice:
12 𝑤𝑖𝑡ℎ 𝑡ℎ𝑒 𝑡ℎ𝑖𝑟𝑑 𝑏𝑎𝑙𝑙 𝑖𝑛 𝑡ℎ𝑒 𝑡ℎ𝑖𝑟𝑑 𝑝𝑜𝑠𝑖𝑡𝑖𝑜𝑛: 12𝟑
23 𝑤𝑖𝑡ℎ 𝑡ℎ𝑒 𝑡ℎ𝑖𝑟𝑑 𝑏𝑎𝑙𝑙 𝑖𝑛 𝑡ℎ𝑒 𝑓𝑖𝑟𝑠𝑡 𝑝𝑜𝑠𝑖𝑡𝑖𝑜𝑛: 𝟏23

Three Balls Together


123, 234, 345, 456, 567, 678 ⇒ 6 𝑊𝑎𝑦𝑠
Subtract the number of ways three balls can be together from our count of two balls together.
42 − 6 = 36
Complementary Counting
The number of valid ways is
56 − 36 = 20

§2.3.J Forming Teams


If order is not important, then we are selecting, not arranging.

Example 2.82: Making Opposing Teams

𝐴, 𝐵, 𝐶, 𝐷

4
( )=6
2

P a g e 83 | 100
Get all the files at: https://bit.ly/azizhandouts
Aziz Manva (azizmanva@gmail.com)

𝐴𝐵 𝑣𝑠 𝐶𝐷
𝐴𝐶 𝑣𝑠 𝐵𝐷
𝐴𝐷 𝑣𝑠 𝐵𝐶
𝐵𝐶 𝑣𝑠 𝐴𝐷
𝐵𝐷 𝑣𝑠 𝐴𝐶
𝐶𝐷 𝑣𝑠 𝐴𝐵

Example 2.83: Making Opposing Teams


I have ten basketball players, whom I wish to get to practice in teams of five versus five. In how many ways can I
do this if:
A. I have a Blue team and a Red Team
B. I just want two teams, without distinguishing between the teams.

Part A
We need to break the players into a Blue team and a Red Team. Suppose we select players for the Blue Team.
Then, the players for the Red Team are automatically decided.

𝑏 × ⏟
1 = ⏟ 𝑏
𝑪𝒉𝒐𝒊𝒄𝒆𝒔 𝒇𝒐𝒓 𝑪𝒉𝒐𝒊𝒄𝒆𝒔 𝒇𝒐𝒓 𝑻𝒐𝒕𝒂𝒍
𝑩𝒍𝒖𝒆 𝑻𝒆𝒂𝒎 𝑹𝒆𝒅 𝑻𝒆𝒂𝒎 𝑪𝒉𝒐𝒊𝒄𝒆𝒔
The number of ways to pick five players out of ten for the Blue Team is
10 10! 10 × 9 × 8 × 7 × 6
( )= = = 252 𝑤𝑎𝑦𝑠
5 5! 5! 2×3×4×5
Part B
Suppose that the players are:
𝐴, 𝐵, 𝐶, 𝐷, 𝐸, 𝐹, 𝐺, 𝐻, 𝐼, 𝐽
Here, we want to divide ten players into two teams of five each, but the teams are not distinguishable. Hence,
start by assuming, that the teams are called Blue, and Red, in which case, we can divide the players in:
10 10! 10 × 9 × 8 × 7 × 6
( )= = = 252 𝑤𝑎𝑦𝑠
5 5! 5! 2×3×4×5
But, since the teams are indistinguishable, the following arrangements are the same.
{𝐴, 𝐵, 𝐶, 𝐷, 𝐸} , ⏟
(⏟ {𝐹, 𝐺, 𝐻, 𝐼, 𝐽}) = (⏟
{𝐴, 𝐵, 𝐶, 𝐷, 𝐸} , ⏟
{𝐹, 𝐺, 𝐻, 𝐼, 𝐽})
𝑇𝑒𝑎𝑚 1 𝑇𝑒𝑎𝑚 2 𝑇𝑒𝑎𝑚 2 𝑇𝑒𝑎𝑚 1
And hence, we need to divide the answer that we got above by the number of ways that two teams can be
arranged among themselves:
252
= 126 𝑊𝑎𝑦𝑠
2

Example 2.84: Opposing Teams with some people left out


I have eleven basketball players, whom I wish to get to practice in a Blue team (with five players) and a Red
Team (with five players). One player will not in any team, and will be benched. In how many ways can I do this?
Selecting Benched Player First
Benched Player
The benched player is one out of 11 players, and he can be chosen in
11 𝑊𝑎𝑦𝑠
Selecting Blue and Red Teams
Having chosen the benched player, the question reduces to dividing ten players into a Blue Team and a Red
Team and that can be done (which we have seen above) in:
10
( ) 𝑊𝑎𝑦𝑠
5

P a g e 84 | 100
Get all the files at: https://bit.ly/azizhandouts
Aziz Manva (azizmanva@gmail.com)

Multiplication Rule
By the multiplication rule, the final number of ways is:
10
11 × ( ) = 11 × 252 = 2,772
5
Selecting Blue Team First
Imagine that we go about our selection team by team:
➢ First the Blue Team
➢ Then the Red Team
➢ Finally, the benched player
Blue Team
There are 11 players, of which we want to choose five, which can be done in:
11 11!
( )=
⏟5 6! 5!
𝐵𝑙𝑢𝑒
𝑇𝑒𝑎𝑚
Red Team
Once we have chosen the five players for the Blue Team, we are only left with six players. Of these players, we
want to choose five players, which can be done in
6 6!
( ) =
⏟5 1! 5!
𝑅𝑒𝑑
𝑇𝑒𝑎𝑚
Benched Player
Once we have chosen the Blue and the Red Team, we don’t have a choice for the benched player. He must be the
player remaining, and hence, he can only be chosen in
1 𝑤𝑎𝑦
Multiplication Rule
We need to choose
(𝐵𝑙𝑢𝑒 𝑇𝑒𝑎𝑚) & (𝑅𝑒𝑑 𝑇𝑒𝑎𝑚) & (𝐵𝑒𝑛𝑐ℎ𝑒𝑑 𝑃𝑙𝑎𝑦𝑒𝑟)
And this can be done in
11! 6! 11! 10!
× × ⏟
1 = = 11 × = 11 × 252 = 2,772

6! 5! ⏟
1! 5! 5! 5! 5! 5!
𝐵𝑒𝑛𝑐ℎ𝑒𝑑 𝑃𝑙𝑎𝑦𝑒𝑟
𝐵𝑙𝑢𝑒 𝑇𝑒𝑎𝑚 𝑅𝑒𝑑 𝑇𝑒𝑎𝑚

Example 2.85
The basketball coach wants to have a four versus four match among his players. In how many ways can he do
this, if, he has:
A. Eight players
B. Nine Players
Part A
We need to choose four from the eight players available to form the first team, giving us:
8 8! 8×7×6×5
8 𝐶ℎ𝑜𝑜𝑠𝑒 4 = ( ) = = = 70
4 4! 4! 4×3×2
Now, having chosen four players, we are left, by default, with the four players who must form our second team,
giving us
4
( ) = 1 𝑊𝑎𝑦
4
But, we are interested in the match, and the teams are not identifiable. Hence, just as we divide by 2 to count the
number of handshakes, we must divide by 2 here as well.
Hence, the total number of ways is:

P a g e 85 | 100
Get all the files at: https://bit.ly/azizhandouts
Aziz Manva (azizmanva@gmail.com)

70 × 1
= 35
2
Part B
We need to choose four from the nine players available to form the first team, giving us:
9 9! 9×8×7×6
9 𝐶ℎ𝑜𝑜𝑠𝑒 4 = ( ) = = = 126
4 4! 4! 4×3×2
Now, having chosen four players, we are left, by default, with the four players who must form our second team,
giving us
5
( ) = 5 𝑊𝑎𝑦𝑠
4
But, we are interested in the match, and the teams are not identifiable. Hence, just as we divide by 2 to count the
number of handshakes, we must divide by 2 here as well.
Hence, the total number of ways is:
126 × 5
= 315
2

Example 2.86: Restrictions


The sports team coach has a first-string team and a second-string team, each consisting of five players. The
coach has identified three superstars among the ten players, two of whom must be selected in the first-string
team, leaving only one for the second-string team. In how many ways can the coach form:
A. The Second-String Team
B. The First-string Team
C. Both the teams

Strategy
The coach has divided the players into two types:
➢ Superstars
➢ Regular Player

These two are distinct, so a player falls in exactly one of these categories:

3 + ⏟
7 = 10
𝑺𝒖𝒑𝒆𝒓𝒔𝒕𝒂𝒓𝒔 𝑹𝒆𝒈𝒖𝒍𝒂𝒓 𝑷𝒍𝒂𝒚𝒆𝒓𝒔

Part A: Second-String Team


The second-string team has five players, out of which exactly one is a superstar. We must choose
➢ exactly one super star out of the three available superstars
3 3!
=( )= =3
1 1! 2!
➢ exactly four regular players from the seven available regular players
7 7! 7×6×5
=( )= = = 35
4 4! 3! 6

The total number of ways to select the team is:



3 × 35
⏟ = 105
𝑺𝒆𝒍𝒆𝒄𝒕𝒊𝒏𝒈 𝑺𝒆𝒍𝒆𝒄𝒕𝒊𝒏𝒈
𝑺𝒖𝒑𝒆𝒓𝒔𝒕𝒂𝒓 𝑹𝒆𝒈𝒖𝒍𝒂𝒓 𝑷𝒍𝒂𝒚𝒆𝒓𝒔

Part B: First-String Team


The first-string team also has five players, out of which exactly two are superstars. We must choose
➢ exactly two superstars out of the three available superstars:

P a g e 86 | 100
Get all the files at: https://bit.ly/azizhandouts
Aziz Manva (azizmanva@gmail.com)

3 3!
=( )= =3
2 1! 2!
➢ exactly three regular players from the seven available regular players
7 7! 7×6×5
=( )= = = 35
3 4! 3! 6

The total number of ways to select the team is:



3 × 35
⏟ = 105
𝑺𝒆𝒍𝒆𝒄𝒕𝒊𝒏𝒈 𝑺𝒆𝒍𝒆𝒄𝒕𝒊𝒏𝒈
𝑺𝒖𝒑𝒆𝒓𝒔𝒕𝒂𝒓 𝑹𝒆𝒈𝒖𝒍𝒂𝒓 𝑷𝒍𝒂𝒚𝒆𝒓𝒔

The number of ways to select the first-string team is the same as the number of ways to select the second-string
team. Is this surprising?

No. Because
𝑛 𝑛
( )=( )
𝑟 𝑛−𝑟
Part C: Both the Teams
Number of ways to select the
➢ First Team = 105
➢ Second Team = 105

Bogus Solution
You might want to multiply the two numbers giving you
1052 = 11,025
But this would be wrong.

Why?
Because, once you select
➢ five players for the first-string team, the remaining five players are automatically the ones that make up
the second-string team
➢ OR five players for the second-string team, the remaining five players are automatically the ones that
make up the first-string team

Hence, the total number of ways


𝐹𝑖𝑟𝑠𝑡 − 𝑆𝑡𝑟𝑖𝑛𝑔 𝑇𝑒𝑎𝑚 = 𝑆𝑒𝑐𝑜𝑛𝑑 − 𝑆𝑡𝑟𝑖𝑛𝑔 𝑇𝑒𝑎𝑚 = 𝐵𝑜𝑡ℎ 𝑇𝑒𝑎𝑚𝑠 = 105

§2.3.K Repetition

Example 2.87
How many 4-digit numbers greater than 1000 are there that use the four digits of 2012? (AMC 8 2012/10)
Ans = 9

Consider two cases.


First Digit is 2
Remaining digits are 0,1,2 which can be arranged in
3! 𝑤𝑎𝑦𝑠 = 6
First Digit is 1
Remaining digits are 0,2,2. We can only choose the position of the zero, which can be done in

P a g e 87 | 100
Get all the files at: https://bit.ly/azizhandouts
Aziz Manva (azizmanva@gmail.com)

3
( ) 𝑊𝑎𝑦𝑠 = 6
1

𝑇𝑜𝑡𝑎𝑙 = 6 + 3 = 9

Example 2.88
A mathematical organization is producing a set of commemorative license plates. Each plate contains a
sequence of five characters chosen from the four letters in 𝐴𝐼𝑀𝐸 and the four digits in 2007. No character may
appear in a sequence more times than it appears among the four letters in 𝐴𝐼𝑀𝐸 or the four digits in 2007. A set
𝑁
of plates in which each possible sequence appears exactly once contains 𝑁 license plates. Find 10. (AIME
2007/II/1)

We want to choose five digits out of eight, and then we want to arrange them.

Choosing five digits out of eight can be done in:


8 8×7×6
( )= = 56 𝑊𝑎𝑦𝑠
5 6

But the number of ways of arranging these five digits will depend upon whether the digits are repeated or not.
Hence, we proceed based on casework.

Case I: No zero or One Zero is Used


If there is a maximum of one zero, then the five digits can be chosen in
7
( ) = 21 𝑊𝑎𝑦𝑠
5
And they can be arranged in
5! = 120 𝑊𝑎𝑦𝑠
Giving a total of
21 × 120 = 2520
Case II: Two Zeros Used
If there are two zeros, then the two zeroes are always selected, which can be done in only
1 𝑊𝑎𝑦
The remaining three digits can be chosen in
6
( ) = 20 𝑊𝑎𝑦𝑠
3
And they can be arranged in
5!
= 60 𝑊𝑎𝑦𝑠
2!
Giving a total of
20 × 60 = 1200 𝑊𝑎𝑦𝑠
Final Answer
The total ways for both the cases are:
𝑁 3720
𝑁 = 2520 + 1200 = 3720 ⇒ = = 372
10 10

Example 2.89
A mathematical organization is producing a set of commemorative license plates. Each plate contains a
sequence of five characters chosen from:
𝐴𝐼𝑀𝐸 𝑰𝑰 2007

P a g e 88 | 100
Get all the files at: https://bit.ly/azizhandouts
Aziz Manva (azizmanva@gmail.com)

Since the two Roman Characters for 𝑰𝑰 are written in a different font/colour, they are distinguishable from the I
used in AIME. No character may appear in a sequence more times than it appears above. Find the number of
distinct license plates. (AIME 2007/II/1, Adapted)

Zeros Not Zeros Repeated


Repeated
I Not Repeated 𝐴 = 6720 𝐶 = 2100
I Repeated 𝐵 = 2100 𝐷 = 180
8820 2280 11100

Case A
Here we only have 8 digits to pick from (since we cannot repeat the I or the zero).
8
( ) × 5! = 6720
5
Case B
The two I’s can be selected in only
1 𝑊𝑎𝑦
The remaining three digits can be chosen in
7
( ) = 35 𝑊𝑎𝑦𝑠
3
And they can be arranged in
5!
= 60 𝑊𝑎𝑦𝑠
2
Giving a total of
20 × 60 = 2100 𝑊𝑎𝑦𝑠
Case C
The two zeroes can be selected in only
1 𝑊𝑎𝑦
The remaining three digits can be chosen in
7
( ) = 35 𝑊𝑎𝑦𝑠
3
And they can be arranged in
5!
= 60 𝑊𝑎𝑦𝑠
2
Giving a total of
20 × 60 = 2100 𝑊𝑎𝑦𝑠
Case D
The two zeroes and the two I’s can be selected in only
1 𝑊𝑎𝑦
The remaining one digit can be chosen in
6
( ) = 6 𝑊𝑎𝑦𝑠
1
And they can be arranged in
5!
= 30 𝑊𝑎𝑦𝑠
2! 2!
Giving a total of
6 × 30 = 180 𝑊𝑎𝑦𝑠

Example 2.90
Stones are numbered 1, 2, 3, 4, 5, 6, 7, 8, 9, 10. Three groups of stones can be selected so that the sum of each

P a g e 89 | 100
Get all the files at: https://bit.ly/azizhandouts
Aziz Manva (azizmanva@gmail.com)

group is 11. For example, one arrangement is {1, 10}, {2, 3, 6}, {4, 7}. Including the example, how many
arrangements are possible? (CEMC Grade 8 2012-24)

We need to partition the set {1,2,3,4,5,6,7,8,9,10} {1,2,8}{4,7}{5,6}


into four parts with the three of the parts being {1,3,7}{2,9}{5,6}
used, and one part containing the numbers which {1,4,6}{2,9}{3,8}
are not used. {2,3,6}{1,10}{4,7}
{2,4,5}{1,10}{3,8}
To handle this via cases: From the above, we get
Case I: All three sets have exactly two elements 5 𝑐𝑎𝑠𝑒𝑠
1 + 10 = 2 + 9 = 3 + 8 = 4 + 7 = 5 + 6 Remaining Cases
We have five sets. Out of the five sets, we have to Two or more groups of size 3 or more are not
select any three. possible.
5 5! 5×4
( )= = = 10
3 2! 3! 2 Hence, the final answer is:
10 + 5 = 15
Case II: One set has three elements, and other two
sets have two elements

§2.3.L Consecutive

Example 2.91
In how many ways can five friends of different heights stand in a line so that no three consecutive people have
increasing height?

Complementary Arrangements
Let
𝐴 = 𝑝𝑒𝑜𝑝𝑙𝑒 𝑎𝑡 𝑠𝑝𝑜𝑡𝑠 123 𝑎𝑟𝑒 𝑖𝑛 𝑖𝑛𝑐𝑟𝑒𝑎𝑠𝑖𝑛𝑔 ℎ𝑒𝑖𝑔ℎ𝑡 𝑜𝑟𝑑𝑒𝑟
𝐵 = 𝑝𝑒𝑜𝑝𝑙𝑒 𝑎𝑡 𝑠𝑝𝑜𝑡𝑠 234 𝑎𝑟𝑒 𝑖𝑛 𝑖𝑛𝑐𝑟𝑒𝑎𝑠𝑖𝑛𝑔 ℎ𝑒𝑖𝑔ℎ𝑡 𝑜𝑟𝑑𝑒𝑟
𝐶 = 𝑝𝑒𝑜𝑝𝑙𝑒 𝑎𝑡 𝑠𝑝𝑜𝑡𝑠 345 𝑎𝑟𝑒 𝑖𝑛 𝑖𝑛𝑐𝑟𝑒𝑎𝑠𝑖𝑛𝑔 ℎ𝑒𝑖𝑔ℎ𝑡 𝑜𝑟𝑑𝑒𝑟

One at a Time
For A, we need to choose 3 people out of 5, which can be done in
5
( ) 𝑤𝑎𝑦𝑠
3

We can arrange them in height order in exactly one way. The remaining two spots can be filled in
2 × 1 = 2 𝑤𝑎𝑦𝑠
Total number of ways that we can achieve 𝐴 is
5
( ) × 2 = 10 × 2 = 20 𝑊𝑎𝑦𝑠
3

Similarly:
𝑛(𝐴) = 𝑛(𝐵) = 𝑛(𝐶) = 20 𝑊𝑎𝑦𝑠

Two at a Time
𝐴 ∩ 𝐵 is the event where spots 1234 are in height order, which can be done in
5
= ( ) 1! = 5 𝑊𝑎𝑦𝑠
4

P a g e 90 | 100
Get all the files at: https://bit.ly/azizhandouts
Aziz Manva (azizmanva@gmail.com)

𝐵 ∩ 𝐶 is the event where spots 2345 are in height order, which can also be done in
5
= ( ) 1! = 5 𝑊𝑎𝑦𝑠
4

𝐴 ∩ 𝐶 is the event where spots 12345 are in height order, which can be done in
= 1 𝑊𝑎𝑦 𝑜𝑛𝑙𝑦

Three at a Time
𝐴 ∩ 𝐵 ∩ 𝐶 is also the event where spots 12345 are in height order, and this can again be done in
1 𝑊𝑎𝑦

⏟ + 𝑛(𝐵) + 𝑛(𝐶) − [𝑛(𝐴


𝑛(𝐴 ∪ 𝐵 ∪ 𝐶) = 𝑛(𝐴) ⏟ ∩ 𝐵) + 𝑛(𝐵 ∩ 𝐶) + 𝑛(𝐴 ∩ 𝐶)] + 𝑛(𝐴
⏟ ∩ 𝐵 ∩ 𝐶)
𝑶𝒏𝒆 𝒂𝒕 𝒂𝒕 𝒂 𝑻𝒊𝒎𝒆 𝑻𝒘𝒐 𝒂𝒕 𝒂 𝑻𝒊𝒎𝒆 𝑻𝒉𝒓𝒆𝒆 𝒂𝒕 𝒂 𝒕𝒊𝒎𝒆
= 20 + 20 + 20 − (5 + 5 + 1) + 1 = 60 − 11 + 1 = 50

Complementary Counting
The number of valid arrangements is
5! − 50 = 120 − 50 = 70

§2.4 Algebraic Applications


§2.4.A Tossing Coins

Example 2.92
I have a 1794 Flowing Hair Silver Dollar, a highly valuable coin among the first ones issued by the US Mint. The
obverse side has the Bust of Liberty engraved on it. The reverse side has an eagle surrounded by a wreath. I toss
the coin thrice. Count the number of outcomes with:
A. Zero Eagles
B. One Eagles
C. Two Eagles
D. Three Eagles

For the sake of simplicity, we just count Heads and Tails.


Strategy
We have three coin tosses:
⏟ ⏟ ⏟
𝑇𝑜𝑠𝑠 1 𝑇𝑜𝑠𝑠 2 𝑇𝑜𝑠𝑠 3

Zero Heads
Enumeration:
Zero Heads means that all the tosses must be Tails, and this can be done in just one way:
𝑇𝑎𝑖𝑙
⏟ 𝑇𝑎𝑖𝑙⏟ 𝑇𝑎𝑖𝑙 ⏟
𝑇𝑜𝑠𝑠 1 𝑇𝑜𝑠𝑠 2 𝑇𝑜𝑠𝑠 3
Combinations:
This is equivalent to choosing zero out of three objects, which can be done in
3 3!
( )= = 1 𝑊𝑎𝑦
0 0! 3!
One Head
Enumeration:
One Head means that exactly one of the tosses must be Heads and this can be done in three ways:

P a g e 91 | 100
Get all the files at: https://bit.ly/azizhandouts
Aziz Manva (azizmanva@gmail.com)


𝐻𝑒𝑎𝑑 ⏟ ⏟ 𝑂𝑅 ⏟ 𝐻𝑒𝑎𝑑
⏟ ⏟ 𝑂𝑅 ⏟ ⏟ 𝐻𝑒𝑎𝑑
⏟ = 3 𝑊𝑎𝑦𝑠
𝑇𝑜𝑠𝑠 1 𝑇𝑜𝑠𝑠 2 𝑇𝑜𝑠𝑠 3 𝑇𝑜𝑠𝑠 1 𝑇𝑜𝑠𝑠 2 𝑇𝑜𝑠𝑠 3 𝑇𝑜𝑠𝑠 1 𝑇𝑜𝑠𝑠 2 𝑇𝑜𝑠𝑠 3
Combinations:
From the diagram above, of the three tosses we must choose exactly one to be Heads, which is the same as
selecting one object of three, which can be done in:
3 3!
( )= = 3 𝑊𝑎𝑦𝑠
1 1! 2!
Two Heads
Enumeration:
Two Heads means that exactly one of the tosses must be Tails and this can be done in three ways:

𝑇𝑎𝑖𝑙𝑠 ⏟ ⏟ 𝑂𝑅 ⏟ 𝑇𝑎𝑖𝑙𝑠 ⏟ ⏟ 𝑂𝑅 ⏟ ⏟ 𝑇𝑎𝑖𝑙𝑠

𝑇𝑜𝑠𝑠 1 𝑇𝑜𝑠𝑠 2 𝑇𝑜𝑠𝑠 3 𝑇𝑜𝑠𝑠 1 𝑇𝑜𝑠𝑠 2 𝑇𝑜𝑠𝑠 3 𝑇𝑜𝑠𝑠 1 𝑇𝑜𝑠𝑠 2 𝑇𝑜𝑠𝑠 3
Combinations:
From the diagram above, of the three tosses we must choose exactly two to be Heads, which is the same as
selecting two objects out of three, which can be done in:
3 3 3!
( )=( )= = 3 𝑊𝑎𝑦𝑠
2 1 2! 1!
Three Heads
Enumeration:
Three Heads means that all the tosses must be Heads, and this can be done in just one way:
𝐻𝑒𝑎𝑑𝑠
⏟ 𝐻𝑒𝑎𝑑𝑠
⏟ 𝐻𝑒𝑎𝑑𝑠

𝑇𝑜𝑠𝑠 1 𝑇𝑜𝑠𝑠 2 𝑇𝑜𝑠𝑠 3
Combinations:
This is equivalent to choosing three out of three objects, which can be done in
3 3!
( )= = 1 𝑊𝑎𝑦
3 0! 3!

Example 2.93
In the previous example, we calculated the number of outcomes with zero, one, two and three heads when
tossing a coin thrice. Tabulate the outcomes you calculated in the previous example.

No. of Heads
Zero 3 3!
( )= =1
0 0! 3!
One 3 3!
( )= =3
1 1! 2!
Two 3 3!
( )= =3
2 2! 1!
Three 3 3!
( )= =1
3 3! 0!
Total 8 Outcomes

Example 2.94
I toss a coin four times, and record each outcome as it happens. What is the:
A. Total Number of Outcomes
B. Outcomes with Zero, One, Two, Three and Four Heads respectively.

For total outcomes, we use the idea of objects with repetition. We have four tosses, and in each toss, we have
two choices:

P a g e 92 | 100
Get all the files at: https://bit.ly/azizhandouts
Aziz Manva (azizmanva@gmail.com)

𝑇𝑜𝑡𝑎𝑙 𝑂𝑢𝑡𝑐𝑜𝑚𝑒𝑠 = ⏟
2 × ⏟
2 × ⏟
2 × ⏟
2 = 24 = 16
𝑇𝑜𝑠𝑠 1 𝑇𝑜𝑠𝑠 2 𝑇𝑜𝑠𝑠 3 𝑇𝑜𝑠𝑠 4

No. of Heads
Zero 4 4!
( )= =1
0 0! 4!
One 4 4!
( )= =4
1 1! 3!
Two 4 4!
( )= =6
2 2! 2!
Three 4 4!
( )= =4
3 3! 1!
Four 4 4!
( )= =1
4 4! 0!
Total 16 Outcomes

Example 2.95
I toss a coin five times, and record each outcome as it happens. What is the:
A. Total Number of Outcomes
B. Outcomes with Zero, One, Two, Three, Four and Five Heads respectively.

For total outcomes, we use the idea of objects with repetition. We have four tosses, and in each toss, we have
two choices:
𝑇𝑜𝑡𝑎𝑙 𝑂𝑢𝑡𝑐𝑜𝑚𝑒𝑠 = 25 = 32

No. of Heads
Zero 5 5!
( )= =1
0 0! 5!
One 5 5!
( )= =5
1 1! 4!
Two 5 5!
( )= = 10
2 2! 3!
Three 5 5!
( )= = 10
3 3! 2!
Four 5 5!
( )= =5
4 4! 1!
Five 5 5!
( )= =1
5 5! 0!
Total 32 Outcomes

2.96: Tossing A Coin: Number of Heads and Total Outcomes


I toss a coin 𝑛 times:
𝑇𝑜𝑡𝑎𝑙 𝑜𝑢𝑡𝑐𝑜𝑚𝑒𝑠 = 2𝑛
𝑛
𝑂𝑢𝑡𝑐𝑜𝑚𝑒𝑠 𝑤𝑖𝑡ℎ 𝑚 ℎ𝑒𝑎𝑑𝑠: ( )
𝑚

Of course, we need to put the condition that:


0≤𝑚≤𝑛
Else, the number of outcomes will be zero.

P a g e 93 | 100
Get all the files at: https://bit.ly/azizhandouts
Aziz Manva (azizmanva@gmail.com)

§2.4.B Rolling Dice

Example 2.97
I toss five eight-sided dice. In how many ways can exactly four of the dice show a two.

I want four of the dice to show a two.


𝑇𝑤𝑜
⏟, 𝑇𝑤𝑜
⏟, 𝑇𝑤𝑜
⏟, 𝑇𝑤𝑜
⏟, 𝑁𝑜𝑡 𝑇𝑤𝑜 ⇒ 1 𝑊𝑎𝑦

𝐷𝑖𝑒 1 𝐷𝑖𝑒 2 𝐷𝑖𝑒 3 𝐷𝑖𝑒 4 𝐷𝑖𝑒 5

However, the roll which is not two, could be in any position.


Hence, we need to multiply by
5

Further, the number which is not two, can be anything from


{1,3,4,5,6,7,8} ⇒ 7 𝐶ℎ𝑜𝑖𝑐𝑒𝑠
Hence, we need to multiply by
7

Hence, the final answer


1 × 5 × 7 = 35

(Combinations) Example 2.98


I toss five eight-sided dice. In how many ways can exactly four of the dice show a two.

5 7
( ) × ( ) = 5 × 7 = 35
4 1

Example 2.99
I toss nine eight-sided dice. In how many ways can exactly seven of the dice show a six.

9 8 9! 9×8
( )×( ) = ×8= × 8 = 288
7 1 2! 7! 2!
§2.4.C Choosing Cards

Example 2.100
A. I draw three cards from a standard pack of cards. In how many ways can exactly two of the cards be Red.
B. I draw four cards from a standard pack of cards. In how many ways can exactly one of them be an Ace.
C. I draw five cards from a standard pack of cards. In how many ways can exactly two of them be Black
D. I draw ten cards from a standard pack of cards. In how many ways can exactly three of them be Spades.

3 3!
( )= =3
2 1! 2!
4
( )=4
1
5
( ) = 10
2
10 10 × 9 × 8
( )= = 120
3 6

Example 2.101
P a g e 94 | 100
Get all the files at: https://bit.ly/azizhandouts
Aziz Manva (azizmanva@gmail.com)

I draw ten cards from a standard pack of cards. In how many ways can exactly six of the cards be Kings if the
cards are drawn:
A. Without replacement
B. With replacement

Part A: Without Replacement


There are only four kings.
And we want six.

Hence, without replacement, we can do this in


𝑍𝑒𝑟𝑜 𝑊𝑎𝑦𝑠

Part B: With Replacement


10 10! 10 × 9 × 8 × 7
( )= = = 210
6 4! 6! 24
D. Set Theory

Example 2.102
(Formula): A set has n elements. What is the number of subsets that have r elements?
By definition, order is not important in a set.
The number of subsets with r elements
= Number of ways of selecting r elements out of n elements
𝑛!
= 𝑛𝐶𝑟 =
𝑟! (𝑛 − 𝑟)!

Example 2.103: Subsets of a Set


𝐴 = {𝑥: 0 < 𝑥 < 10, 𝑥 ∈ 𝑁}
List the elements of X in roster form and then find the number of subsets of A that contain
A. Three elements (which are all odd)
B. Three elements (which are all even)
C. Two elements (which are both prime)
D. Five numbers
A = {1, 2, 3, 4, 5, 6, 7, 8, 9}
5! 5×4
A. Selecting three elements ⏟ out of five = 5𝐶3 = = = 10
{1,3,5,7,9}
2! (3)! 2
4!
B. Selecting three even numbers ⏟ out of four = 4𝐶3 = =4
{2,4,6 8}
1! (3)!
4! 4×3
C. Selecting two prime numbers ⏟ out of four = 4𝐶2 = = =6
{2,3,5,7}
2! (2)! 2
9! 9×8×7×6
𝐷. Selecting five numbers out of all nine numbers = 9𝐶5 = = = 126
5! (4)! 4×3×2

2.104: Sum of Combinations: Combinatorial


𝒏
𝑪𝟎 + 𝒏𝑪𝟏 + ⋯ + 𝒏𝑪𝒏 = 𝟐𝒏

Count the number of ways of selecting zero or more balls from 𝑛 balls in two ways.

P a g e 95 | 100
Get all the files at: https://bit.ly/azizhandouts
Aziz Manva (azizmanva@gmail.com)
𝑛 𝑛 𝑛
Combinations: ⏟ 𝐶0 + ⏟ 𝐶1 + ⋯ + ⏟ 𝐶𝑛 = 𝐿𝐻𝑆
𝑆𝑒𝑙𝑒𝑐𝑡 0 𝑆𝑒𝑙𝑒𝑐𝑡 1 𝑆𝑒𝑙𝑒𝑐𝑡 𝑛
𝐵𝑎𝑙𝑙𝑠 𝐵𝑎𝑙𝑙 𝐵𝑎𝑙𝑙𝑠

Multiplication Principle: ⏟
2 × ⏟
2 2 = 2𝑛 = 𝑅𝐻𝑆
× …× ⏟
𝐹𝑖𝑟𝑠𝑡 𝑆𝑒𝑐𝑜𝑛𝑑 𝑛𝑡ℎ
𝐵𝑎𝑙𝑙 𝐵𝑎𝑙𝑙 𝐵𝑎𝑙𝑙
But, the number of ways counted both ways must be the same.
∴ 𝐿𝐻𝑆 = 𝑅𝐻𝑆
E. Sums of Subsets

Example 2.105
The set {1,2,3,4} has 𝑛 subsets. Let 𝑠𝑚 be the sum of the elements of the 𝑚𝑡ℎ subset. Find 𝑛 + 𝑠1 + 𝑠2 + ⋯ + 𝑠𝑛 .

𝑛 = 24 = 16
We do this using cases.
0 Element Sets
{𝜙} ⇒ 𝑆𝑢𝑚 = 0
1 Element Sets
{1}, {2}, {3}, {4} ⇒ 𝑆𝑢𝑚 = 1 + 2 + 3 + 4 = 10
2 Element Sets
{1,2}, {1,3}, {1,4}, {2,3}, {2,4}, {3,4}
Note that each element of the original set occurs twice. Hence, the sum is:
2(1 + 2 + 3 + 4) = 2(10) = 20
3 Element Sets
{1,2,3}, {1,2,4}, {1,3,4}, {2,3,4}
Compare this with:
{1,2,3, 𝟒}, {1,2, 𝟑, 4}, {1, 𝟐, 3,4}, {𝟏, 2,3,4}
And hence the sum we want is equal to:
4(1 + 2 + 3 + 4) − 1(1 + 2 + 3 + 4) = 30
4 Element Sets
{1,2,3,4} ⇒ 𝑆𝑢𝑚 = 10

And hence, the final answer is:


0 + 10 + 20 + 30 + 10 = 70

Example 2.106

§2.5 Geometric Applications


§2.5.A: Geometrical Problems

Example 2.107: Tiling/Parity


A "domino" is made up of two small square. Which of the "checkerboards" illustrated below CANNOT be
covered exactly and completely by a whole number of non-overlapping dominoes? (AMC 8 1991/5)
Ans = 3 * 5

Example 2.108
Points R, S and T are vertices of an equilateral triangle, and points X, Y and Z are midpoints of its sides. How
many noncongruent triangles can be drawn using any three of these six points as vertices? (AMC 8 2001/23)

P a g e 96 | 100
Get all the files at: https://bit.ly/azizhandouts
Aziz Manva (azizmanva@gmail.com)

Ans = 4

Example 2.109
How many different patterns can be made by shading exactly two of the nine squares? Patterns that can be
matched by flips and/or turns are not considered different. For example, the patterns shown below are not
considered different. (AMC 8 1990/25)
Ans = 8

Example 2.110: Triangle Inequality


How many different isosceles triangles have integer side lengths and perimeter 23? (AMC 8 2005/15)
Ans = 6

Example 2.111: Intersections


A circle and two distinct lines are drawn on a sheet of paper. What is the largest possible number of points of
intersection of these figures? (AMC 8 2002/1)
Ans = 5

§2.5.B: Points on a circle

Example 2.112: Making Triangles

§2.5.C: Three-Dimensional Geometry

Example 2.113: Making Triangles


How many distinct triangles can be constructed by connecting three different vertices of a cube? (Two triangles
are distinct if they have different locations in space.) (MathCounts 1996 National Sprint)

8 8×7×6
( )= = 56 𝑇𝑟𝑖𝑎𝑛𝑔𝑙𝑒𝑠
3 6

§2.5.D: Dots on Two Parallel Lines

Example 2.114: Making Triangles


How many distinct triangles can be drawn using three of the dots below as vertices? (AMC 8 2005/21)
Ans = 18

How many non-congruent triangles have vertices at three of the eight points in the array shown below? (AMC 8
2009/20)
Ans = 8

Example 2.115
In the diagram, line A has seven points:
(1,2)(2,2), … (7,2)
And line B has nine points:
(1,1)(2,1), … (9,1)
Not all of the points are shown. Using the
points, how many

P a g e 97 | 100
Get all the files at: https://bit.ly/azizhandouts
Aziz Manva (azizmanva@gmail.com)

A. Distinct triangles with non-zero area can be made?


B. Squares can be made?
C. Rectangles can be made?
D. Parallelograms can be made?
E. Quadrilaterals can be made?

In total, you can make


Part A
We need three vertices to make a triangle. 6
Case I: If all three vertices are on the same line, then Part C
we get a triangle with zero area, which does not If you choose a point on the top row to be the top
meet the condition given in the question. left corner of the rectangle, there has to be a 90°
Case II: If one vertex is on line A, and the other two angle created with the bottom row. This is only
vertices are on line B. This gives us: possible when the point on the bottom row is
7 9 exactly below the point on the top row.
( ) ( ) = 7 × 36 = 252
1 2 Mathematically, this means that the 𝑥 coordinate of
Case III: One vertex is on line B, and the other two the bottom point is decided as soon as you pick the
vertices are on line A. This gives us: top point.
7 9
( ) ( ) = 21 × 9 = 189
2 1
Total Ways You also need to choose the top right corner of the
= 252 + 189 = 441 rectangle.
Hence, the final numbers of choices
7 7×6
We can also get the same answer using =( )= = 21
complementary counting 2 2
The total number of ways to choose 3 points out of Part D
7 + 9 = 16 points is: Parallelograms of side length 1 for the top and
16 bottom row will be:
( )
3 6 × 8 = 48

7 𝑆𝑖𝑑𝑒
But, we do not want the ( ) ways where the three 𝐿𝑒𝑛𝑔𝑡ℎ 1
3
9 Similarly, parallelograms of other side lengths for
vertices are all on line A, and the ( ) where the
3 the top and bottom row will be
three vertices are all on line B. And, hence, the final 5×7 + ⏟
⏟ 4×6 + ⏟ 3×5 + ⏟ 2×4 + ⏟ 1×3
answer is: 𝑆𝑖𝑑𝑒 𝑆𝑖𝑑𝑒 𝑆𝑖𝑑𝑒 𝑆𝑖𝑑𝑒 𝑆𝑖𝑑𝑒
16 7 9 𝐿𝑒𝑛𝑔𝑡ℎ 2 𝐿𝑒𝑛𝑔𝑡ℎ 3 𝐿𝑒𝑛𝑔𝑡ℎ 4 𝐿𝑒𝑛𝑔𝑡ℎ 5 𝐿𝑒𝑛𝑔𝑡ℎ 6
( )−( )−( )
3 3 3
Simplifying and adding, we get the total as
Part B
= 48 + 35 + 24 + 15 + 8 + 3 = 133
Part E
Direct Method
7 9
( )( )
2 2
Complementary Counting
The only square that you can make is a square of 16 7 9 9 7 7 9
( ) − ( ) − ( ) −( )( ) − ( )( )
side length 1. 4 4 4 1 3 1 3

§2.5.E: Points on a Plane

Example 2.116
From n distinct points in a plane, what is the maximum number of:

P a g e 98 | 100
Get all the files at: https://bit.ly/azizhandouts
Aziz Manva (azizmanva@gmail.com)

A. Triangles
B. Circles
C. Points of Intersection of the circles

§2.5.F: Chessboard Squares


A standard chessboard consists of a grid of squares comprising eight rows and eight columns. Many questions
in counting and probability are based on chessboards. (The questions below do not need knowledge of chess,
they just make use of the “board”).

Example 2.117: Making Squares


See the grid on the chessboard. The largest square that can be formed is
8 × 8 ⇒ 𝑇ℎ𝑒 𝑒𝑛𝑡𝑖𝑟𝑒 𝑐ℎ𝑒𝑠𝑠𝑏𝑜𝑎𝑟𝑑 ⇒ 1 𝑆𝑞𝑢𝑎𝑟𝑒
The smallest square that can be formed is
1 × 1 ⇒ 64 𝑆𝑞𝑢𝑎𝑟𝑒𝑠
In such a manner, we can make squares:
(1 × 1), (2 × 2), … (8 × 8)
A. Find the number of squares of size 8, 7 and 6 respectively.
B. Find a pattern for the number of squares that can be formed.
C. What is the total number of squares that can be formed?
(Squares must be aligned to the grid points. They cannot be placed halfway
along a square from the existing grid).

Part A
Squares of Size 8
This is straightforward. It’s the maximum size, and you can fit only one square of that size on the chessboard.
Squares of Size 7
Consider the bottom left corner of the square
𝐶𝑜𝑙𝑢𝑚𝑛𝑠: 𝐶𝑜𝑙𝑢𝑚𝑛 𝐴, 𝑜𝑟 𝐶𝑜𝑙𝑢𝑚𝑛 𝐵 ⇒ 2 𝐶ℎ𝑜𝑖𝑐𝑒𝑠
𝑅𝑜𝑤𝑠: 𝑅𝑜𝑤 1 𝑜𝑟 𝑅𝑜𝑤 2 ⇒ 2 𝐶ℎ𝑜𝑖𝑐𝑒𝑠
𝑇𝑜𝑡𝑎𝑙 𝐶ℎ𝑜𝑖𝑐𝑒𝑠 = 2 × 2 = 4
Squares of Size 6
Consider the bottom left corner of the square
𝐶𝑜𝑙𝑢𝑚𝑛𝑠: 𝐶𝑜𝑙𝑢𝑚𝑛 𝐴, 𝐶𝑜𝑙𝑢𝑚𝑛 𝐵, 𝐶𝑜𝑙𝑢𝑚𝑛 𝐶 ⇒ 3 𝐶ℎ𝑜𝑖𝑐𝑒𝑠
𝑅𝑜𝑤𝑠: 𝑅𝑜𝑤 1, 𝑅𝑜𝑤 2 𝑜𝑟 𝑅𝑜𝑤 3 ⇒ 3 𝐶ℎ𝑜𝑖𝑐𝑒𝑠
𝑇𝑜𝑡𝑎𝑙 𝐶ℎ𝑜𝑖𝑐𝑒𝑠 = 3 × 3 = 9
Part B
1 + 4 + 9 + 16 + ⋯ + 64
Part C
We want to find

P a g e 99 | 100
Get all the files at: https://bit.ly/azizhandouts
Aziz Manva (azizmanva@gmail.com)

𝑛(𝑛+1)(2𝑛+1)
Recall that 1 + 22 + ⋯ + 𝑛2 = and we
can use it to find:
6
8(9)(17)
𝑛 = 8 ⇒ 1 + 2 2 + 32 + 42 + ⋯ + 82 = = 204
6

Example 2.118
An 8 by 8 checkerboard has alternating black and white squares. How many distinct
squares, with sides on the grid lines of the checkerboard (horizontal and vertical) and
containing at least 4 black squares, can be drawn on the checkerboard? (MathCounts
2003 Chapter Team)

6(7)(13)
𝑛 = 6 ⇒ 1 + 22 + ⋯ + 62 = = 91
6
§2.5.G: Chessboard Rectangles

Example 2.119: Making Rectangles


Just as we can form squares on a chessboard, so we can form rectangles also.
Count the number of rectangles that can be formed on a chessboard
(Rectangles must be aligned to the grid points. They cannot be placed
halfway along a square from the existing grid).

The chessboard is
8 𝑏𝑦 8
And hence it actually has
9 𝐿𝑖𝑛𝑒𝑠 𝑏𝑦 9 𝐿𝑖𝑛𝑒𝑠

We can choose two distinct horizontal lines in:


9
( ) = 36 𝑊𝑎𝑦𝑠
2
And we can choose two distinct vertical lines in:
9
( ) = 36 𝑊𝑎𝑦𝑠
2
And to make a rectangle, we need to choose two distinct horizontal lines, and two distinct vertical lines. These
will intersect at four places, giving us the corners of a rectangle. This can be done in:
36 × 36 = 62 × 62 = 64 = 1296 𝑊𝑎𝑦𝑠

§2.5.H: Lattice Grids

Example 2.120: Making Squares

Example 2.121: Making Rectangles

§2.6 Equations/Further Topics


§2.6.A: Equations

122 Examples

P a g e 100 | 100

You might also like